60
1 Revisão Geral de Termologia Prof. Marcos Guimarães Sampaio (só prá moçadinha) Termometria 01) (ITA SP) O verão de 1994 foi particularmente quente nos Estados Unidos da América. A diferença entre a máxima temperatura do verão e a mínima no inverno anterior foi de 60 °C. Qual o valor dessa diferença na escala Fahrenheit? a) 108 °F b) 60 °F c) 140 °F d) 33 °F e) 92 °F Resolução: ∆C/5 = ∆F/9 60/5 = ∆F/9 ∆F = 108 °F (Resposta A) 02) (MACK SP) Uma pessoa mediu a temperatura de seu corpo, utilizando-se de um termômetro graduado na escala Fahrenheit, e encontrou o valor 97,7 o F. Essa temperatura, na escala Celsius, corresponde a: a) 36,5 o C b) 37,0 o C c) 37,5 o C d) 38,0 o C e) 38,5 o C Resolução: (F 32)/9 = C/5 (97,7 32)/9 = C/5 C = 36,5 ºC (Resposta A) 03) (UFPI - PI) O Aquecimento Global é um fenômeno climático de larga extensão. As previsões mais catastróficas para a região Amazônica incluem o desaparecimento completo da floresta se a temperatura média da região tiver um aumento superior aos 5ºC. Com isso a temperatura média anual da cidade de Manaus passaria a ser de 33ºC, que lida na escala Kelvin corresponderia a: a) 300 K b) 310 K c) 290 K d) 306 K e) 302 K Resolução: T(K) = C + 273 T(K) = 33 + 273 = 306 K. (Resposta D) 04) (IFAL - AL) Uma pessoa que estava viajando para Londres, no dia seguinte, procura se informar a respeito das condições do tempo no local de destino. Num canal de TV a cabo, toma conhecimento de que a temperatura naquela cidade e de 26,6°F, pois, na Inglaterra, costuma-se usar termômetros graduados na escala Fahrenheit. Baseando-se nessa informação, o viajante chegara a seguinte conclusão: a) Terá que levar pesados agasalhos para suportar uma temperatura abaixo do zero absoluto. b) Com uma temperatura alem dos 36°C, não devera se preocupar com agasalhos. c) Levara roupas leves, pois a temperatura esta entre 27°C e 36° C. d) Como a temperatura esta abaixo de 0°C, providenciara roupas adequadas a um inverno rigoroso. e) Usará suas roupas normais, pois as temperaturas nas duas escalas são iguais para todos os valores. Resolução: (F 32)/9 = C/5 (26,6 32)/9 = C/5 C = 3 ºC. (Resposta D) 05) (UFPE - PE) As escalas de temperatura mais conhecidas são Celsius (ºC) e Fahrenheit (ºF). Nessas escalas, o ponto de congelamento da água corresponde a 0ºC e 32ºF, e o ponto de ebulição corresponde a 100ºC e 212ºF. A equivalência entre as escalas é obtida por uma função polinomial do 1º grau, ou seja, uma função da forma f(x) = ax + b, em que f(x) é a temperatura em grau Fahrenheit (ºF) e x a temperatura em grau Celsius (ºC). Se em um determinado dia a temperatura no centro do Recife era de 29ºC, a temperatura equivalente em grau Fahrenheit (ºF) era de: a) 84ºF b) 84,02ºF c) 84,1ºF d) 84,12ºF e) 84,2ºF Resolução: (F 32)/9 = C/5 (F 32)/9 = 29/5 F = 84,2ºF (Resposta E) 06) (PUC RS) Podemos caracterizar uma escala absoluta de temperatura quando: a) dividimos a escala em 100 partes iguais. b) associamos o zero da escala ao estado de energia cinética mínima das partículas de um sistema. c) associamos o zero da escala ao estado de energia cinética máxima das partículas de um sistema. d) associamos o zero da escala ao ponto de fusão do gelo. e) associamos o valor 100 da escala ao ponto de ebulição da água.

Revisão geral-de-termologia

Embed Size (px)

Citation preview

1

Revisão Geral de

Termologia

Prof. Marcos Guimarães

Sampaio

(só prá moçadinha)

Termometria

01) (ITA – SP) O verão de 1994 foi

particularmente quente nos Estados Unidos da

América. A diferença entre a máxima

temperatura do verão e a mínima no inverno

anterior foi de 60 °C. Qual o valor dessa

diferença na escala Fahrenheit?

a) 108 °F

b) 60 °F

c) 140 °F

d) 33 °F

e) 92 °F

Resolução: ∆C/5 = ∆F/9 60/5 = ∆F/9

∆F = 108 °F (Resposta A)

02) (MACK – SP) Uma pessoa mediu a

temperatura de seu corpo, utilizando-se de um

termômetro graduado na escala Fahrenheit, e

encontrou o valor 97,7 oF. Essa temperatura, na

escala Celsius, corresponde a:

a) 36,5 oC

b) 37,0 oC

c) 37,5 oC

d) 38,0 oC

e) 38,5 oC

Resolução: (F – 32)/9 = C/5 (97,7 –

32)/9 = C/5 C = 36,5 ºC (Resposta A)

03) (UFPI - PI) O Aquecimento Global é um

fenômeno climático de larga extensão. As

previsões mais catastróficas para a região

Amazônica incluem o desaparecimento

completo da floresta se a temperatura média da

região tiver um aumento superior aos 5ºC. Com

isso a temperatura média anual da cidade de

Manaus passaria a ser de 33ºC, que lida na

escala Kelvin corresponderia a:

a) 300 K

b) 310 K

c) 290 K

d) 306 K

e) 302 K

Resolução: T(K) = C + 273 T(K) = 33

+ 273 = 306 K. (Resposta D)

04) (IFAL - AL) Uma pessoa que estava

viajando para Londres, no dia seguinte, procura

se informar a respeito das condições do tempo

no local de destino. Num canal de TV a cabo,

toma conhecimento de que a temperatura

naquela cidade e de 26,6°F, pois, na Inglaterra,

costuma-se usar termômetros graduados na

escala Fahrenheit. Baseando-se nessa

informação, o viajante chegara a seguinte

conclusão:

a) Terá que levar pesados agasalhos para

suportar uma temperatura abaixo do zero

absoluto.

b) Com uma temperatura alem dos 36°C, não

devera se preocupar com agasalhos.

c) Levara roupas leves, pois a temperatura esta

entre 27°C e 36° C.

d) Como a temperatura esta abaixo de 0°C,

providenciara roupas adequadas a um inverno

rigoroso.

e) Usará suas roupas normais, pois as

temperaturas nas duas escalas são iguais para

todos os valores.

Resolução: (F – 32)/9 = C/5 (26,6 –

32)/9 = C/5 C = – 3 ºC. (Resposta D)

05) (UFPE - PE) As escalas de temperatura

mais conhecidas são Celsius (ºC) e Fahrenheit

(ºF). Nessas escalas, o ponto de congelamento

da água corresponde a 0ºC e 32ºF, e o ponto de

ebulição corresponde a 100ºC e 212ºF.

A equivalência entre as escalas é obtida por uma

função polinomial do 1º grau, ou seja, uma

função da forma f(x) = ax + b, em que f(x) é a

temperatura em grau Fahrenheit (ºF) e x a

temperatura em grau Celsius (ºC). Se em um

determinado dia a temperatura no centro do

Recife era de 29ºC, a temperatura equivalente

em grau Fahrenheit (ºF) era de:

a) 84ºF

b) 84,02ºF

c) 84,1ºF

d) 84,12ºF

e) 84,2ºF

Resolução: (F – 32)/9 = C/5 (F –

32)/9 = 29/5 F = 84,2ºF (Resposta E)

06) (PUC – RS) Podemos caracterizar uma

escala absoluta de temperatura quando:

a) dividimos a escala em 100 partes iguais.

b) associamos o zero da escala ao estado de

energia cinética mínima das partículas de um

sistema.

c) associamos o zero da escala ao estado de

energia cinética máxima das partículas de um

sistema.

d) associamos o zero da escala ao ponto de

fusão do gelo.

e) associamos o valor 100 da escala ao ponto de

ebulição da água.

2

Resolução: Zero absoluto corresponde ao

estado de agitação mínimo das partículas o que

significa energia cinética mínima. (Resposta B)

07) Imagine dois termômetros graduados. Um

na escala Celsius e o outro na escala Fahrenheit.

Sendo assim, responda às questões seguintes:

a) Quando o termômetro Celsius acusar uma

variação de temperatura igual a 20 ºC determine

a variação de temperatura acusada por um

termômetro Fahrenheit e por um termômetro

Kelvin.

b) Quando um termômetro Celsius acusar uma

temperatura de 20 ºC, qual será a temperatura

acusada, no mesmo local por um termômetro

Fahrenheit e por um Kelvin.

Resolução:

a) ∆C/5 = ∆F/9 20/5 = ∆F/9 ∆F =

36 ºF e ∆T(K) = ∆C ∆T(K) = 20 K.

b) (F – 32)/9 = C/5 (F – 32)/9 = 20/5

F = 68ºF e T(K) = C + 273 = 20 + 273

= 293 K. (Respostas: a) 36 ºF e 20 K; b) 68ºF

e 293 K)

08) (UFRPE - PE) Duas escalas termométricas,

ºX e ºY, têm suas respectivas temperaturas, TX e

TY, relacionadas pela expressão 2TX – 3TY + 2 =

0. Pode-se afirmar que uma variação de

temperatura de 30 ºX corresponde, na escala Y,

a uma variação de:

a) 10 ºY

b) 20 ºY

c) 30 ºY

d) 40 ºY

e) 50 ºY

Resolução: Para cada 1ºX de variação teremos,

em ºY: 2.1 – 3TY + 2 = 0 TY = 4/3 ºY.

assim, para uma variação de 30 ºX teremos

30.4/3 = 40ºY. (Resposta: D)

09) Três afirmações são feitas a respeito de

temperatura.

I) Zero absoluto equivale à temperatura em que

ocorre o congelamento da água.

II) Podemos obter água a - 350 ºC.

III) Quando um corpo tem sua temperatura

variando 50 ºC pode-se dizer que, se a escala

fosse a Kelvin, a variação de temperatura,

numericamente, seria a mesma.

a) Apenas I é falsa

b) Apenas II é falsa

c) Apenas III é falsa

d) Apenas III é verdadeira

e) As três afirmações são falsas.

Resolução:

I- (F) O congelamento da água sob pressão

normal ocorre a 273 K.

II- (F) A menor temperatura teoricamente

possível é o 0 K ou – 273 ºC.

III- (V) Já que o intervalo entre os pontos fixos

de fusão do gelo e ebulição da água são os

mesmos.

(Resposta D)

10) (ACAFE – SC) Três blocos de metais

diferentes foram aquecidos de forma contínua e

suas temperaturas medidas, instante após

instante, por três termômetros, simultaneamente.

Um, na escala Celsius, outro na escala

Fahrenheit e o terceiro na escala Kelvin.

A figura abaixo mostra três gráficos da

temperatura versus tempo, T X t,

confeccionados a partir dos dados obtidos. As

curvas de aquecimento, em cada gráfico, foram

obtidas por termômetros diferentes.

Analisando os gráficos, é correto o que se

afirma em:

a) No gráfico C, a curva de aquecimento 1

corresponde às medidas realizadas pelo

termômetro na escala Fahrenheit e a curva de

aquecimento 2 corresponde às medidas

realizadas pelo termômetro na escala Celsius.

b) No gráfico A, a curva de aquecimento 2

corresponde às medidas realizadas pelo

termômetro na escala Celsius e a curva de

aquecimento 1 corresponde às medidas

realizadas pelo termômetro na escala Kelvin.

c) No gráfico B, a curva de aquecimento 2

corresponde às medidas realizadas pelo

termômetro na escala Fahrenheit e a curva de

aquecimento 1 corresponde às medidas

realizadas pelo termômetro na escala Celsius.

d) No gráfico B, a curva de aquecimento 1

corresponde às medidas realizadas pelo

termômetro na escala Fahrenheit e a curva de

aquecimento 2 corresponde às medidas

realizadas pelo termômetro na escala Kelvin.

e) No gráfico C, a curva de aquecimento 1

corresponde às medidas realizadas pelo

termômetro na escala Kelvin e a curva de

aquecimento 2 corresponde às medidas

realizadas pelo termômetro na escala

Fahrenheit.

Resolução: A partir do zero absoluto, a relação

gráfica entre as escalas é a seguinte:

-273

0

-459,4

F

C

K

3

a) F: o encontro entre as escalas ocorre a -40.

b) V: a distância numérica entre as escalas será

sempre igual a 100 unidades.

c) F: como está explicado no item anterior

d) F: a curva para a escala Kelvin inicia no 0

(zero absoluto).

e) F: como está explicado no item anterior.

(Resposta B)

11) (CEFET – PR) A superfície gelada do

pequeno Plutão é composta por nitrogênio,

metano e traços de monóxido de carbono. A

temperatura do planeta anão varia ao longo de

sua órbita porque, no decorrer de sua trajetória,

aproxima-se do Sol até 30 UA e afasta-se até 50

UA. Existe uma tênue atmosfera que congela e

cai sobre o planeta anão quando este se afasta

do Sol. Sendo assim, dependendo da sua

posição em relação ao Sol, a temperatura sobre

a superfície do planeta anão varia de –230ºC a –

210ºC. Pode-se afirmar que: (UA = Unidade

Astronômica)

a) essas temperaturas não são lidas num

termômetro graduado na escala Kelvin, pois a

menor temperatura nesse termômetro é 0 K.

b) não se medem essas temperaturas num

termômetro graduado na escala Celsius, pois sua

escala varia de 0ºC a 100ºC.

c) se medem essas temperaturas com

termômetros graduados na escala Celsius, pois é

o único que mede temperaturas abaixo de zero.

d) na escala Fahrenheit, o módulo da variação

da temperatura sobre a superfície do pequeno

Plutão corresponde a 36ºF.

e) na escala Fahrenheit, o módulo da variação

da temperatura sobre a superfície do pequeno

Plutão corresponde a 20ºF.

Resolução: ∆C/5 = ∆F/9 (-210 – (-230))/5 =

∆F/9 ∆F = 36 ºF. (Resposta D)

12) (ITA – SP) Para medir a febre de pacientes,

um estudante de medicina criou sua própria

escala linear de temperaturas.

Nessa nova escala, os valores de 0 (zero) e 10

(dez) correspondem respectivamente a 37°C e

40°C. A temperatura de mesmo valor numérico

em ambas escalas é aproximadamente

a) 52,9 °C. b) 28,5 °C. c) 74,3 °C.

d) –8,5°C. e) –28,5°C.

Resolução: (t – t1)/(t2 – t1) = (C – C1)/(C2 – C1)

(t – 0)/(10 – 0) = (C – 37)(40 – 37) (x –

0)/(10 – 0) = (x – 37)(40 – 37) x/10 = (x –

37)/3 x 28,5 ºC. (Resposta B)

13) (MACK – SP) Joãozinho, seguindo as

orientações de seu professor de Física, construiu

uma nova escala termométrica. Ao nível do mar,

ele atribuiu o valor -20 °J para a temperatura do

gelo fundente e 130 °J para a temperatura de

ebulição da água. A medida, que nessa escala

tem valor coincidente com o da escala Celsius,

refere-se à temperatura:

a) 20 °J

b) 30 °J

c) 40 °J

d) 50 °J

e) 60 °J

Resolução: (t – t1)/(t2 – t1) = (C – C1)/(C2 – C1)

(t – (-20))/(130 – (-20)) = (C – 0)/(100 – 0)

(x + 20)/150 = x/100 x = 40. (Resposta

C)

14) (MACK – SP) Um profissional,

necessitando efetuar uma medida de

temperatura, utilizou um termômetro cujas

escalas termométricas inicialmente impressas ao

lado da coluna de mercúrio estavam ilegíveis.

Para atingir seu objetivo, colocou o termômetro

inicialmente numa vasilha com gelo fundente,

sob pressão normal, e verificou que no

equilíbrio térmico a coluna de mercúrio atingiu

8,0 cm. Ao colocar o termômetro em contato

com água fervente, também sob pressão normal,

o equilíbrio térmico se deu com a coluna de

mercúrio atingindo 20,0 cm de altura. Se nesse

termômetro utilizarmos as escalas Celsius e

Fahrenheit e a temperatura a ser medida for

expressa pelo mesmo valor nas duas escalas, a

coluna de mercúrio terá altura de:

a) 0,33 cm

b) 0,80 cm

c) 3,2 cm

d) 4,0 cm

e) 6,0 cm

Resolução: A temperatura comum nas escalas

Celsius e Fahrenheit é: (F – 32)/9 = C/5

(x – 32)/9 = x/5 x = -40

Resposta C

15) (UFMS – MS) Através de experimentos,

biólogos observaram que a taxa de canto de

grilos de uma determinada espécie estava

relacionada com a temperatura ambiente de uma

maneira que poderia ser considerada linear.

Experiências mostraram que, a uma temperatura

de 21º C, os grilos cantavam, em média, 120

vezes por minuto; e, a uma temperatura de 26º

C, os grilos cantavam, em média, 180 vezes por

minuto. Considerando T a temperatura em graus

0

100

8

20

-40 y

(0 – (-40))/(100 – (-40)) = (8 –

y)/(20 – y) y = 3,2 cm

4

Celsius e n o número de vezes que os grilos

cantavam por minuto, podemos representar a

relação entre T e n pelo gráfico abaixo.

Supondo que os grilos estivessem cantando, em

média, 156 vezes por minuto, de acordo com o

modelo sugerido nesta questão, estima-se que a

temperatura deveria ser igual a

a) 21,5 ºC .

b) 22 ºC .

c) 23 ºC .

d) 24 ºC .

e) 25,5 ºC .

Resolução:

Resposta D

16) (UNIFESP – SP) O texto a seguir foi

extraído de uma matéria sobre congelamento de

cadáveres para sua preservação por muitos anos,

publicada no jornal “O Estado de S. Paulo” de

21.07.2002:

Após a morte clínica, o corpo é resfriado com

gelo. Uma injeção de anticoagulantes é

aplicada e um fluido especial é bombeado para

o coração, espalhando-se pelo corpo e

empurrando para fora os fluidos naturais. O

corpo é colocado numa câmara com gás

nitrogênio, onde os fluidos endurecem em vez

de congelar. Assim que atinge a temperatura de

-321°, o corpo é levado para um tanque de

nitrogênio líquido, onde fica de cabeça para

baixo.

Na matéria, não consta a unidade de

temperatura usada. Considerando que o valor

indicado de -321° esteja correto e que pertença a

uma das escalas (Kelvin, Celsius ou

Fahrenheit), pode-se concluir que foi usada a

escala:

a) Kelvin, pois se trata de um trabalho científico

e esta é a unidade adotada pelo Sistema

Internacional.

b) Fahrenheit, por ser um valor inferior ao zero

absoluto e, portanto, só pode ser medido nessa

escala.

c) Fahrenheit, pois as escalas Celsius e Kelvin

não admitem esse valor numérico de

temperatura.

d) Celsius, pois só ela tem valores numéricos

negativos para a indicação de temperaturas.

e) Celsius, por tratar-se de uma matéria

publicada em língua portuguesa e essa ser a

unidade adotada oficialmente no Brasil.

Resolução: A menor temperatura teoricamente

possível é o 0 K ou – 273 ºC ou -459,4 ºF.

Portanto a única escala que se enquadra com o

valor de temperatura descrito no texto é a

Fahrenheit. (Resposta C)

17) Uma escala termométrica X é construída de

modo que a temperatura de 0 °X corresponde a

– 4 °F, e a temperatura de 100 °X corresponde a

68°F. Nesta escala X, a temperatura de fusão do

gelo vale:

a) 10 °X b) 20 °X c) 30 °X

d) 40 °X e) 50 °X

Resolução:

18) Um termômetro de mercúrio é calibrado

com o ponto de gelo a 2cm de altura da coluna

de mercúrio e o ponto de vapor a 12cm.

Obtenha uma função que relacione a

temperatura T(°C) com a altura h (cm) e

determine a temperatura T(ºC) quando h = 10

cm.

Resolução:

19) Os pontos de fusão do gelo e de ebulição da

água na escala Fahrenheit são, respectivamente,

32 °F e 212 °F. Um termômetro A, graduado na

escala Fahrenheit, e outro B, graduado na escala

Celsius, são colocados simultaneamente em um

frasco contendo água quente.

Verifica-se que o termômetro A apresenta uma

leitura que supera em 80 unidades a leitura do

termômetro B. Podemos afirmar que a

temperatura da água no frasco é:

a) 60 °C b) 80 °C c) 112 °C

d) 50 °F e) 112 °F

Resolução: F = C + 80 e : (F – 32)/9 = C/5

(C + 80 – 32)/9 = C/5 C = 60 ºC

(Resposta A)

x

26

156

180

21 120

(x – 21)/(26 – 21) = (156 –

120)/( 180 – 120) (x –

21)/5 = 36/60 x = 24 ºC

x

100

32

68

0 -4

(x – 0)/(100 – 0)) = (32 – (-4)/(68

– (-4) x/100 = 36/72 x =

50. (Resposta E)

C

100

x

12cm

0 2cm

(C – 0)/(100 – 0)) = (x – 2)/12 –

2) C/100 = (x – 2)/10 C =

10x – 20 (Resposta: T(ºC) = 10

T(ºX) – 20)

5

20) (MACK – SP) Um termômetro defeituoso

está graduado na escala Fahrenheit, indicando

30 °F para o ponto de gelo e 214 °F para o

ponto de vapor. Neste termômetro, a única

temperatura medida corretamente, corresponde

a:

a) 0 °F b) 30 °F c) 40 °F

d) 50 °F e) 122 °F

21). Buscando satisfazer uma diversificada

clientela, algumas operadoras de turismo

lançaram a “viagem surpresa”: o viajante

compra um pacote para um local desconhecido,

só sabendo seu destino ao desembarcar. A única

informação é dada alguns dias antes da viagem

e refere-se à temperatura média no local a ser

visitado. Numa dessas viagens, um passageiro

recebeu o seguinte comunicado: a temperatura

média local à época da viagem é 82ºF. Esse

passageiro deve:

a) levar chapéu, filtro solar, bermudas e roupas

leves.

b) levar agasalhos leves, pois o clima é ameno.

c) precaver-se com capotes e agasalhos de lã,

pois nessa temperatura há possibilidade de

encontrar neve.

d) desistir de viajar, pois a temperatura é tão

baixa que dificilmente conseguirá sair do hotel.

Resolução: Comparando com a escala Celsius,

temos:

(F – 32)/9 = C/5 (82 – 32)/9 = C/5

C = 87,7 ºF. (Resposta: A)

22) Um estudante construiu um termômetro,

adotando uma escala termométrica obtida do

seguinte modo:

- mergulhou uma haste metálica em gelo

fundente. Algum tempo depois, constatou que a

medida da haste era de 20 cm. A esta medida

atribuiu o valor de 50 graus Ypsolon (50 ºY);

- em seguida, mergulhou a mesma haste

metálica em um recipiente contendo água em

ebulição. Algum tempo depois, constatou que a

haste, dilatada, passou a medir 20,3 cm. A esta

medida atribuiu o valor de 80 graus Ypsolon (80

ºY).

Sendo assim, responda às questões a

seguir:

a) Ao mergulhar a haste metálica, por algum

tempo, na água contida em um recipiente, seu

comprimento se torna igual a 20,2 cm.

Determine, em graus Ypsolon, a temperatura da

água no recipiente.

b) A temperatura de 90 ºY está associada a que

medida de comprimento da haste metálica ?

Resolução: A equação de conversão será (y –

50)/(80 – 50) = (L – 20)/(20,3 – 20) (y –

50)/30 = (L – 20)/0,3 y – 50 = 100L – 2000

a) y – 50 = 100.20,2 – 2000 y = 70 ºY.

b) 90 – 50 = 100L – 2000 L = 20,4 cm.

(Respostas: a) 70 ºY; b) 20,4 cm)

23) (ENEM) Nos processos industriais, como na

indústria de cerâmica, é necessário o uso de

fornos capazes de produzir elevadas

temperaturas e, em muitas situações, o tempo de

elevação dessa temperatura deve ser controlado,

para garantir a qualidade do produto final e a

economia do processo

Em uma indústria de cerâmica o forno é

programado para elevar a temperatura ao longo

do tempo de acordo com a função em que T é o

valor da temperatura atingida pelo forno, em

graus Celsius, e t é o tempo, em minutos,

decorrido desde o instante em que o forno é

ligado.

Uma peça deve ser colocada nesse forno quando

a temperatura for 48 oC e retirada quando a

temperatura for 200 °C.

O tempo de permanência dessa peça no forno é,

em minutos, igual a

a) 100 b)108 c) 128 d) 130 e) 150

Resolução:

x

214

x

212

30 32

(x – 30)/(214 – 30) = (x –

32)/(212 – 32) (x – 30) 184 =

(x – 32)/180 x = 122 ºF

(Resposta: E)

6

Temperatura inicial do forno T(t0) = 7/5.t0 + 20

T(0) =7/5.0 + 20 = 20 ºC.

A temperatura do forno será 48 ºC (momento

em que a peça será introduzida em: T(t) = 7/5.t

+ 20 48 = 7/5.t + 20 t1 = 20 min

Após 100 minutos a temperatura da água será:

T(t2) = 7/5.t2 + 20 T(100) = 7/5.100 + 20 =

160 ºC.

Após isso, atingirá a temperatura de 200 ºC

quando: T(t3) = 2/125t3² - 16/5t3 + 320 200 =

2/125t3² - 16/5t3 + 320 t3 = 50 min.

Assim a peça permanecerá no forno durante t =

(t2 – t1) + t3 = (100 – 20) + 50 = 130 min.

(Resposta D)

Dilatação Térmica

24) (UFES – ES) Uma barra de metal tem

comprimento igual a 10,000 m a uma

temperatura de 10,0 °C e comprimento igual a

10,006 m a uma temperatura de 40 °C. O

coeficiente de dilatação linear do metal é:

a) 1,5 × 10-4

°C-1

b) 6,0 × 10-4

°C-1

c) 2,0 × 10-5

°C-1

d) 2,0 × 10-4

°C-1

e) 3,0 × 10-4

°C-1

Resolução: ∆L = L0.α.∆ 0,006 = 10. α.30

α = 2.10–5

ºC–1

(Resposta C)

25) (UFU – MG) Uma ponte de aço tem 1 000m

de comprimento. O coeficiente de dilatação

linear do aço é de 11. 10-6

°C–1

. A expansão da

ponte, quando a temperatura sobe de 0 para

30°C, é de:

a) 33cm.

b) 37cm.

c) 41cm.

d) 52cm.

e) 99cm.

Resolução: ∆L = L0.α.∆ ∆L = 1000. 11.

10-6

.30 = 0,33m ou 33 cm (Resposta A)

26) (MACK – SP) Duas barras metálicas, de

diferentes materiais, apresentam o mesmo

comprimento a 0 °C. Ao serem aquecidas, à

temperatura de 100 °C, a diferença entre seus

comprimentos passa a ser de 1 mm. Sendo 2,2.

10–5

°C–1

o coeficiente de dilatação linear do

material de uma barra e 1,7.10–5

°C–1

o do

material da outra, o comprimento dessas barras

a 0 °C era:

a) 0,2 m

b) 0,8 m

c) 1,0 m

d) 1,5 m

e) 2,0 m

Resolução: ∆L = L0.α1.∆ – L0.α2.∆

1.10-3

= L0. 2,2. 10–5

.100 – L0. 1,7.10–5

.100

L0 = 2,0 m (Resposta E)

27) (UERJ – RJ) Uma torre de aço, usada para

transmissão de televisão, tem altura de 50 m

quando a temperatura ambiente é de 40 0C.

Considere que o aço dilata-se, linearmente, em

média, na proporção de 1/100.000, para cada

variação de 1 0C.

À noite, supondo que a temperatura caia para 20 0C, em valor absoluto a variação de

comprimento da torre, em centímetros, será de:

a) 1,0

b) 1,5

c) 2,0

d) 2,5

Resolução:

∆L = L0.α.∆ ∆L=50.( 1/100.000).(-20) =

1/100 m ou 1 cm. (Resposta A)

28) Entre dois trilhos consecutivos de uma via

férrea, deixa-se um espaço apenas suficiente

para facilitar livremente a dilatação térmica dos

trilhos de 0 ºC até a temperatura de 70 ºC. O

coeficiente de dilatação térmica linear do

material dos trilhos é 1,0. 10–5

°C–1

. Cada trilho

mede 20m a 0 °C. Qual o espaço entre dois

trilhos consecutivos na temperatura de 0 °C?

Resolução: ∆L = L0.α.∆ ∆L = 20.

1,0. 10–5

.70 = 0,014m = 1,4 cm., ou seja, 0,7 cm

para cada lado. Desse modo, entre doius trilhos

sucessivos devermos ter d = 1,4 cm. (Resposta:

1,4 cm)

29) Uma telha de alumínio tem dimensões

lineares de 20cm x 500cm e seu coeficiente de

dilatação linear é igual a 2,2 x 10–5

ºC–1

. A telha,

ao ser exposta ao sol durante o dia, experimenta

uma variação de temperatura de 20 ºC. A

dilatação superficial máxima da chapa, em cm²,

durante esse dia, será.

a) 1,1

b) 2,2

c) 4,4

d) 6,6

e) 8,8

Resolução: ∆A = A0.β.∆ ∆A =

(20.500).(2. 2,2 x 10–5

).(20) = 8,8 cm².

(Resposta E)

30) Para se ligar estrutura metálicas em prédios

usa-se a técnica de rebitagem em que para se

colocarem os rebites é preferível que:

a) eles estejam à mesma temperatura da chapa;

b) eles estejam à temperatura superior a da

chapa, geralmente aquecidos ao rubro;

c) eles estejam resfriados a temperaturas abaixo

da chapa;

7

d) qualquer das possibilidades acima ocorre

desde que fiquem bem colocados

Resolução: A uma temperatura inferior a da

chapa, o diâmetro do rebite será menor que o da

chapa permitindo a introdução do rebite.

Quando a temperatura se elevar, o rebite estará

perfeitamente encaixado. (Resposta C)

31) (FCC – SP) Uma peça sólida tem uma

cavidade cujo volume vale 8cm³ a 20ºC. A

temperatura da peça varia para 920 ºC e o

coeficiente de dilatação linear do sólido (12.10–6

ºC–1

) pode ser considerado constante. Supondo

que a pressão interna da cavidade seja sempre

igual à externa, a variação percentual do volume

da cavidade foi de:

a) 1,2%.

b) 2,0%.

c) 3,2%.

d) 5,8%.

e) 12%.

Resolução: ∆V = V0.γ. ∆ sendo γ = 3α. Assim

∆V = V0.(3. 12.10–6

).900 = 0,0324. V0. Portanto

3,24/100 V0 = 3,24% V0.(Resposta C)

32) (UFPI – PI) Muitos sistemas de

aquecimento elétrico em que é necessário

manter certa temperatura, tais como estufas,

fornos, ferro com graduações apropriadas para

passar diversos tipos de tecidos, usam

termostato para manter sua temperatura mais ou

menos constante. Esses reguladores de

temperatura têm o mesmo princípio de

funcionamento. De uma forma simplificada,

pode-se dizer que eles possuem uma espiral

bimetálica como mostrada na figura abaixo.

Considere essa espiral bimetálica formada por

latão na parte interna e de aço na parte externa.

Com relação ao adequado funcionamento destes

sistemas reguladores de temperatura, coloque

V, para verdadeiro, ou F, para falso.

1 ( ) À medida que a temperatura do sistema de

aquecimento elétrico vai caindo, a espiral

bimetálica se contrai, e sua extremidade se

move para direita, isto é, para fora, abrindo o

contato elétrico.

2 ( ) À medida que a temperatura da espiral

bimetálica aumenta, o latão se dilata mais que o

aço e, quando a temperatura atinge o valor

máximo necessário, a espiral se move para

direita ou para fora, desligando o circuito.

3 ( ) Quando a temperatura da espiral aumenta,

além da necessária para o respectivo sistema de

aquecimento, o latão se dilata menos que o aço,

e a espiral se move para direita ou para fora,

desligando o circuito.

4 ( ) Quando a temperatura do sistema diminui,

o aço se contrai mais que o latão, e a espiral

bimetálica fecha novamente o circuito, até o

sistema atingir a temperatura máxima da

graduação do sistema de aquecimento.

Resolução: Observe que: internamente: latão

(2,0.10–5

) e externamente: aço (1,3.10–5

).

1- (F) pois o latão sofrerá maior contração

térmica que o aço, já que seu coeficiente de

dilatação é maior.

2- (V) pois o latão sofrerá maior dilatação.

3- (F) No aquecimento o latão se dilata mais que

o aço. Seu coeficiente é maior.

4- (F) no resfriamento o latão contrai mais que o

aço. (Resposta: F; V; F; F)

33) (IFCE – CE) Uma barra de aço de

comprimento L0 = 0,700 m é encaixada

praticamente sem folga em uma cavidade

lubrificada, como mostra a figura.

A temperatura inicial da barra é de 20,0 ºC e seu

coeficiente = 12,0 x 10-

6 ºC

-1.

Quando uma das extremidades da barra é

mantida fixa e a outra é puxada por uma força

T, seu comprimento se altera de acordo com a

equação L = L0 (1 + T/f), com f = 6,00 x 107 N

(t < 0 se T comprime a barra). Supondo-se que a

cavidade não se deforma, a força horizontal que

a barra exerce sobre a parede da cavidade à

direita, para uma temperatura de 22,0ºC, é igual

a

a) 1220 N

b) 1420 N

c) 1620 N

d) 1520 N

e) 1440 N

Resolução: L0 = 70cm e ∆L será ∆L = L0.α.∆

= 70. 12,0 x 10-6

.(22 – 20) = 168.10-5

= 0,00168

cm L = 70,00168cm. A força será L = L0 (1

8

+ T/f) 70,00168 = 70(1 + T/6.107) t =

1440 N. (Resposta E)

34) (IFAL – AL) Um fato bem conhecido

levado em conta por técnicos, engenheiros,

cientistas e outros profissionais ao se

defrontarem com a tarefa de construir algum

equipamento, projetar uma construção, ou

mesmo efetuar um experimento científico, é que

as dimensões de um corpo aumentam quando

aumentamos sua temperatura.

Ressalvando algumas exceções, este é um

comportamento geral dos sólidos, líquidos e

gases. Sobre este fenômeno, denominado

dilatação térmica, analise as afirmativas a

seguir:

I. Toda dilatação térmica, em verdade, se

processa nas três dimensões espaciais:

comprimento, largura e altura.

II. A variação das dimensões de um corpo

depende, exclusivamente, do material de que ele

é feito e da sua variação de temperatura sofrida.

III. No caso dos líquidos, observamos que a

dilatação real deles depende do recipiente no

qual estão inseridos e é sempre menor que a

dilatação aparente observada.

Levando-se em conta os conceitos

cientificamente corretos, das afirmações acima

está(ao) correta(s):

a) apenas I.

b) apenas II.

c) apenas III.

d) I e II.

e) I, II e III.

Resolução:

I- (V) As dilatações ocorrem em todas as

direções.

II- (F) Depende também de suas dimensões

iniciais, já que a dilatação também é

proporcional a ela.

III- (F) A dilatação do real de um líquido não

depende da dilatação do frasco. Apenas a

aparente depende. (Resposta A)

35) (ACAFE – SC) O fenômeno da dilatação

térmica pode ser usado para manter um sistema

numa temperatura aproximadamente constante

através de um dispositivo chamado termostato.

Usa-se uma lâmina bimetálica (duas hastes de

materiais diferentes, porém com mesmo

comprimento inicial) que se encurva à medida

que a temperatura muda e que pode abrir ou

fechar circuitos elétricos, ou válvulas, e assim

manter o sistema, como um ferro elétrico, um

forno ou um sistema de ar condicionado em

uma temperatura prédeterminada.

A figura abaixo mostra esquematicamente um

dispositivo desse tipo onde, ao aumentar a

temperatura da lâmina, ela se curva para

_______ se o coeficiente de dilatação da haste

A for _____ coeficiente de dilatação da haste B.

A alternativa correta que completa as lacunas

acima, em seqüência, é:

a) baixo - menor que o

b) cima - maior que o

c) cima - menor que o

d) baixo - igual ao

e) cima - igual ao

Resolução: Se αA > αB e no caso de

aquecimento a lâmina se curva para cima

fechando o circuito. Portanto: cima; maior que

o. (Resposta B)

36) (ACAFE – SC) Em um laboratório de física,

um grupo de alunos aquece uma barra metálica

homogênea, A, de comprimento L0, a partir de

uma temperatura inicial T0

e mede o

comprimento da barra para cada grau de

aumento da temperatura. Um segundo grupo

esfria uma barra idêntica, B, a partir da mesma

temperatura inicial T0 e mede o comprimento da

barra para cada grau de diminuição da

temperatura. Depois de encerradas as medidas,

os alunos colocam os dados em um mesmo

gráfico L x T, onde L representa o comprimento

das barras A e B, e T representa a temperatura

correspondente.

A alternativa que apresenta o gráfico correto é:

Resolução: Se as barras são idênticas, para cada

grau de variação de temperatura, a dilatação será

igual a contração. Portanto. (Resposta D)

37) (MACK – SP) Um conjunto de certos

dispositivos para laboratório, proveniente dos

Estados Unidos, contém dentre outros produtos,

uma pequena lâmina retangular. No livreto de

informações, existe a recomendação para não

expor essa lâmina a temperaturas inferiores a 30

ºF e tampouco superiores a 85 ºF. Um estudante

9

brasileiro, interessado em saber de que material

era constituída a lâmina, descobriu, através de

um ensaio, que, durante um aquecimento nesse

intervalo de temperatura, a área da lâmina

aumentava de 0,165%. Consultando a tabela

abaixo, pode-se afirmar que, possivelmente, a

lâmina é de material

Material

Coeficiente de

dilatação linear ( oC

-

1)

Ferro 12.10-6

Ouro 15.10-6

Bronze 18.10-6

Alumínio 22.l0-6

Chumbo 27.10-6

a) ferro.

b) ouro.

c) bronze.

d) alumínio.

e) chumbo.

Resolução: ∆ = 55 ºF = 30,55ºC e ∆A =

A0.β.∆ 0,165% A0 = A0.β.30,55 β =

5,4 . 10-5

e como β = 2α temos α =

27.10-5

ºC-1

(chumbo). (Resposta E)

38) (CEFET – PR) Analise as assertivas abaixo

e marque (V) verdadeiro ou (F) falso.

1- ( ) A dilatação real de um líquido será maior

do que a dilatação aparente observada.

2- ( ) Entre 0ºC e 4ºC há um aumento da

temperatura da água e aumento também da sua

densidade.

3- ( ) O vidro temperado Pirex pode ser levado

diretamente ao fogo sem trincar porque seu

coeficiente de dilatação é maior que o do vidro

comum.

4- ( ) Ao aquecermos uma aliança de ouro, 18

quilates, ocorrerá dilatação em todas as

dimensões, inclusive para a região interna da

aliança.

5- ( ) Três gases diferentes, com o mesmo

volume e estando a mesma temperatura inicial,

se forem aquecidos de forma idêntica e

mantidos a uma pressão constante, apresentarão

o mesmo coeficiente de dilatação volumétrica.

A alternativa correta é:

a) V, F, F, V, F

b) F, F, V, V, F

c) V, V, F, F, V

d) F, F, F, V, F

e) V, V, F, V, F

Resolução:

1-(V) O coeficiente de dilatação de um líquido

é, normalmente, maior que o do frasco. Assim a

dilatação real é maior que a aparente.

2-(V) Ocorrerá uma contração em seu volume e

conseqüentemente em sua densidade.

3-(F) ele não trinca por sofrer pouquíssima

dilatação em virtude do seu baixo coeficiente.

4-(F) ocorrerá um aumento no diâmetro da

aliança.

5-(V) Já que o mesmo depende do volume

inicial e da natureza do gás. (Resposta C)

39) (UFC – CE) Numa experiência de

laboratório, sobre dilatação superficial, foram

feitas várias medidas das dimensões de uma

superfície S de uma lâmina circular de

vidro em função da temperatura T. Os

resultados das medidas estão representados no

gráfico abaixo.

Com base nos dados experimentais fornecidos

no gráfico, pode-se afirmar corretamente que o

valor numérico do coeficiente de dilatação

linear do vidro é:

a) 24x10–6

oC

–1.

b) 18x10–6

oC

–1.

c) 12x10–6

oC

–1.

d) 9x10–6

oC

–1.

e) 6x10–6

oC

–1

Resolução: ∆A = A0.β.∆ (25,00180 –

25,00000) = 25. β.(34 – 30) β = 18,1.10–6

e

sendo β = 2α temos α = 0,05.10–6

ºC–1

(Resposta D)

40) (UFRN – RN) O dispositivo mostrado na

figura abaixo é utilizado em alguns laboratórios

escolares, para determinar o coeficiente de

dilatação linear de um sólido. Nesse dispositivo,

o sólido tem a forma de um tubo de

comprimento L0, inicialmente a temperatura

ambiente, no qual se faz passar vapor de água

em ebulição até que o tubo atinja a temperatura

do vapor ao entrar em equilíbrio térmico com

este. Há, no dispositivo, dois termômetros, TE1

e TE2, e um micrômetro, MI.

Face ao acima exposto, é correto afirmar que,

para a determinação do coeficiente de dilatação

linear desse tubo,

10

a) tanto o termômetro TE1 como o TE2 medem

a variação de temperatura do tubo, e o

micrômetro mede o comprimento inicial do

tubo.

b) o termômetro TE1 mede a temperatura

ambiente, o termômetro TE2 mede a

temperatura do vapor, e o micrômetro mede a

variação de comprimento do tubo.

c) o termômetro TE1 mede a temperatura do

vapor, o termômetro TE2 mede a temperatura

ambiente, e o micrômetro mede o comprimento

final do tubo.

d) tanto o termômetro TE1 como o TE2 medem

a variação de temperatura do tubo, e o

micrômetro mede a variação de comprimento do

tubo

Resolução: Os termômetros indicarão a

variação de temperatura do tubo em relação a do

ambiente e o micrômetro medirá a dilatação do

tubo. (Resposta E)

41) (UEBA – BA) Uma peça de zinco é

construída a partir de uma chapa quadrada de

lado 30cm, da qual foi retirado um pedaço de

área de 500cm². Elevando-se de 50°C a

temperatura da peça restante, sua área final, em

cm², será mais próxima de: Dado: coeficiente de

dilatação linear do zinco = 2,5 . 10–5

°C–1

a) 400

b) 401

c) 405

d) 408

e) 416

Resolução: A peça restante terá área inicial de

30.30 – 500 = 400 cm². A dilatação será ∆A =

A0.β.∆ ∆A = 400.(2. 2,5 . 10–5

).50 = 1cm²

e assim a área final da peça será 401 cm².

(Resposta B)

42) (UNESP – SP) Duas lâminas metálicas, a

primeira de latão e a segunda de aço, de mesmo

comprimento à temperatura ambiente, são

soldadas rigidamente uma à outra, formando

uma lâmina bimetálica, conforme a figura.

O coeficiente de dilatação térmica linear do

latão é maior que o do aço. A lâmina bimetálica

é aquecida a uma temperatura acima da

ambiente e depois resfriada até uma temperatura

abaixo da ambiente. A figura que melhor

representa as formas assumidas pela lâmina

bimetálica, quando aquecida (forma à esquerda)

e quando resfriada (forma à direita), é

Resolução: Ao ser aquecido, o latão dilata mais

que o aço e ao ser resfriado contrai mais.

(Resposta C)

43) (CESGRANRIO – RJ) Um bloco de certo

metal tem seu volume dilatado de 200 cm³ para

206 cm³ quando sua temperatura aumenta de 20

ºC para 520 ºC. Se um fio desse mesmo metal,

tendo 100 cm de comprimento a 20ºC for

aquecido até a temperatura de 520 ºC, então seu

comprimento em centímetros passará a valer:

a) 101.

b) 102.

c) 103.

d) 106.

e) 112.

Resolução: ∆V = V0.γ. ∆ (206 – 200) =

2000.(3α).(520 – 20) α = 2.10–5

°C–1

e ∆L =

L0.α.∆ ∆L = 100. 2.10–5

.500 = 1cm.

Assim L = 101 cm. (Resposta A)

44) (UFMT – MT – Modificado) Uma peça

retangular maciça de alumínio encaixa-se

perfeitamente em um buraco feito em outra

chapa de alumínio. Analise as afirmativas e dê

como resposta a soma dos números que

antecedem as afirmações corretas:

(01) Se a peça e a chapa forem aquecidas até

uma mesma temperatura, a peça passará a não

se encaixar perfeitamente no buraco da chapa.

(02) Se somente a chapa for aquecida, a peça se

encaixará com folga no buraco da chapa.

(04) Se somente a chapa for resfriada, a peça

passará a não se encaixar no buraco da chapa.

(08) Se somente a peça for resfriada, então ela

passará a se encaixar com folga no buraco da

chapa.

(16) Se somente a peça for aquecida, então ela

passará a não se encaixar no buraco da chapa.

Resolução:

01 (F) A dilatação diametral será a mesma.

02 (V) Pois o diâmetro da chapa irá aumentar.

04 (V) Pois seu diâmetro será reduzido em

relação ao da peça.

08 (V) Pois seu diâmetro se tornará menor que o

da chapa.

16 (V) Seu diâmetro se tornará maior que o da

chapa.

11

(Resposta: 30 (02 + 04 + 08 + 16))

45) (ACAFE – SC) Um fio, de coeficiente linear

= 2x10-5

ºC-1, tem comprimento igual a

160cm. Esse fio é dobrado de modo a formar

um quadrado e suas extremidades são soldadas.

Após sofrer um aumento de temperatura de

250ºC, a área interna desse quadrado: 2.

b) 2. 2.

2.

Resolução: A área inicial do quadrado (lado

160 cm/4 = 40 cm) será 40.40 = 1600 cm². O

perímetro do novo quadrado será: ∆L = L0.α.∆

∆L = 160. 2.10–5

.250 ∆L = 0,8 cm. (L =

160,8/4 = 40,2 cm. A nova área do quadrado

fica 40,2² = 1616,04 cm². Assim o acréscimo de

área é 1616,04 – 1600 = 16,04 cm².(Resposta

B)

46) (UNILASALLE – SP) Uma técnica

empregada por algumas indústrias para

aumentar a pressão sobre parafusos de fixação é

a de utilizar parafusos com bitola levemente

superior à do orifício de fixação. Nesse caso,

aquece-se a placa onde será fixado o parafuso.

Isso faz com que o orifício se dilate e o parafuso

possa ser colocado. Após o sistema atingir a

temperatura ambiente, o parafuso fica

praticamente preso ao orifício. Suponha que um

parafuso de aço de diâmetro 1,0000 cm deve ser

fixado num orifício circular de 0,9978 cm de

diâmetro, em uma placa também de aço. A

variação mínima na temperatura da placa, para

que o parafuso possa ser fixado, é a seguinte

(dado: αaço = 11.10-6

ºC-1

)

a) 400 ºC.

b) 200 ºC.

c) 100 ºC.

d) 75 ºC.

e) 50 ºC.

Resolução: ∆L = L0.α.∆ (0,9978 – 1,0000)

= 0,9978. 11.10-6

. ∆ ∆ = -200 ºC.

(Resposta B)

47) (UFPI – PI) O comportamento de dilatação

anômalo da água é verificado pelo fato de esta

se contrair quando sua temperatura aumenta de

0 °C a 4 °C. A partir desta temperatura, a água

se dilata como a maioria das substâncias.

Com relação a esse assunto, coloque V, para

verdadeiro, ou F, para falso.

1 ( ) Em regiões em que a temperatura atinge

valores abaixo de 0°C, os lagos ficam

congelados a partir da superfície. Isso ocorre

porque a água da superfície, ao se resfriar até

4°C, atinge sua densidade máxima, e por

convecção, vai para o fundo.

2 ( ) Numa região de clima muito frio, onde a

temperatura ambiente chega a 0°C ou menos, a

água mais fria por ser mais densa, desce para o

fundo do lago, ocorrendo o congelamento

inicialmente no fundo.

3 ( ) Numa região de clima muito frio, quando a

temperatura das superfícies dos lagos se tornam

inferior a 4°C, não ocorrerá mais movimentação

da água por diferença de densidade. Assim,

forma-se gelo na superfície quando a

temperatura ambiente atingir 0°C, sendo que a

água do fundo do lago continuará líquida.

4 ( ) Quando a temperatura ambiente de regiões

muito frias (onde a temperatura chega abaixo de

0°C) está subindo, o nível da água dos lagos

subirá somente a partir do momento em que a

temperatura da água passar de 4°C.

Resolução:

1 (V) a água mais densa tende a ir para o fundo.

2(F) no fundo a água encontra-se a 4 ºC.

Portanto o congelamento ocorrerá

primeiramente na superfície.

3(V). A água mais densa e mais quente

permanecerá no fundo.

4(V) A dilatação da água torna-se normal após 4

ºC .

(Respostas: 1(V); 2(F); 3(V); 4(V).

48) (ENEM) Durante uma ação de fiscalização

em postos de combustíveis, foi encontrado um

mecanismo inusitado para enganar o

consumidor. Durante o inverno, o responsável

por um posto de combustível compra álcool por

R$ 0,50/litro, a uma temperatura de 5 °C. Para

revender o líquido aos motoristas, instalou um

mecanismo na bomba de combustível para

aquecê-lo, para que atinja a temperatura de 35

°C, sendo o litro de álcool revendido a R$ 1,60.

Diariamente o posto compra 20 mil litros de

álcool a 5 ºC e os revende.

Com relação à situação hipotética descrita no

texto e dado que o coeficiente de dilatação

volumétrica do álcool é de 1×10-3

ºC-1

,

desprezando-se o custo da energia gasta no

aquecimento do combustível, o ganho

financeiro que o dono do posto teria obtido

devido ao aquecimento do álcool após uma

semana de vendas estaria entre

a) R$ 500,00 e R$ 1.000,00.

b) R$ 1.050,00 e R$ 1.250,00.

c) R$ 4.000,00 e R$ 5.000,00.

d) R$ 6.000,00 e R$ 6.900,00.

e) R$ 7.000,00 e R$ 7.950,00.

Resolução: Em 1 semana temos 20000L.7 =

140000L. A dilatação será: ∆V = V0.γ. ∆ =

140000. 1×10-3

. (35 – 5) ∆V = 4200 L. O

ganho financeiro será 4200. R$ 1,10 =

R$4620,00. (Resposta C)

12

49) A gasolina é vendida por litro, mas em sua

utilização como combustível, a massa é o que

importa. Um aumento da temperatura do

ambiente leva a um aumento no volume da

gasolina. Para diminuir os efeitos práticos dessa

variação, os tanques dos postos de gasolina são

subterrâneos. Se os tanques não fossem

subterrâneos:

I. Você levaria vantagem ao abastecer o carro na

hora mais quente do dia pois estaria comprando

mais massa por litro de combustível.

II. Abastecendo com a temperatura mais baixa,

você estaria comprando mais massa de

combustível para cada litro.

III. Se a gasolina fosse vendida por kg em vez

de por litro, o problema comercial decorrente da

dilatação da gasolina estaria resolvido.

Destas considerações, somente

a) I é correta.

b) II é correta.

c) III é correta.

d) I e II são corretas.

e) II e III são corretas.

Resolução:

I(F) o combustível estaria dilatado.

II(V) o combustível estaria comprimido.

III(V) PIS dilatado ou comprimido a massa do

combustível não se alteraria com a dilatação.

(Resposta E)

50) (ITA – SP) Um pequeno tanque,

completamente preenchido com 20,0 litros de

gasolina a 0°F, é logo a seguir transferido para

uma garagem mantida à temperatura de 70°F.

Sendo = 0,0012 °C–1

o coeficiente de

expansão volumétrica da gasolina, a alternativa

que melhor expressa o volume de gasolina em

litros que vazará em conseqüência do seu

aquecimento até a temperatura da garagem é

a) 0,507 b) 0,940 c) 1,68

d) 5,07 e) 0,17

Resolução: ∆ = (5/9).70 = 38,88 ºC. Assim

∆V = V0.γ. ∆ = 20.0,0012.38,88 0,940 L.

(Resposta B)

51) (PUC – RS) Um vendedor de gasolina

colocou 20,0 x 10³ litros de gasolina no tanque

de seu caminhão, à temperatura de 15,0 oC.

Supondo que ele tenha vendido toda a gasolina

à temperatura de 35,0 oC, e que o coeficiente de

dilatação volumétrica dessa gasolina seja igual a

1,00 x 10-3

oC

-1, o acréscimo de volume, em

litros, devido à expansão térmica, foi

a) 20

b) 40

c) 100

d) 200

e) 400

Resolução: ∆V = V0.γ. ∆ = 20.10³. 1,00 x 10-3

.

(35 – 15) = 400 L. (Resposta E)

52) (UFF-RJ) O dono de um posto de gasolina

consulta uma tabela de coeficientes de dilatação

volumétrica obtendo γálcool = 10–3

°C–1

.

Assim, ele verifica que se comprar 14 000 litros

do combustível em um dia em que a

temperatura do álcool é de 20°C e revendê-los

num dia mais quente, em que esta temperatura

seja de 30°C, estará ganhando:

a) 1,4 . 10² litros.

b) 1,4 . 10³ litros.

c) 5,2 . 10³ litros.

d) 1,5 . 104 litros.

e) 5,2 . 104 litros.

Resolução: ∆V = V0.γ. ∆ = 14000. 10–3

. (30 –

20) = 140 L. (Resposta A)

53) Um recipiente tem, a 0 oC, capacidade

volumétrica de 20cm³ e a 100 °C sua capacidade

é de 20,01cm³. Quando ele é completamente

preenchido com certo líquido a 0°C,

transbordam 0,05cm³ ao ser feito o referido

aquecimento. Determine: o coeficiente de

dilatação térmica volumétrica real do líquido.

Resolução: O coeficiente de dilatação do frasco

é ∆V = V0.γ. ∆ (20,01 – 20) = 20. γ.(100 –

0) γF = 0,5. 10-5

ºC-1

. O coeficiente aparente

é ∆V = V0.γ. ∆ 0,05 = 20.γ.(100 – 0)

γAP = 0,25.10-5

ºC-1

O coeficiente real do líquido será γR = γF + γAP =

0,5. 10-5

+ 0,25.10-5

= 0,75.10-5

ºC-1

(Resposta:

0,75.10-5

ºC-1

)

Calorimetria

54) (UFRRJ – RJ) Uma pessoa bebe 200 gramas

de água a 20°C. Sabendo-se que a temperatura

do seu corpo é praticamente constante e vale

36,5 °C, a quantidade de calor absorvido pela

água é igual a: (calor específico da água =

1cal/g°C)

a) 730cal.

b) 15600cal.

c) 3300cal.

d) 1750cal

e) 0,01750cal.

Resolução: Q = m.c.∆ = 200.1.(36,5 – 20) =

3300 cal. (Resposta C)

55) (UNILASALLE – SP) Para tomar

chimarrão, aquecem-se 500 g de água a 20 ºC

até 80 ºC. A quantidade de energia fornecida à

água, desprezando-se perdas, é a seguinte (dado:

calor específico da água, c = 1 cal/gºC e 1caloria

vale aproximadamente 4,0 J)

a) 120 kJ.

b) 120 MJ.

13

c) 120 J.

d) 100 kJ.

e) 100 J.

Resolução: Q = m.c.∆ = 500.1.(80 – 20) =

30000 cal x 4 = 120000L = 120 kJ. (Resposta

A)

56) (UERJ – RJ) Considere X e Y dois corpos

homogêneos, constituídos por substâncias

distintas, cujas massas correspondem,

respectivamente, a 20 g e 10 g.

O gráfico abaixo mostra as variações da

temperatura desses corpos em função do calor

absorvido por eles durante um processo de

aquecimento.

Determine as capacidades térmicas de X e Y e,

também, os calores específicos das substâncias

que os constituem.

Resolução: CX = Q/∆ = 80/(281 – 273) = 10

cal/K ou 10cal/ºC e CY = Q/∆ = 40/(283 – 273)

= 4 cal/k ou 4 cal/ºC.

cX = CX/m = 10/20 = 0,5 cal/gºC e cY = CY/m =

4/10 = 0,4 cal/gºC.

(Resposta: capacidades térmicas CX =

10cal/ºC e CY = 4 cal/ºC; calores específicos

cX = 0,5 cal/gºC e cY = 0,4 cal/gºC)

57) (PUC – RS) Responder à questão seguinte

com base no gráfico a seguir, referente à

temperatura em função do tempo, de um corpo

que está sendo aquecido e que absorve 20cal/s.

A capacidade térmica do corpo é

a) 20 cal/ºC

b) 30 cal/ºC

c) 40 cal/ºC

d) 50 cal/ºC

e) 60 cal/ºC

Resolução: C = Q/∆ = (20.100)/(60 – 20) = 50

cal/ºC. (Resposta D)

58) (UNESP – SP) O gráfico representa a

temperatura em função do tempo de um líquido

aquecido em um calorímetro.

Considerando-se desprezível a capacidade

térmica do calorímetro e que o aquecimento foi

obtido através de uma resistência elétrica,

dissipando energia à taxa constante de 120 W, a

capacidade térmica do líquido vale:

a) 12 J/oC.

b) 20 J/oC.

c) 120 J/oC.

d) 600 J/oC.

e) 1 200 J/oC.

Resolução: 120 W = 120 J/s e C = Q/∆ =

(120.300)/(54 – 24)= 10/30 = 1200 J/ o

C.

(Resposta E)

59) (UFRN – RN) Diariamente, Dona

Leopoldina coloca uma lata de refrigerante, cuja

temperatura é de 30 ºC, numa caixa térmica

contendo gelo e, após esperar algumas horas,

bebe o refrigerante a uma temperatura de

aproximadamente 5 ºC.

Nesse caso, é correto afirmar que a diminuição

da temperatura do refrigerante se explica

porque, no interior da caixa térmica, a lata de

refrigerante

a) cede calor para o gelo, e este cede calor para

ela, porém numa quantidade menor que a

recebida.

b) recebe frio do gelo, para o qual cede calor,

porém numa quantidade menor que o frio

recebido.

c) cede calor para o gelo, e este cede calor para

ela, porém numa quantidade maior que a

recebida.

d) recebe frio do gelo, para o qual cede calor,

porém numa quantidade maior que o frio

recebido.

Resolução: O fluxo do calor se dá nos dois

sentidos, sendo maior do refrigerante (maior

temperatura) para o gelo (menor temperatura).

(Resposta A)

60) (UFMS – MS) Quando a temperatura de

100g de água (calor específico = 4,2 J/(g.K)

diminui de 25ºC para 5ºC, há liberação de calor.

14

É correto afirmar que a quantidade de calor

liberada pela água é;

a) 8 400 Joule.

b) – 8 400 Joule.

c) 2 000/4,2 Joule.

d) – 2 000/4,2 Joule.

e) – 84/100 Joule.

Resolução: Q = m.c.∆ = 100. 4,2 (5 – 20) = –

8400 J. (Resposta B)

61) (UERJ – RJ) Um adulto, ao respirar durante

um minuto, inspira, em média, 8,0 litros de ar a

20 ºC, expelindo-os a 37 ºC.

Admita que o calor específico e a densidade do

ar sejam, respectivamente, iguais a 0,24 cal.g-1

.

ºC-1

e 1,2 g/L-1

.

Nessas condições, a energia mínima, em

quilocalorias, gasta pelo organismo apenas no

aquecimento do ar, durante 24 horas, é

aproximadamente igual a:

a) 15,4

b) 35,6

c) 56,4

d) 75,5

Resolução: A massa de ar em 1 minuto será d =

m/v 1,2 = m/8 m = 9,8 g. Em 24 horas

teremos 24.60.9,6 = 13824 g. Assim Q = m.c.∆

= 13824. 0,24. (37 – 20) = 56401 cal ou 56,4

kcal. (Resposta C)

62) (CEFET – PR) Um pequeno aquecedor

elétrico de imersão é usado para aquecer 100 g

de água para uma xícara de café instantâneo. O

aquecedor está rotulado com “200 W”, o que

significa que ele converte energia elétrica em

energia térmica com essa taxa. Calcule o tempo

necessário para levar toda essa água de 20ºC

para 100ºC, ignorando quaisquer perdas.

(Considerar cágua = 1cal/gºC e 1 cal = 4,19 J)

a) 40 s.

b) 20 s.

c) 400 s.

d) 167,6 s.

e) 1676 s.

Resolução: Q = m.c.∆ = 100.4,19.(100 – 20) =

33520 J. Assim Pot = Q/∆t 200 = 35520/∆t

∆t = 167,6 s. (Resposta D)

63) (PUCCAMP – SP) Admita que o corpo

humano transfira calor para o meio ambiente na

razão de 2,0kcal/min. Se esse calor pudesse ser

aproveitado para aquecer água de 20°C até

100°C, a quantidade de calor transferida em 1

hora aqueceria uma quantidade de água, em kg,

igual a: (calor específico da água = 1,0kcal/kg

°C)

Resolução: Em 1 hora teremos Q = 2.60 = 120

kcal. Assim: Q = m.c.∆ 120 = m.1.20 m

= 6 kg. (Resposta: 6 kg)

64) (MACK – SP) Na festa de seu aniversário, o

aniversariante comeu salgadinhos e bebeu

refrigerantes, ingerindo o total de 2 000 kcal.

Preocupado com o excesso alimentar, ele

pensou em perder as “calorias” adquiridas,

ingerindo água gelada a 12 ºC, pois, estando o

interior de seu organismo a 37 ºC, as “calorias”

adquiridas seriam consumidas para aquecer a

água.

Admitindo que seu raciocínio esteja correto, o

volume de água a 12 ºC que deve beber é:

a) 60 litros.

b) 65 litros.

c) 70 litros.

d) 80 litros.

e) 90 litros.

Resolução: Q = m.c.∆ 2000000= m.1. (37

– 12) m = 80000g o que corresponde a 80 L.

(Resposta D)

65) (UNESP – SP) Massas iguais de cinco

líquidos distintos, cujos calores específicos

estão dados na tabela adiante, encontram-se

armazenadas, separadamente e à mesma

temperatura, dentro de cinco recipientes com

boa isolação e capacidade térmica desprezível.

Se cada líquido receber a mesma quantidade de

calor, suficiente apenas para aquecê-lo, mas sem

alcançar seu ponto de ebulição, aquele que

apresentará temperatura mais alta, após o

aquecimento, será:

a) a água.

b) o petróleo.

c) a glicerina.

d) o leite.

e) o mercúrio.

Resolução: O de menor calor específico sofrerá

maior aquecimento já que cada grama

necessitará de menos calor para variar sua

temperatura em 1 grau. Portanto, em ordem

decrescente de temperaturas teremos mercúrio;

petróleo; glicerina; leite; água. (Resposta E)

66) (ENEM) A energia geotérmica tem sua

origem no núcleo derretido da Terra, onde as

temperaturas atingem 4.000 ºC. Essa energia é

primeiramente produzida pela decomposição de

materiais radiativos dentro do planeta.

Em fontes geotérmicas, a água, aprisionada em

um reservatório subterrâneo, é aquecida pelas

15

rochas ao redor e fica submetida a altas

pressões, podendo atingir temperaturas de até

370 ºC sem entrar em ebulição. Ao ser liberada

na superfície, à pressão ambiente, ela se

vaporiza e se resfria, formando fontes ou

gêiseres. O vapor de poços geotérmicos é

separado da água e é utilizado no

funcionamento de turbinas para gerar

eletricidade. A água quente pode ser utilizada

para aquecimento direto ou em usinas de

dessalinização. Roger A. Hinrichs e Merlin Kleinbach. Energia e meio

ambiente. Ed. ABDR (com adaptações).

Depreende-se das informações acima que as

usinas geotérmicas

a) utilizam a mesma fonte primária de energia

que as usinas nucleares, sendo, portanto,

semelhantes os riscos decorrentes de ambas.

b) funcionam com base na conversão de energia

potencial gravitacional em energia térmica.

c) podem aproveitar a energia química

transformada em térmica no processo de

dessalinização.

d) assemelham-se às usinas nucleares no que diz

respeito à conversão de energia térmica em

cinética e, depois, em elétrica.

e) transformam inicialmente a energia solar em

energia cinética e, depois, em energia térmica.

Resolução: A energia térmica resultante das

reações nucleares é transformada em cinética e

em seguida em elétrica, como nas usinas

nucleares. (Resposta D)

67) (FUVEST – SP) Um fogão, alimentado por

um botijão de gás, com as características

descritas no quadro abaixo, tem em uma de suas

bocas um recipiente com um litro de água que

leva 10 minutos para passar de 20 ºC a 100 ºC.

Para estimar o tempo de duração de um botijão,

um fator relevante é a massa de gás consumida

por hora. Mantida a taxa de geração de calor das

condições acima, e desconsideradas as perdas de

calor, a massa de gás consumida por hora, em

uma boca de gás desse fogão, é

aproximadamente

a) 8 g

b) 12 g

c) 48 g

d) 320 g

e) 1920 g

Resolução: 1L contém 1kg de água. Assim, em

10 minutos temos Q = m.c.∆ = 1.1.(100 – 20)

= 80 kJ e como o calor de combustão é 40000

kJ/kg, temos 40000 = 80/m m = 0,002 kg ou

2 g e em 1 hora fica 2 x 60 min/10 min = 12 g.

(Resposta B)

68) (UERJ – RJ) O gráfico abaixo representa o

consumo de oxigênio de uma pessoa que se

exercita, em condições aeróbicas, numa

bicicleta ergométrica. Considere que o

organismo libera, em média, 4,8 kcal para cada

litro de oxigênio absorvido.

A energia liberada no período entre 5 e 15

minutos, em kcal, é:

a) 48,0

b) 52,4

c) 67,2

d) 93,6

Resolução: Q = 1,4 (15 – 5) = 14 L x 4,8 = 67,2

kcal. (Resposta C)

69) (UERJ – RJ) O excesso de gordura no

organismo é nocivo à saúde. Considere uma

pessoa, com massa corporal estável, que deseje

perder gordura, sem alterar sua dieta alimentar.

Para essa pessoa, um dispêndio energético de 9

kcal em atividades físicas corresponde à perda

de 1 g de gordura corporal.

Para perder 6,0 kg de gordura, o tempo, em

minutos, que ela necessita dedicar a atividades

físicas, despendendo, em média, 12 kcal/min,

corresponde a:

a) 2,0. 10²

b) 4,5.10³

c4

d) 6,0.105

Resolução: A perda de 6000 g de gordura

requer um dispêndio energético de 6000.9 =

54000 kcal. Assim, ∆t = 54000/12 = 4500

minutos (Resposta B)

70) (MACK – SP – Modificado) A preocupação

com a qualidade e com o tempo de vida leva o

homem moderno urbano a manter uma dieta

alimentar adequada, acompanhada de exercícios

físicos corretos.

- Considere que, para Pedro, o valor energético

adequado, consumido por almoço, seja de 700

kcal de alimentos.

- Considere, ainda, que Pedro, no almoço, ingira

100 g de arroz, 100 g de feijão, 150 g de bife e

16

50 g de batata frita, além de uma lata de

refrigerante.

Consultando a tabela, pode-se afirmar que, para

consumir o excesso energético ingerido, Pedro

deve correr, aproximadamente:

Alimento Energia kcal/g

Arroz 3,6

Feijão 3,4

Bife 3,8

Batata frita 1,4

Considere:

1) Lata de refrigerante 48 kcal

2) Energia consumida em corrida 1080

kcal/hora

a) 80 minutos.

b) 40 minutos.

c) 34 minutos.

d) 60 minutos.

e) 92 minutos.

Resolução: A energia será 100.3,6 + 100.3,4 +

150.3,8 + 50.1,4 + 48 = 360 + 340 + 570 + 48 =

1318 kcal. O excesso é 1318 – 700 = 618 kcal.

Assim, ∆t = 618/1080x 60 min 34 min =

(Resposta C)

71) (UFF – RJ) Para se resfriar um motor em

funcionamento, é necessário acionar seu sistema

de refrigeração, podendo-se usar as substâncias

ar ou água.

A massa de ar m1 e a massa de água m2 sofrem

a mesma variação de temperatura e

proporcionam a mesma refrigeração ao motor.

Neste caso, a razão m1/m2 é:

Dados: calor específico da água = 1,0 cal/g oC

calor específico do ar = 0,25 cal/g oC

a) 4,0

b) 0,67

c) 2,0

d) 0,25

e) 1,0

Resolução: Q1 = Q2 m1.c1.∆ = m2.c2.∆

m1. 1 = m2.0,25 m1. / m2 = 0,25. (Resposta

D)

72) (UFPI – PI) Para perfurar um buraco num

bloco de cobre, de 1,00 kg de massa, um

operário utilizou uma furadeira de potência de

200W durante 200s . Supondo-se que somente

70% da energia envolvida no processo de

perfuração tenha sido absorvida pelo bloco na

forma de calor, o aumento médio da

temperatura do bloco foi de, aproximadamente:

Dados: o calor específico do cobre é

0,093cal/goC e 1cal = 4,2 J.

a) 30,7 oC.

b) 71,7 oC.

c) 1,02×10² oC.

d) 9,30×10² oC.

e) 9,52×10³ oC.

Resolução: Pot = Q/∆t 70%.200 = Q/200

Q = 28000 J. Sendo Q = m.c.∆ fica 28000

= 1000. 0,093.4,2.∆ ∆ 71,7 ºC.

(Resposta B)

73) (UERJ – RJ) Duas barras metálicas A e B,

de massas mA=100 g e mB=120 g, inicialmente à

temperatura de 0 ºC, são colocadas, durante 20

minutos, em dois fornos. Considere que toda a

energia liberada pelas fontes térmicas seja

absorvida pelas barras.

O gráfico a seguir indica a relação entre as

potências térmicas fornecidas a cada barra e o

tempo de aquecimento.

Após esse período, as barras são retiradas dos

fornos e imediatamente introduzidas em um

calorímetro ideal.

O diagrama abaixo indica a variação da

capacidade térmica de cada barra em função de

sua massa.

A temperatura que corresponde ao equilíbrio

térmico entre as barras A e B é, em ºC,

aproximadamente igual a:

a) 70

b) 66

c) 60

d) 54

Resolução: As quantidades de calor fornecidas

a cada barra em 20 min serão dadas através das

áreas do 1º diagrama. Assim: QA = 600.20/2 =

6000 cal e QB = 240.20/2 = 2400 cal. As

temperaturas iniciais das barras serão, portanto:

CA = QA/∆A 80 = 6000/∆A ∆A = 75

ºC e CB = QB/∆B 48 = 2400/∆B ∆B =

50 ºC. O equilíbrio térmico se dará a:

mA.cA.∆A + mB.cB.∆ B = 0 80( – 75) +

17

48( – 50) = 65,625 ºC.

(Resposta B)

74) (UFF – RJ) Duelo de Gigantes:

O rio Amazonas é o maior rio do mundo em

volume d’água com uma vazão em sua foz de,

aproximadamente, 175 milhões de litros por

segundo.

A usina hidroelétrica de Itaipu também é a

maior do mundo, em operação. A potência

instalada da usina é de 12,6.109 W. Suponha que

toda essa potência fosse utilizada para aquecer a

água que flui pela foz do rio Amazonas, sem

que houvesse perdas de energia.

Veja, 24/09/ 2003. (Adaptado)

Nesse caso, a variação de temperatura dessa

água, em grau Celsius, seria da ordem de:

Dados:

calor específico da água c = 1,0 cal/g oC,

densidade da água = 1,0 g/cm³ e 1 cal = 4,2

Joules

a) 10–2

b) 10–1

c) 100

d) 101

e) 102

Resolução: Pot = Q/∆t = m.c.∆/∆t

12,6.109 = 175. 10

9.1.∆/1 ∆ = 0,072 =

7,2 . 10–2

ºC. ordem de grandeza 10–1

já que 7,2

> 10 (Resposta B)

75) (UFG – GO) O cérebro de um homem

típico, saudável e em repouso, consome uma

potência de aproximadamente 16 W. Supondo

que a energia gasta pelo cérebro em 1 min fosse

completamente usada para aquecer 10 ml de

água, a variação de temperatura seria de,

aproximadamente,

Densidade da água: 1,0·10³ kg/m³

Calor específico da água: 4,2·10³ J/kg·oC

a) 0,5 oC

b) 2 oC

c) 11 oC

d) 23 oC

e) 48 oC

Resolução: Pot = Q/∆t 16 = Q/60

Q = 960 J e, sendo Q = m.c.∆, fica 960 =

10.4,2. ∆ ∆ = 22,85 ºC. (Resposta D)

76) (FGV – SP) Os trajes de neopreme, um

tecido emborrachado e isolante térmico, são

utilizados por mergulhadores para que certa

quantidade de água seja mantida próxima ao

corpo, aprisionada nos espaços vazios no

momento em que o mergulhador entra na água.

Essa porção de água em contato com o corpo é

por ele aquecida, mantendo assim uma

temperatura constante e agradável ao

mergulhador. Suponha que, ao entrar na água,

um traje retenha 2,5 L de água inicialmente a

21°C. A energia envolvida no processo de

aquecimento dessa água até 35°C é:

Dados: densidade da água = 1 kg/L e calor

específico da água = 1 cal/(g°C)

a) 25,5 kcal.

b) 35,0 kcal.

c) 40,0 kcal.

d) 50,5 kcal.

e) 70,0 kcal.

Resolução: Q = m.c.∆ = 2500. 1. (35 – 21) =

35000cal ou 35 kcal. (Resposta B)

77) (UNIFESP – SP) Dois corpos, A e B, com

massas iguais e a temperaturas tA = 50°C e tB =

10°C, são colocados em contato até atingirem a

temperatura de equilíbrio. O calor específico de

A é o triplo do de B. Se os dois corpos estão

isolados termicamente, a temperatura de

equilíbrio é:

a) 28°C

b) 30°C

c) 37°C

d) 40°C

e) 45°C

Resolução: QA + QB = 0 mA.cA.∆A +

mB.cB.∆B = 0 m. 3 cB.( – 50) + m.cB.(

– 10) = 0 3 – 150 + – 10 = 0 = 40

ºC. (Resposta D)

78) (UFRJ – RJ) Três amostras de um mesmo

líquido são introduzidas num calorímetro

adiabático de capacidade térmica desprezível:

uma de12 g a 25 °C, outra de 18 g a 15 °C e a

terceira de 30 g a 5 °C.

Calcule a temperatura do líquido quando se

estabelecer o equilíbrio térmico no interior do

calorímetro.

Resolução: QA + QB + QC = 0 mA.cA.∆A +

mB.cB.∆B + mC.cC.∆C = 0 12.c.( – 25) +

18.c.( – 15) + 30.c.( – 5) = 0 = 12 ºC.

(Resposta: 12 ºC)

79) (FUVEST – SP) Dois recipientes iguais A e

B, contendo dois líquidos diferentes,

inicialmente a 20°C, são colocados sobre uma

placa térmica, da qual recebem

aproximadamente a mesma quantidade de calor.

Com isso, o líquido em A atinge 40°C, enquanto

o líquido em B, 80°C. Se os recipientes forem

retirados da placa e seus líquidos misturados, a

temperatura final da mistura ficará em torno de:

18

a) 45°C

b) 50°C

c) 55°C

d) 60°C

e) 65°C

Resolução: A relação entre as capacidades

térmicas dos líquidos é: CA/CB = (Q/∆A)/

(Q/∆B) CA/CB = ∆B/∆A CA/CB = (80

– 20)/(40 – 20) = 3 ou seja CA = 3CB

Na mistura fica QA + QB = 0 mA.cA.∆A +

mB.cB.∆B = 0 CA. ( – 40) + CB. ( – 80)

= 0 3CB. ( – 40) + CB. ( – 80) = 0

3 – 120 + – 80 = 0 = 50 ºC. (Resposta

B)

80) (UFMG – MG) Numa aula de Física, o

Professor Carlos Heitor apresenta a seus alunos

esta experiência: dois blocos, um de alumínio e

outro de ferro, de mesma massa e, inicialmente,

à temperatura ambiente, recebem a mesma

quantidade de calor, em determinado processo

de aquecimento.

O calor específico do alumínio e o do ferro são,

respectivamente, 0,90 J/goC e 0,46 J/g

oC.

Questionados quanto ao que ocorreria em

seguida, dois dos alunos, Alexandre e Lorena,

fazem, cada um deles, um comentário:

• Alexandre: “Ao final desse processo de

aquecimento, os blocos estarão à mesma

temperatura.”

• Lorena: “Após esse processo de aquecimento,

ao se colocarem os dois blocos em contato,

fluirá calor do bloco de ferro para o bloco de

alumínio.”

Considerando-se essas informações, é correto

afirmar que:

a) apenas o comentário de Alexandre está certo.

b) apenas o comentário de Lorena está certo.

c) ambos os comentários estão certos.

d) nenhum dos dois comentários está certo.

Resolução: Ao receberem iguais quantidades de

calor, o bloco de ferro fica mais aquecido pois

seu calor específico é menor, sendo que cada

grama desse material necessita de menos calor

para cada grau de variação em sua temperatura.

Assim Alexandre está errado pois o ferro está

mais aquecido. Lorena está certa pois o fluxo de

calor é maior do ferro para o alumínio.

(Resposta B)

81) (INATEL – MG) Calor de combustão é a

quantidade de calor liberada na queima de uma

unidade de massa do combustível. O calor de

combustão do gás de cozinha é 6,0.106cal/kg.

Calcule o volume de água, em litros, que pode

ser aquecido de 20°C a 100°C com um botijão

de gás de cozinha de 13 kg, admitindo que esse

processo tenha uma eficiência de 40%. Dados:

calor específico da água c = 1cal/g°C, densidade

da água d = 1,0.10³g/L)

Resolução: Em 1 botijão temos CC = Q/m

6,0.106 = Q/13 Q = 78. 10

6 cal. Sendo a

eficiência 40% então o calor aproveitado será Q

= 40%.78. 106

= 31,2. 106 cal. A massa de água

será Q = m.c.∆ 31,2. 106 = m.1. (100 – 0)

m = 31,2. 104g = 312.10³g = 312 kg e sendo

a densidade igual a 1 teremos V = 312 L.

(Resposta: 312 litros)

82) (FUVEST – SP) O processo de

pasteurização do leite consiste em aquecê-lo a

altas temperaturas, por alguns segundos, e

resfriá-lo em seguida. Para isso, o leite percorre

um sistema, em fluxo constante, passando por

três etapas:

I) O leite entra no sistema (através de A), a 5 ºC,

sendo aquecido (no trocador de calor B) pelo

leite que já foi pasteurizado e está saindo do

sistema.

II) Em seguida, completa-se o aquecimento do

leite, através da resistência R, até que ele atinja

80 ºC. Com essa temperatura, o leite retorna a

B.

III) Novamente em B, o leite quente é resfriado

pelo leite frio que entra por A, saindo do

sistema (através de C), a 20ºC.

Em condições de funcionamento estáveis, e

supondo que o sistema seja bem isolado

termicamente, pode-se afirmar que a

temperatura indicada pelo termômetro T, que

monitora a temperatura do leite na saída de B, é

aproximadamente de

a) 20 ºC

b) 25 ºC

c) 60 ºC

d) 65 ºC

e) 75 ºC

Resolução: QCED = m.c.(20 – 80) = -60mc

QREC = m.c. ( – 5) e, sendo QCED +QREC = 0,

fica -60mc + m.c. ( – 5) = 0 – 5 = 60 e

= 65 ºC. (Resposta D)

83) (UFPR – PR) Um recipiente termicamente

isolado contém 500 g de água na qual se

mergulha uma barra metálica homogênea de

250 g. A temperatura inicial da água é 25,0 °C e

19

a da barra 80,0 °C. Considere o calor específico

da água igual a 1,00 cal/g.°C, o do metal igual a

0,200 cal/g.°C e despreze a capacidade térmica

do recipiente. Com base nesses dados, é correto

afirmar que:

(01) A temperatura final de equilíbrio térmico é

de 52,5 °C.

(02) O comprimento da barra permanece

constante durante o processo de troca de calor.

(04) A temperatura inicial da barra, na escala

kelvin, é de 353 K.

(08) A quantidade de calor recebida pela água é

igual à cedida pela barra.

(16) A energia interna final da água, no

equilíbrio térmico, é menor que sua energia

interna inicial.

Soma = ( )

Resolução:

01(F) QA + QB = 0 mA.cA.∆A + mB.cB.∆B

= 0 500.1.( - 25) + 250.0,2. ( - 80) = 0

= 30 ºC.

02(F) Já que a barra diminui sua temperatura

então sofrerá uma contração térmica.

04(V) T(K) = C + 273 = 80 + 273 = 353 K

08(V) Já que não ocorrem trocas de calor com o

meio externo e tampouco com o recipiente.

16(F) Já que a água experimentou uma elevação

em sua temperatura. (Resposta: 12 (04 + 08))

84) (ITA – SP) Colaborando com a campanha

de economia de energia, um grupo de escoteiros

construiu um fogão solar, consistindo de um

espelho de alumínio curvado que foca a energia

térmica incidente sobre uma placa coletora.

O espelho tem um diâmetro efetivo de 1,00m e

70% da radiação solar incidente é aproveitada

para de fato aquecer certa quantidade de água.

Sabemos ainda que o fogão solar demora 18,4

minutos para aquecer 1,00 L de água desde a

temperatura de 20 °C até 100 °C, e que

4,186.10³ J é a energia necessária para elevar a

temperatura de 1,00 L de água de 1,000 K.

Com base nos dados, estime a intensidade

irradiada pelo Sol na superfície da Terra, em

W/m². .Justifique.

Resolução:

A potência solar útil no espelho será: Pot = Q/∆t

Pot = m.c.∆/∆t =

1kg.418,6J/(kg.K).80K/18,4.60s = 30,33 J/s =

30,33 W. A potência total será 30,33/70% =

43,33 W. assim a intensidade da radiação é: I =

Pot/A = 43,33/,r² = 43,33/.0,5² 55,17 W/m².

(Resposta: 55,17 W/m²)

85) (UFPE – PE) Um certo volume de um

líquido A, de massa M e que está inicialmente a

20ºC, é despejado no interior de uma garrafa

térmica que contém uma massa 2M de um outro

líquido, B, na temperatura de 80ºC. Se a

temperatura final da mistura líquida resultante

for de 40ºC, podemos afirmar que a razão cA/cB

entre os calores específicos das substâncias A e

B vale:

a) 6

b) 4

c) 3

d) ½

e) 1/3

Resolução: QA + QB = 0 mA.cA.∆A +

mB.cB.∆B = 0 M. cA.(40 – 20) + 2M.

cB.(40 – 80) = 0 20cA – 80.cB = 0 cA/cB

= 4. (Resposta B)

86) (ENEM) Nos últimos anos, o gás natural

(GNV: gás natural veicular) vem sendo

utilizado pela frota de veículos nacional, por ser

viável economicamente e menos agressivo do

ponto de vista ambiental.

O quadro compara algumas características do

gás natural e da gasolina em condições

ambiente.

d (g/cm³)

GNV 0,8

Gasolina 738

Apesar das vantagens no uso de GNV, sua

utilização implica algumas adaptações técnicas,

pois, em condições ambientes, o volume de

combustível necessário, em relação ao de

gasolina, para produzir a mesma energia, seria:

a) muito maior, o que requer um motor muito

mais potente.

b) muito maior, o que requer que ele seja

armazenado a alta pressão.

c) igual, mas sua potência será muito menor.

d) muito menor, o que o torna o veículo menos

eficiente.

e) muito menor, o que facilita sua dispersão para

a atmosfera.

Resolução: A baixa densidade do GNV requer

que, para se obter uma massa equivalente a da

gasolina, o volume deverá ser bem maior

requerendo armazenamento sb alta pressão para

reduz[i-lo a ponto de ser armazenado.

(Resposta B)

Mudanças de estados físicos da

matéria

87) (UNESP – SP) Nos quadrinhos da tira, a

mãe menciona as fases da água conforme a

mudança das estações.

20

Entendendo “boneco de neve” como sendo

“boneco de gelo” e que com o termo

“evaporou” a mãe se refira à transição

água/vapor, pode-se supor que ela imaginou a

seqüência gelo/água/vapor/água. As mudanças

de estado que ocorrem nessa seqüência são:

a) fusão, sublimação e condensação.

b) fusão, vaporização e condensação.

c) sublimação, vaporização e condensação.

d) condensação, vaporização e fusão.

Resolução: gelo/água fusão; água/vapor

vaporização; vapor/água condensação.

(Resposta B)

88) (UFRN – RN - Modificado) A existência da

água em seus três estados físicos, sólido, líquido

e gasoso, torna nosso Planeta um local peculiar

em relação aos outros Planetas do Sistema

Solar. Sem tal peculiaridade, a vida em nosso

Planeta seria possivelmente inviável. Portanto,

conhecer as propriedades físicas da água ajuda a

melhor utilizá-la e assim contribuir para a

preservação do Planeta.

Na superfície da Terra, em altitudes próximas

ao nível do mar, os estados físicos da água estão

diretamente relacionados à sua temperatura

conforme mostrado no gráfico abaixo. Esse

gráfico representa o comportamento de uma

massa de 1,0 g de gelo a uma temperatura

inicial de – 50 oC, colocada em um calorímetro

que, ligado a um computador, permite

determinar a temperatura da água em função da

quantidade de calor que lhe é cedida.

Observando-se o gráfico, pode-se concluir que a

quantidade de calor necessária para elevar a

temperatura do gelo a 0 oC até água a 100

oC é:

a) 180 cal.

b) 200 cal.

c) 240 cal.

d) 100 cal

Resolução: Q = 205 – 25 = 180 cal. (Resposta

A)

89) (IFPE – PE) Uma amostra de determinada

substância com massa 30g encontra-se

inicialmente no estado liquido, a 60°C. Está

representada pelo gráfico ao lado a temperatura

dessa substância em função da quantidade de

calor por ela cedida.

Analisando esse gráfico, é correto afirmar que:

a) a temperatura de solidificação da substância é

10°C.

b) o calor específico latente de solidificação é -

1,0 cal/g.

c) o calor específico sensível no estado líquido é

1/3 cal/g°C.

d) o calor específico sensível no estado sólido é

1/45 cal/g°C.

e) ao passar do estado líquido a 60°C para o

sólido a 10°C a substância perdeu 180 cal.

Resolução:

a(F) o gráfico mostra que a temperatura se torna

constante a 30 ºC. É a temperatura de mudança

de estado (solidificação).

b(V) L = Q/m = -(60 – 30)/30 = -1 cal/g

c(F) c = Q/m.∆ = 30/30.30 = 1/30 cal/g°C

d(F) c = Q/m.∆ = 30/30.20 = 1/20 cal/g°C

e(F) o gráfico mostra 90 calorias.

(Resposta B)

90) (UFG – GO) No gráfico, está representada a

variação da temperatura em função do tempo de

uma massa de 200 g d’água. Suponha que a

fonte forneceu energia térmica a uma potência

constante desde o instante t = 0, e que toda essa

energia foi utilizada para aquecer a água. Dado:

c = 1,0 cal/g°C.

Analisando-se o gráfico, pode-se afirmar que

1-( ) no primeiro minuto, não ocorreu absorção

de calor.

21

2-( ) a potência da fonte térmica é de 16

kcal/min.

3-( ) a temperatura da água para t = 2 min é 80

°C.

4-( ) de 0 a 9 minutos, ocorreram três transições

de fase.

Resolução:

1(F) A fonte tinha potência constante. A água

encontrava-se a 0 oC e no estado sólido.

2(V) Pot = m.c.∆/ ∆t = 200.1.100/1,25 = 16000

cal/min ou 16 kcal/min

3(V) Pot = m.c.∆/∆t 16000 = 200.1./1

= 80 oC.

4(F) ocorreram 2. Fusão e vaporização.

(Resposta: 1-F; 2-V; 3-V; 4-F)

91) (PUC – RS) Muitas pessoas, ao cozinharem,

se preocupam com a economia de gás e adotam

algumas medidas práticas, como:

I. deixar o fogo baixo do início ao fim, pois

assim se obtém cozimento mais rápido;

II. baixar o fogo quando a água começa a ferver,

pois a temperatura permanece constante durante

a ebulição;

III. deixar o fogo alto do início ao fim, obtendo

uma constante elevação de temperatura, mesmo

após o início da ebulição.

Pela análise das afirmativas, conclui-se que

somente

a) está correta a I.

b) está correta a II.

c) está correta a III.

d) estão corretas a I e a III.

e) estão corretas a II e a III.

Resolução:

I-F a água demorará mais tempo para chegar a

sua temperatura de ebulição e ali estabilizar.

II-V a temperatura permanecerá constante dali

por diante, independentemente do fluxo do

calor, apenas mudando de estado físico.

III-F durante a mudança de estado a temperatura

permanece constante. (Resposta B)

92) (UFRN – RN) Cotidianamente são usados

recipientes de barro (potes, quartinhas, filtros

etc.) para esfriar um pouco a água neles contida.

Considere um sistema constituído por uma

quartinha cheia d´água. Parte da água que chega

à superfície externa da quartinha, através de

seus poros, evapora, retirando calor do barro e

da água que o permeia. Isso implica que

também a temperatura da água que está em seu

interior diminui nesse processo.

Tal processo se explica porque, na água que

evapora, são as moléculas de água

a) com menor energia cinética média que

escapam do líquido, aumentando, assim, a

energia cinética média desse sistema.

b) que, ao escaparem do líquido, aumentam a

pressão atmosférica, diminuindo, assim, a

pressão no interior da quartinha.

c) com maior energia cinética média que

escapam do líquido, diminuindo, assim, a

energia cinética média desse sistema.

d) que, ao escaparem do líquido, diminuem a

pressão atmosférica, aumentando, assim, a

pressão no interior da quartinha.

Resolução: A água, ao evaporar, absorve

energia do sistema, resfriando-o. (Resposta C)

93) (ITA – SP) Um pedaço de gelo flutua em

equilíbrio térmico com uma certa quantidade de

água depositada em um balde. À medida que o

gelo derrete, podemos afirmar que

a) o nível da água no balde aumenta, pois

haverá uma queda de temperatura da água.

b) o nível da água no balde diminui, pois haverá

uma queda de temperatura da água.

c) o nível da água no balde aumenta, pois a

densidade da água é maior que a densidade do

gelo.

d) o nível da água no balde diminui, pois a

densidade da água é maior que a densidade do

gelo.

e) o nível da água no balde não se altera.

Resolução: Ao fundir, a parte submersa do gelo

sofre uma contração térmica de mesmo volume

que a parte emersa do gelo. Assim o nível da

água não se altera com o derretimento do gelo.

(Resposta E)

94) (CEFET – PR) Uma estudante colocou em

um recipiente 2,0 litros de água cuja densidade é

1 g/cm³, inicialmente a 20 ºC, para ferver.

Distraindo-se, esqueceu a água no fogo por um

certo tempo e, quando percebeu, metade da água

havia evaporado. Curiosa, desejou saber que

quantidade de calor a água havia consumido no

processo.

Sendo o calor específico e o calor de

vaporização da água, respectivamente, iguais a

1,0 cal/g. ºC e 540 cal/g, encontrou:

a) 700 kcal.

b) 620 kcal.

c) 160 kcal.

d) 540 kcal.

e) 80 kcal.

Resolução: Q = QS + QL = m.c.∆ + m1.LF =

2000.1.(100 – 20) + 1000.540 = 700000cal ou

700 kcal. (Resposta A)

95) (FUVEST – SP) Um recipiente de isopor,

que é um bom isolante térmico, tem em seu

interior água e gelo em equilíbrio térmico. Num

dia quente, a passagem de calor por suas

paredes pode ser estimada, medindo-se a massa

de gelo Q presente no interior do isopor, ao

22

longo de algumas horas, como representado no

gráfico.

Esses dados permitem estimar a transferência de

calor pelo isopor, como sendo,

aproximadamente, de: (Calor latente de fusão do

gelo ≈ 320 kJ/kg)

a) 0,5 kJ/h

b) 5 kJ/h

c) 120 kJ/h

d) 160 kJ/h

e) 320 kJ/h

Resolução: Fluxo = Q/∆t = m.L/∆t = (20 – 8).

320/24 = 160 kJ. (Resposta D)

96) (ACAFE – SC) Patrícia deseja “gelar” um

refrigerante que se encontra à temperatura

ambiente. Para isso, dispõe de dois recipientes:

um com certa massa de água e outro com igual

massa de gelo, ambas a uma temperatura de

0ºC.

Para conseguir o seu intento, da melhor forma

possível, é aconselhável que Patrícia mergulhe o

refrigerante:

a) no gelo, porque inicialmente ele absorve

calor e não aumenta de temperatura.

b) no gelo, porque ele tem maior calor

específico do que a água.

c) na água, porque o equilíbrio térmico se dará a

uma temperatura menor.

d) na água, porque ela tem maior calor

específico que o gelo.

e) no gelo, porque ele contém menos calor do

que a água.

Resolução: Enquanto o gelo estiver derretendo

sua temperatura permanece constante,

abaixando apenas a temperatura do refrigerante.

(Resposta A)

97) (UFES – ES) Quantas calorias são

necessárias para vaporizar 1,00 litro de água, se

a sua temperatura é, inicialmente, igual a 10,0

°C?

Dados:

- calor específico da água: 1,00 cal/g°C;

- densidade da água: 1,00 g/cm³;

- calor latente de vaporização da água: 540

cal/g.

a) 5,40 × 104 cal

b) 6,30 × 105 cal

c) 9,54 × 106 cal

d) 5,40 × 101 cal

e) 6,30 × 10² cal

Resolução: Q = m.c.∆ + m.LV = 1000.1.(100 –

10) + 1000.540 = 90000 + 540000 = 630000cal

ou 6,30 × 105 cal. (Resposta B)

98) (UNICAMP – SP) Em um dia quente, um

atleta corre dissipando 750 W durante 30 min.

Suponha que ele só transfira esta energia para o

meio externo através da evaporação do suor e

que todo o seu suor seja aproveitado para sua

refrigeração. Adote L = 2 500 J/g para o calor

latente de evaporação da água na temperatura

ambiente.

a) Qual é a taxa de perda de água do atleta em

kg/min?

b) Quantos litros de água ele perde em 30 min

de corrida?

Resolução: a) Pot = Q/∆t 750 = Q/30.60

Q = 1350000 J. A massa de água Q = m.L

1350000 = m.2500 m = 540 g. A taxa será

540g/30 min = 18g/min = 0,018 kg/min.

b) Em 30 min perde 0,018 x 30 = 0,540 g o que

corresponde a 0,54 litros.

(Respostas: a) 18g/min; b) 0,54 litros)

99) (PUC – RS) Colocam-se 420g de gelo a 0°C

num calorímetro com água a 30°C. Após

atingida a temperatura de equilíbrio térmico,

verifica-se que sobraram 20g de gelo. Sendo de

80cal/g o calor de fusão da água, é correto

afirmar que a temperatura final de equilíbrio

térmico e a quantidade de calor ganho pelo gelo

são, respectivamente,

a) 30°C e 50kcal

b) 30°C e 45kcal

c) 15°C e 40kcal

d) 0°C e 38kcal

e) 0°C e 32kcal

Resolução: O equilíbrio térmico se deu a 0 oC

já que no recipiente sobraram ainda 20 g de

gelo.

A quantidade de calor absorvida pelo gelo é Q =

m.L = (420 – 20).80 = 32000 cal ou 32 kcal..

(Resposta E)

100) (UERJ – RJ) Uma menina deseja fazer um

chá de camomila, mas só possui 200 g de gelo a

0 oC e um forno de microondas cuja potência

máxima é 800 W. Considere que a menina está

no nível do mar, o calor latente de fusão do gelo

é 80 cal/g , o calor específico da água é 1

cal/goC e que 1 cal vale aproximadamente 4

joules.

Usando esse forno sempre na potência máxima,

o tempo necessário para a água entrar em

ebulição é:

a) 45 s.

b) 90 s.

c) 180 s.

d) 360 s.

23

Resolução: Pot = (Q1 + Q2)/∆t 800/4 =

(m.L + m.c.∆)/∆t 200 = (200.80 +

200.1.100)/∆t ∆t = 180 s. (Resposta C)

101) (UERJ – RJ) Uma bolinha de aço a 120º C

é colocada sobre um pequeno cubo de gelo a 0º

C.

Em escala linear, o gráfico que melhor

representa a variação, no tempo, das

temperaturas da bolinha de aço e do cubo de

gelo, até alcançarem um estado de equilíbrio, é:

Resolução: Inicialmente, enquanto derrete, a

temperatura do gelo se mantém constante. Após

isso a água resultante tem sua temperatura

elevada até o equilíbrio térmico com a bolinha

de aço. (Resposta D)

102) (UERJ – RJ) Quatro esferas metálicas e

maciças, E1 , E2 , E3 e E4 , todas com a mesma

massa, são colocadas simultaneamente no

interior de um recipiente contendo água em

ebulição. A tabela abaixo indica o calor

específico e a massa específica do metal que

constitui cada esfera.

Atingido o equilíbrio térmico, essas esferas são

retiradas da água e colocadas imediatamente na

superfície de um grande bloco de gelo que se

encontra na temperatura de fusão.

A esfera que fundiu a maior quantidade de gelo

e a esfera que produziu a cavidade de menor

diâmetro no bloco de gelo é respectivamente:

a) E3 ; E4

b) E2 ; E4

c) E1 ; E3

d) E1 ; E2

Resolução: Com as esferas aquecidas a 100 ºC,

a de maior calor específico deverá ceder uma

quantidade maior de calor ao gelo até o

equilíbrio térmico. Assim a esfera que forma a

maior cavidade é a E1. Como as massas são

iguais, a esfera que produzirá a cavidade de

menor volume e, conseqüentemente, de menor

diâmetro é a de maior densidade, ou seja, E3.

(Resposta C)

103) (UFGO – GO) Em um copo grande,

termicamente isolado, contendo água à

temperatura ambiente (25ºC), são colocados 2

cubos de gelo a 0ºC. A temperatura da água

passa a ser, aproximadamente, de 1ºC. Nas

mesmas condições se, em vez de 2, fossem

colocados 4 cubos de gelo iguais aos anteriores,

ao ser atingido o equilíbrio, haveria no copo:

a) apenas água acima de 0ºC

b) apenas água a 0ºC

c) gelo a 0ºC e água acima de 0ºC

d) gelo e água a 0ºC

e) apenas gelo a 0ºC

Resolução: A quantidade de calor cedida pelo

gelo será suficiente para reduzir a temperatura

1ºC ficando o sistema estabilizado a 0 oC com

parte sólida e parte líquida, pois a 0 oC os

fluxos de calor trocados entre as partes se

tornam iguais, (Resposta D)

104) (UFF – RJ) Gelo seco nada mais é que gás

carbônico (CO2) solidificado e sua aplicação vai

de efeitos especiais em shows à conservação de

alimentos. Tal substância é conhecida desde

meados do século XIX e recebeu esse nome

devido ao fato de não passar pela fusão, quando

submetida à pressão atmosférica e à temperatura

ambiente, como ocorre com o gelo comum.

Considere um cubo de 0,10 kg de gelo seco, a -

78 oC, e um bloco de gelo comum de 1,0 kg, a -

10 oC,colocados em um recipiente.

Desprezando a capacidade térmica do recipiente

e a troca de calor com o ambiente:

a) determine a temperatura de equilíbrio

térmico;

b) descreva os elementos que comporão o

sistema no equilíbrio térmico.

Dados:

Temperatura de sublimação do gelo seco = - 78 oC

Temperatura de fusão do gelo comum = 0 oC

Calor latente de vaporização do gelo seco = 134

cal/g

Calor específico do vapor de gelo seco = 0,20

cal/g oC

Calor específico do gelo comum = 0,50 cal/g oC

Resolução:

24

a) Abaixando a temperatura do gelo comum até

a do gelo seco, temos: Q = m.c.∆ = 1000.0,5. (-

78 – (-10)) = - 34000 cal.

Cedendo pouco a pouco esta quantidade de

calor ao sistema, temos, para sublimar o gelo

seco:

Q = m.LS = 100.134 = 13400cal. Sobram,

portanto, 34000 – 13400 = 20600 cal. No

recipiente temos vapor de gelo seco e gelo

comum, ambos a -78 ºC. Com o calor restante, a

temperatura do sistema fica: Q = m.c.∆ +

m.c.∆ 20600 = 1000.0,5. ( – (-78)) +

100.0,2. ( – (-78)) 20600 = 500 +

39000 + 20 + 1560 -19960 = 520 =

-38,38 ºC.

b) No recipiente temos vapor de gelo seco e

gelo comum a -38,38 ºC.

(Resposta: a) -38,38 ºC.; b) vapor de gelo seco

e gelo comum)

105) (FATEC – SP - Modificado) Um frasco

contém 20 g de água a 0 °C. Em seu interior é

colocado um objeto de 50 g de alumínio a 80

°C. Os calores específicos da água e do

alumínio são respectivamente 1,0 cal/g°C e 0,10

cal/g°C e o calor latente de vaporização da água

é 540 cl/g.

Supondo não haver trocas de calor com o frasco

e com o meio ambiente, após atingida a

temperatura de equilíbrio, Qual a quantidade de

calor necessária para que toda toda a água do

sistema vaporize?

a) 12,9 kcal

b) 10 kcal

c) 5,4 kcal

d) 12,48 kcal

e) 6,24 kcal.

Resolução: QA + QB = 0 mA.cA.∆A +

mB.cB.∆B = 0 20.1.( - 0 ) + 50.0,1.( -

80) = 0 20 + 5 - 400 = 0 = 16 ºC.

Assim Q = mA.cA.∆A + mA.LV + mB.cB.∆B =

20.1.(100 – 16) + 20.540 + 50.0,1.(100 – 16) =

1680 + 10800 + 420 = 12900 cal ou 12,9 kcal.

(Resposta A)

106) (UFPR – PR) Um esquiador desce, com

velocidade constante, uma encosta com

inclinação de 30° em relação à horizontal. A

massa do esquiador e de seu equipamento é 72

kg. Considere que todo o calor gerado pelo

atrito no movimento seja gasto na fusão da

neve, cujo calor latente de fusão é 3,6 x 105

J/kg, e suponha a aceleração da gravidade igual

a 10 m/s2. Determine a massa de neve fundida

após o esquiador descer 90 m na encosta.

Expresse o resultado em gramas.

Resolução: A energia dissipada durante a

descida é Q = m.g.h = 72.10.90.sen 30º = 32400

J.

Na descida de 90 metros, a quantidade de gelo

fundida será Q = m.LF 32400 = m. 3,6 x

105 m = 0,09 kg = 90 g. (Resposta: 90 g)

Complementos de mudanças

de fases (estados físicos)

01) Conhecer os processo de mudanças de

estados físicos da matéria requer alguns

conhecimentos básicos. O primeiro deles é saber

qual a denominação de cada uma das mudanças

e qual a influência da pressão na temperatura de

mudança de estado. Assim, analise as

afirmações seguintes e aponte a única incorreta:

a) Qualquer passagem da fase líquida para a

fase gasosa é denominada evaporação.

b) A passagem da fase gasosa para a fase líquida

é denominada condensação ou liquefação.

c) Fusão é a passagem do estado sólido para o

estado líquido.

d) A passagem do estado gasoso para o estado

sólido recebe o nome de cristalização.

e) A evaporação de um líquido ocorre a uma

temperatura inferior a denominada temperatura

de ebulição.

Resolução: Vaporização é a passagem da fase

líquida para a gasosa e pode ocorrer de três

maneiras distintas; por ebulição, por evaporação

ou por calefação. (Resposta A)

02) (ENEM) A adaptação dos integrantes da

seleção brasileira de futebol à altitude de La Paz

foi muito comentada em 1995, por ocasião de

um torneio, como pode ser lido no texto abaixo.

“A seleção brasileira embarca hoje para La

Paz, capital da Bolívia, situada a 3.700 metros

de altitude, onde disputará o torneio

Interamérica. A adaptação deverá ocorrer em

um prazo de 10 dias, aproximadamente. O

organismo humano, em altitudes elevadas,

necessita desse tempo para se adaptar,

evitando-se, assim, risco de um colapso

circulatório.” (Adaptado da revista Placar, edição fev.1995)

A adaptação da equipe foi necessária

principalmente porque a atmosfera de La Paz,

quando comparada à das cidades brasileiras,

apresenta:

a) menor pressão e menor concentração de

oxigênio.

b) maior pressão e maior quantidade de

oxigênio.

c) maior pressão e maior concentração de gás

carbônico.

d) menor pressão e maior temperatura.

e) maior pressão e menor temperatura.

25

Resolução: À medida que a altitude aumenta

ocorre uma diminuição na pressão atmosférica e

uma conseqüente diminuição do nível de

oxigênio. (Resposta A)

03) (PUC – RS) A temperatura de fusão de uma

substância depende da pressão que é exercida

sobre a mesma substância. O aumento de

pressão sobre um corpo ocasiona, na sua

temperatura de fusão:

a) um acréscimo, se o corpo, ao se fundir, se

expande.

b) um acréscimo, se o corpo, ao se fundir, se

contrai.

c) um decréscimo, se o corpo, ao se fundir, se

expande.

d) um decréscimo para qualquer substância.

e) um acréscimo para qualquer substância.

Resolução: As substâncias ditas com

comportamento normal (se expandem ao fundir)

sofrem uma aumento em sua temperatura de

fusão e ebulição quando ocorre um aumento de

pressão. (Resposta A)

04) Em um passado recente o Paraná foi o maior

produtora de café do mundo, cultura essa que

promoveu o desenvolvimento acentuado de

várias cidades produtoras no estado. A falta de

subsídios para as lavouras de café, a

insegurança dos agricultores frente às políticas

de preços propostas pelo governo e a

possibilidade de destruição da lavoura de café

frente às geadas levaram a erradicação quase

que total das plantações de café, dando lugar a

outros tipos de lavouras como a soja, o trigo e o

algodão, além da formação de enormes áreas de

pastagens.

A geada, um tormento para os proprietários de

certos tipos de lavouras tem sua formação e

ação explicada a seguir de três modos.

Distintos:

I- O vapor-d'água existente na atmosfera sofre

condensação, precipitando-se, suavemente, nas

plantas, sob a forma de pequenos cristais de

gelo.

II- A água existente no interior das plantas

congela quando a temperatura ambiente atinge

um valor muito pequeno, aumentando seu

volume e rompendo células e vasos que

compõem a planta.

III- O vapor-d'água existente na atmosfera

condensa-se em contato com a planta,

transformando-se em pequenas gotículas

(orvalho). Com a queda de temperatura, o

orvalho congela (solidifica) e, ao fato,

denominamos geada.

a) apenas I é correta;

b) apenas II é correta;

c) apenas III é correta:

d) existem duas afirmações corretas apenas

e) as três afirmações estão corretas.

Resolução: A condensação dos vapores em

contato com a planta e a conseqüente queda de

temperatura ocasiona o congelamento das

gotículas e também da água existente no interior

das plantas. Ao congelar, a água aumenta seu

volume rompendo os vasos, resultando na morte

da planta. (Resposta D)

05) (PUC-RS) Há uma relação entre a pressão e

a temperatura nas quais ocorrem as mudanças

de fase. Assim, é correto afirmar que a

temperatura de:

a) fusão do gelo é superior a 0ºC, quando a

pressão é superior a 1atm.

b) fusão do gelo é inferior a 0ºC, quando a

pressão é superior a 1atm

c) fusão do gelo é sempre 0ºC, independente da

pressão.

c) ebulição da água é inferior a 100ºC, quando a

pressão é superior a 1atm.

e) ebulição da água é sempre 100ºC,

independente da pressão.

Resolução: O diagrama representa as fases da

água. Observe que sob pressão de 1 atm o ponto

de fusão é 0 oC e o de ebulição, 100

oC. O

aumento de pressão acarreta uma queda na

temperatura de fusão e um aumento da

temperatura de ebulição. (Resposta B)

06) (UFPI – PI) A experiência do regelo, feita

pela primeira vez pelo físico John Tyndal

(1820-1893), pode facilmente ser reproduzida

da seguinte forma:

- coloque um bloco de gelo a aproximadamente

–2 °C, apoiado sobre dois suportes de madeira,

de modo que a parte central do bloco fique livre;

- prenda pesos nas extremidades de um fio fino

de arame e,

- coloque este fio transversalmente sobre a parte

central do bloco de gelo, deixando os pesos

pendentes.

Observe que o fio de arame atravessa o bloco de

gelo sem que este fique dividido em duas partes.

A explicação para essa observação experimental

é:

a) O arame, estando naturalmente mais

aquecido, funde o gelo, que perde calor para o

ambiente, deixando o bloco novamente sólido.

26

b) O acréscimo de pressão no contato fio-gelo

diminui a temperatura de fusão e produz

derretimento do gelo sob o fio; este, por sua vez,

desloca-se através da água formada, a qual se

congela ao voltar à pressão normal.

c) A pressão exercida pelo arame sobre o gelo

aumenta seu ponto de fusão que, ao ser atingido,

ocorre o degelo. Assim, o fio se desloca através

da água formada, a qual se congela depois que o

fio a atravessa.

d) O ponto de fusão do gelo independe da

pressão do arame sobre ele, o degelo ocorre

simplesmente por causa do contato de

substâncias diferentes, no caso fio-gelo.

Cessado o contato, a água volta a congelar.

e) O acréscimo de pressão sobre o bloco

aumenta a temperatura de fusão e produz

derretimento do gelo sob o fio; este se desloca

através da água formada, a qual perde calor para

o ambiente e congela novamente.

Resolução: O aumento de pressão ocasionado

pelo arame faz com que o gelo sob ele funda

devido a uma diminuição na sua o temperatura

de fusão (no caso inferior a -2º C). A água

resultante desliza sobre o arame retornando a

pressão inicial e congelando imediatamente.

(Resposta B)

07) Você acorda num dia frio, prepara-se e vai

para a escola ou trabalho caminhando,

naturalmente bem agasalhado. Você repara que,

ao respirar, “solta” uma pequena nuvem de

"fumaça" durante a aspiração do ar.

Três possíveis explicações são formuladas para

o fato.

I- Durante a respiração, seu organismo separa o

vapor-d'água do restante dos componentes

aproveitáveis do ar atmosférico. Tal vapor, por

não ser aproveitado, é expelido, o que dá a

impressão da formação de uma pequena “nuvem

de fumaça”.

II- A respiração nos dias frios exige um grande

esforço. A energia necessária para que você

respire vem da queima dos alimentos que

ingere. Como a queima é muito grande, ocorre a

formação de fumaça de vapor cuja quantidade é

muito intensa ao ser expelida.

III- Quando respiramos, aspiramos o ar

atmosférico, expelindo-o em seguida. O vapor-

d'água expelido, em contato com o meio

ambiente, cuja temperatura é menor, a ele cede

calor, condensando-se sob a forma de pequenas

gotículas que, agrupadas, parecem uma pequena

nuvem.

a) apenas I é correta;

b) apenas II é correta;

c) apenas III é correta:

d) existem duas afirmações corretas apenas

e) as três afirmações estão corretas.

Resolução: A “fumaça” é simplesmente o vapor

d’água expelido que condensa sob a forma de

pequenas gotículas formando a “nuvem”.

(Resposta: C)

08) Uma garrafa de vidro cheia de água pode

quebrar, dentro do congelador, por que:

a) a água, ao se congelar, sofre aumento de

volume, e suas moléculas exercem uma força

muito intensa sobre as paredes da garrafa de

vidro.

c) a água quimicamente destrói as paredes do

vidro;

d) a garrafa não pode ter temperatura baixa;

e) o gelo externo à garrafa exerce pressão muito

maior que o gelo interno.

Resolução: O congelamento da água ocasiona

sua expansão durante a formação do gelo. As

moléculas, com isso, exercem forças imensas

nas paredes do recipiente, rompendo-as.

(Resposta A)

09) A figura seguinte indica os diferentes pontos

de ebulição em algumas cidades localizadas em

altitudes diferentes.

a) Qual a relação entre o ponto de ebulição e a

altitude das cidades?

b) Em Santos e em Quito, a água a 0º C se

encontraria em qual estado físico? Justifique.

Resolução: a) À medida que a altitude aumento

ocorre uma queda na pressão e com isso uma

redução na temperatura de ebulição.

b) Em Quito a água está no estado sólido, pois

sua temperatura de fusão é maior que 0 oC já

que a pressão é menor que a atmosférica. Em

Santos a água pode estar no estado sólido,

líquido ou em transição pois 0 oC é justamente

sua temperatura de fusão.

(Respostas: a) Diminui com o aumento da

altitude; b) Quito: estado sólido. Santos:

estado sólido, líquido ou em transição)

10) A decomposição de inúmeras rochas que se

fragmentam em pedaços menores deve-se ao

fato de possuírem fendas ou porosidades que

permitem a infiltração de água. Nos países de

clima muito frio, ou quando ocorre grande

queda de temperatura observa-se a

fragmentação mais acentuada.

Explique fisicamente por que isto ocorre.

Resolução: A água infiltrada nas rochas

congela e durante a formação do gelo ocorre sua

27

expansão e o rompimento das fendas e poros.

(Resposta: Devido ao congelamento da água)

11) (UFPI – PI) Podemos conhecer melhor uma

substância através de seu diagrama de fases, que

são curvas que delimitam as regiões

correspondentes às fases sólida, líquida e gasosa

da substância. A figura abaixo mostra o

diagrama de fase da água, líquido vital aos seres

vivos e que é estudado universalmente. Analise

o diagrama de fase mostrado e assinale, nas

afirmativas, V (para verdadeiro) ou F (para

falso).

1( ) Sob pressão de 4,58 mmHg a à temperatura

de 0,01 ºC há a coexistência das três fases da

água em equilíbrio: sólida, líquida e vapor. Esse

ponto é o ponto triplo.

2( ) A curva AT do diagrama representa a curva

de sublimação, cujos pontos representam os

estados de equilíbrio entre as fases sólida e

liquida.

3( ) À pressão de 2,51 mmHg e à temperatura

ambiente de 20 ºC, o diagrama mostra que a

água está no estado de vapor.

4( ) A curva TB do diagrama mostra que a

fusão é facilitada pelo aumento da pressão.

Resolução:

1)(V) No ponto triplo (intersecção das três

curvas) as condições de temperatura e pressão

determinam a coexistência dos três estados

físicos.

2(F) Entre os estados sólido e gasoso (no caso,

vapor).

3(V) o ponto (pxV) estará determinado abaixo

das curvas AT e TC.

4(V) Já que o aumento de pressão ocasiona uma

queda na temperatura de fusão.

(Resposta: 1(V); 2(F); 3(V); 4(V))

12) (UFMS – MS) Uma cozinheira resolve

ferver água em uma panela de pressão, para

atingir maior temperatura que em panelas

abertas. Para isso, coloca água no seu interior,

onde todo o sistema, panela e água, está em

equilíbrio térmico com o ambiente na

temperatura TA e pressão atmosférica pA. Em

seguida, fecha a panela e coloca-a sobre a

chama de um fogão. A água, no interior da

panela, vai aumentando a pressão e a

temperatura e, após certo tempo entra em

ebulição liberando vapores para o ambiente pela

válvula.

As figuras abaixo representam diagramas da

pressão p x temperatura T da água, onde as

linhas mais finas definem as regiões, de

temperatura e pressão em que a água pode se

apresentar em cada um dos possíveis estados,

sólido (S), líquido (L) e vapor (V). Já as linhas

mais grossas (negrito) representam processos

termodinâmicos.

Assinale qual dos diagramas representa

corretamente o processo termodinâmico que a

água, no interior da panela, sofreu desde o

instante em que começou a ser aquecida,

quando estava à temperatura TA e pressão pA,

até o momento em que entra em ebulição à

temperatura TB e Pressão pB.

Resolução: Partindo do ponto pA,TA (estado

líquido) a água vai gradativamente sofrendo

aumento de pressão e temperatura determinando

na curva de ebulição o ponto pB,TB conforme o

diagrama D. (Resposta D)

13) As afirmações a seguir referem às

substâncias que têm comportamento normal, ou

seja, aumentam de volume durante a fusão.

Analise cada uma delas e indicando se é

verdadeira (V) ou falsa (F).

28

a( ) Caso a substância esteja na fase sólida, sem

alterar a temperatura, ela poderá fundir,

bastando uma diminuição na pressão.

b( ) Caso a substância esteja na fase sólida, sem

alterar a temperatura, ela poderá sublimar,

bastando uma diminuição na pressão.

c( ) Caso a substância esteja na fase líquida,

mantida a pressão constante, e elevando a

temperatura, ela poderá vaporizar,

transformando-se em um vapor.

d( ) Caso a substância seja um vapor, mantida a

pressão constante ele poderá cristalizar devido a

uma queda na temperatura ou transformar-se em

um gás, devido a uma elevação na temperatura.

e( ) Caso a substância esteja submetida à suas

temperatura e pressão tríplices, ou seja, com os

três estados físicos coexistindo, um pequeno

aumento na pressão será suficiente para que

toda a substância passe para a fase sólida.

Resolução:

a(V) b(V) c)(V)

d(V) e(V)

(Resposta: a(V); b(V); C(V); d(V); e(V))

14) (ENEM) Com base em projeções realizadas

por especialistas, prevê-se, para o fim do século

XXI, aumento de temperatura media, no

planeta, entre 1,4 ºC e 5,8 ºC.

Como conseqüência desse aquecimento,

possivelmente o clima será mais quente e mais

úmido bem como ocorrerão mais enchentes em

algumas áreas e secas crônicas em outras. O

aquecimento também provocará o

desaparecimento de algumas geleiras, o que

acarretará o aumento do nível dos oceanos e a

inundação de certas áreas litorâneas.

As mudanças climáticas previstas para o fim do

século XXI

a) provocarão a redução das taxas de

evaporação e de condensação do ciclo da água.

b) poderão interferir nos processos do ciclo da

água que envolvem mudanças de estado físico.

c) promoverão o aumento da disponibilidade de

alimento das espécies marinhas.

d) induzirão o aumento dos mananciais, o que

solucionara os problemas de falta de água no

planeta.

e) causarão o aumento do volume de todos os

cursos de água, o que minimizara os efeitos da

poluição aquática.

Resolução:

15) (CEFET – PR) Analise as afirmativas a

seguir, relacionadas com o estudo da

termologia:

I) Nos dias frios, o pára-brisa de um automóvel

fica embaçado porque o vapor de água existente

no ar se condensa no vidro frio.

II) Para liquefazer um gás é necessário reduzir a

pressão e aumentar a temperatura.

III) Para que a água contida em um recipiente,

inicialmente à temperatura e pressão ambientes,

entre em ebulição, devemos necessariamente

aumentar a pressão e a temperatura.

Podemos afirmar que:

a) somente a afirmativa I é correta.CORRETA

b) somente a afirmativa II é correta.

c) somente a afirmativa III é correta.

d) todas as afirmativas são corretas.

e) todas as afirmativas são incorretas.

Resolução:

I(V) O embaçamento é fruto da condensação

dos vapores no parabrisas do automóvel.

II(V) Reduzindo a pressão transformamos o

líquido em vapor e, em seguida, ao

aumentarmos a temperatura acima da

temperatura crítica o vapor se transforma em um

gás.

III(F) Ou apenas aumentar a temperatura ou

aumentá-la juntamente com um aumento da

pressão.

(Resposta: I(V); II(V); III(F))

16) (ENEM) A tabela a seguir registra a pressão

atmosférica em diferentes altitudes, e o gráfico

relaciona a pressão de vapor da água em função

da temperatura

29

Um líquido, num frasco aberto, entra em

ebulição a partir do momento em que a sua

pressão de vapor se iguala à pressão

atmosférica. Assinale a opção correta,

considerando a tabela, o gráfico e os dados

apresentados, sobre as seguintes cidades:

Natal (RN) nível do mar.

Campos do Jordão (SP) altitude 1628m.

Pico da Neblina (RR) altitude 3014 m.

A temperatura de ebulição será:

a) maior em Campos do Jordão.

b) menor em Natal.

c) menor no Pico da Neblina.

d) igual em Campos do Jordão e Natal.

e) não dependerá da altitude.

Resolução: Os gráficos mostram que a pressão

no Pico da Neblina é de aproximadamente 390

mmHg o que corresponde a uma temperatura de

ebulição de mais ou menos 80 ºC. (Resposta C)

17) (UFC – CE) Ao nível do mar, a água ferve a

100 ºC e congela a 0 ºC. Assinale a alternativa

que indica o ponto de congelamento e o ponto

de fervura da água, em Guaramiranga, cidade

localizada a cerca de 1000 m de altitude.

a) A água congela abaixo de 0 ºC e ferve acima

de 100 ºC.

b) A água congela acima de 0 ºC e ferve acima

de 100 ºC.

c) A água congela abaixo de 0 ºC e ferve abaixo

de 100 ºC.

d) A água congela acima de 0 ºC e ferve abaixo

de 100 ºC.

e) A água congela a 0 ºC e ferve a 100 ºC.

Resolução: O aumento da altitude acarreta uma

diminuição na pressão e, em conseqüência, um

aumento na temperatura de fusão do gelo e

diminuição na temperatura de ebulição da água.

Portanto a água congela acima de 0 oC e ferve a

menos de 100 ºC. (Resposta C)

18) (ENEM) A panela de pressão permite que

os alimentos sejam cozidos em água muito mais

rapidamente do que em panelas convencionais.

Sua tampa possui uma borracha de vedação que

não deixa o vapor escapar, a não ser através de

um orifício central sobre o qual assenta um peso

que controla a pressão. Quando em uso,

desenvolve-se uma pressão elevada no seu

interior. Para a sua operação segura, é

necessário observar a limpeza do orifício central

e a existência de uma válvula de segurança,

normalmente situada na tampa.

O esquema da panela de pressão e um diagrama

de fase da água são apresentados abaixo.

A vantagem do uso de panela de pressão é a

rapidez para o cozimento de alimentos e isto se

deve

a) à pressão no seu interior, que é igual à

pressão externa.

b) à temperatura de seu interior, que está acima

da temperatura de ebulição da água no local.

c) à quantidade de calor adicional que é

transferida à panela.

d) à quantidade de vapor que está sendo liberada

pela válvula.

e) à espessura da sua parede, que é maior que a

das panelas comuns

Resolução: Dentro da panela em operação, a

água está acima de sua temperatura de ebulição

sob pressão normal (100º C). O motivo se deve

ao fato de a temperatura de ebulição da água

aumentar com a elevação da pressão. (Resposta

B)

19) (UFF – RJ) Nas cidades I e II não há

tratamento de água e a população utiliza a

ebulição para reduzir os riscos de contaminação.

A cidade II situa-se a 3000 m de altitude em

relação à cidade I que, por sua vez, localiza-se

ao nível do mar.

Relativamente a essas duas cidades, é correto

afirmar que a temperatura da água em ebulição

numa panela aberta:

a) é menor na cidade I porque, nessa cidade, a

pressão atmosférica é menor;

b) é menor na cidade II porque, nessa cidade, a

pressão atmosférica é maior;

c) é a mesma nas cidades I e II porque a pressão

atmosférica não influi no valor da temperatura

de ebulição da água;

d) é maior na cidade I porque, nessa cidade, a

pressão atmosférica é maior;

e) é maior na cidade II porque, nessa cidade, a

pressão atmosférica é menor.

Resolução: A cidade I por estar situada numa

altitude menor tem a água submetida a uma

maior pressão o que também torna maior sua

temperatura de ebulição. (Resposta E)

20) O diagrama, a seguir, mostra a compressão

isotérmica de duas substâncias, A e B, ambas,

inicialmente, na fase gasosa.

30

Sendo assim, analise cada uma das afirmações a

seguir e registre se é falsa ou verdadeira.

a- A substância A é, inicialmente, um vapor.

b- A substância B é, inicialmente, um gás.

c- A substância A encontra-se a uma

temperatura inferior à sua crítica.

d- A substância B encontra-se a uma

temperatura igual ou superior à sua crítica.

e- A substância A sofre condensação durante

sua compressão.

Resolução: A: estado gasoso sob a forma de um

vapor (temperatura abaixo da crítica),

condensando por compressão isotérmica; B:

estado gasoso sob a forma de um gás

(temperatura acima da crítica), não podendo

condensar por compressão isotérmica. Portanto:

a-V; b-V; c-V; d-V;e-V. (Resposta: a-V; b-V;

c-V; d-V;e-V)

21) (IFBA – BA) “Quem desembarca numa

cidade de altitude elevada costuma sentir logo

de cara o impacto do ar rarefeito em nosso

organismo. Em alguns casos, os efeitos são

perigosos

– um ministro brasileiro desmaiou em uma

solenidade realizada em La Paz, na Bolívia, que

está a 3660 metros de altitude

Quanto maior é a altitude, mais rarefeito é o ar

que respiramos. Também há uma redução na

temperatura, que cai em média 6,5 graus a cada

1.000 metros de altura”. Adaptado de: http://veja.abril.com.br/

Em locais de grandes altitudes, o ar é rarefeito,

pois:

a) a força gravitacional é maior do que em

locais ao nível do mar.

b) a pressão atmosférica é maior, devido à altura

em relação ao nível do mar.

c) a pressão atmosférica é menor, devido à

dilatação dos gases com a redução da

temperatura.

d) as altas temperaturas, comuns nesses locais,

favorecem a evaporação do ar.

e) a pressão atmosférica é menor, devido à

redução da coluna de ar sobre qualquer corpo,

com o aumento da altitude.

Resolução: A atmosfera é uma massa fluida

composta de vapores e gases. À medida que a

altitude aumenta, a coluna fluida acima de

qualquer corpo diminui, ocasionando uma

pressão atmosférica menor. (Resposta: E)

22) (ENEM Os seres humanos podem tolerar

apenas certos intervalos de temperatura e

umidade relativa (UR), e, nessas condições,

outras variáveis, como os efeitos do Sol e do

vento, são necessárias para produzir condições

confortáveis, nas quais as pessoas podem viver

e trabalhar. O gráfico mostra esses intervalos:

Com base nessas informações, pode-se afirmar

que condições ideais são observadas em

a) Curitiba com vento em março, e Campo

Grande, em outubro.

b) Campo Grande com vento em março, e

Curitiba com sol em maio.

c) Curitiba, em outubro, e Campo Grande com

sol em março.

d) Campo Grande com vento em março,

Curitiba com sol em outubro.

e) Curitiba, em maio, e Campo Grande, em

outubro.

Resolução: através da tabela e do gráfico

temos:

Curitiba: março: vento; maio: não ideal;

outubro: Sol

Campo Grande; março: não ideal; maio: Sol;

outubro: ideal. (Resposta: A)

23) Sabe-se que o gelo seco (gás carbônico

congelado) é amplamente utilizado em

espetáculos teatrais, além de situações que

exigem a criação de um cenário diferente. Uma

enorme cortina de fumaça envolve certa região

circunvizinha ao local em que o gelo seco,

colocado em um recipiente com água, foi

exposto ao meio ambiente.

31

Três explicações são dadas ao fato exposto.

I - O gelo seco ferve a água do recipiente em

que foi depositado. A água, ao ferver,

transforma-se em vapor, expandindo-se pelo

ambiente.

II - O gelo seco, em contato com a água do

recipiente, sofre sublimação, transformando-se

em vapor e expandindo-se pelo ambiente. A

nuvem de "fumaça" formada é, simplesmente, o

vapor oriundo da sublimação do gelo seco.

III - O gelo seco, em contato com a água do

recipiente, sofre sublimação. Tal transformação,

por ser endotérmica, exige certa quantidade de

calor. Este calor é cedido, a ele, pelo vapor-

d'água existente próximo ao local onde foi

colocado o recipiente. O vapor-d'água, ao ceder

calor para que o gelo seco, condensa-se, com a

formação de pequenas gotículas d'água (nuvem

ou cerração), dando os efeitos visuais previstos.

Indique a alternativa correta:

a) apenas I é correta;

b) apenas II é correta;

c) apenas III é correta:

d) existem duas afirmações corretas apenas

e) as três afirmações estão corretas.

Resolução: A opção III explica fielmente o fato

da formação das “nuvens”. (Resposta C)

24) (UFRN – RN) Um cliente assíduo de uma

cafeteria só gosta de tomar café frio. Por isso,

sempre que lhe servem uma xícara de café

quente, para apressar o processo de

resfriamento, ele sopra a superfície do líquido

até diminuir a temperatura.

Baseado no modelo cinético molecular, ele

consegue esfriar o café porque, ao soprá-lo,

a) diminui o calor específico do líquido.

b) aumenta o processo de condensação do

líquido.

c) diminui o calor latente do líquido.

d) aumenta o processo de evaporação do

líquido.

Resolução: Ao assoprar, o ambiente torna-se

menos saturado, facilitando a evaporação do

líquido. Este, ao evaporar, absorve calor cedido

pelo café, resfriando-o. (Resposta: D)

25) Gás carbônico e água encontram-se

submetidos às suas respectivas pressão e

temperatura tríplices, ou seja, dentro dos

recipientes em que se encontram, os três estados

físicos coexistem. O que acontece com cada

uma das referidas substâncias, quando a

temperatura for mantida constante e:

a pressão aumentar;

a pressão diminuir.

Resolução:

Gás carbônico Água

a) CO2 torna-se sólido e água torna-se líquida.

b) CO2 torna-se vapor e água torna-se vapor.

(Resposta: a) CO2: sólido e H20: líquida; b)

CO2: e H20 tornam-se vapores)

26) Sabe-se que as sensações térmicas frio e

quente, percebidas por uma pessoa, devem-se à

velocidade com que a pessoa cede ou recebe

calor. Quando alguém está febril, é comum,

molhar um pano com álcool ou água e, em

seguida passá-lo no corpo da pessoa.

Sendo assim:

a- O fato de o álcool e a água estarem a uma

temperatura inferior à da pessoa faz com que o

frio passe do pano para a pessoa.

b- O álcool e a água em contato com o corpo

da pessoa sofrem cristalização, esfriando a

pessoa.

c- A evaporação do álcool ou da água que

molha a pessoa é uma transformação

endotérmica. Tais substâncias, ao evaporarem,

recebem calor da pessoa, a qual, por sua vez,

devido ao fato de ceder calor ao álcool ou à

água, tem sua temperatura diminuída.

d- Todo líquido encontra-se a uma

temperatura menor que à das pessoas.

e- A condensação do álcool ou da água

permite que estas substâncias, ao passarem para

o estado gasoso, recebam calor da pessoa, pois

esta o tem de sobra, já que está com febre.

Resolução: A afirmação de letra C explica

fielmente o fato. (Resposta C)

27) (ENEM) Se, por economia, abaixarmos o

fogo sob uma panela de pressão logo que se

inicia a saída de vapor pela válvula, de forma

simplesmente a manter a fervura, o tempo de

cozimento.

a) será maior porque a panela “esfria”.

b) será menor, pois diminui a perda de água.

c) será maior, pois a pressão diminui.

d) será maior, pois a evaporação diminui.

e) não será alterado, pois a temperatura não

varia.

Resolução: O vapor no interior da panela

atingiu a máxima pressão suportada pela

válvula. A partir daí, mesmo continuando o

S

L

V

S

L

V

32

fornecimento de calor a temperatura da água no

interior da panela se manteria constante, já que a

pressão permaneceria constante em função da

ação da válvula. (Resposta E)

28) (UFMA – MA) As águas colocadas em

moringas de barro são mais frias do que o

ambiente por que:

a) uma pequena porção de água vaza pelos

poros e, ao evaporar, recebe calor da água que

fica.

b) o barro sempre retira calor da água quando

colocado junto dela.

c) o barro, sendo poroso, deixa escapar calor

pelos seus poros, sofrendo sublimação.

d) a água tem alto calor específico, não sofrendo

evaporação.

e) a água tem baixo calor específico, sofrendo

sublimação.

Resolução: A evaporação da água é um

processo endotérmico, ou seja, o vapor, ao se

formar, recebe calor da água do recipiente a

qual reduz sua temperatura, pois teve que ceder

calor a ele. (Resposta A)

29) Em uma cidade A, situada à beira-mar, num

certo dia de inverno e de baixa umidade do ar, a

temperatura chegou a 0 ºC sem que houvesse

formação e precipitação de neve. Mesmo assim,

a sensação de frio era intensa a tal ponto de ser

insuportável aos moradores, até mesmo aos

muito bem agasalhados.

Um dos moradores da cidade A dirigiu-se a uma

cidade B, também situada ao nível do mar. Na

cidade B, a temperatura estava caindo e, ao

atingir 0 ºC, observou-se o inicio da formação e

conseqüente precipitação de neve. O morador da

cidade A, constatou que, na cidade B, a

sensação térmica de frio era menos intensa que

na A, embora as temperaturas ambientes,

enquanto se formava neve em B, eram

aproximadamente iguais. Sendo assim, analise

as afirmações a seguir, as quais tentam explicar

o fato, e descubra a única verdadeira.

a) A neve é mais quente que as gotículas de

água existente na atmosfera.

b) A dificuldade de uma pessoa em se

movimentar na neve exige dela um esforço

muscular maior. Quanto mais a pessoa se

esforçar, menor será a sensação de frio.

c) A neve, ao cair, atrita-se com os gases

existentes na atmosfera. O trabalho do atrito

resulta na transformação de parte da energia

cinética da neve em calor, o qual aquece o meio

ambiente.

d) O vapor-d'água existente na atmosfera se

condensa sob a forma de pequenas gotículas em

suspensão. Com a queda de temperatura, as

gotículas sofrem a solidificação, precipitando-se

em pequenos flocos. Como a solidificação é um

processo exotérmico, ocorre a liberação de certa

quantidade de calor para o meio ambiente,

amenizando a sensação térmica de frio.

e) A cristalização do vapor-d'água da

atmosfera resulta na formação da neve. Como a

cristalização é uma transformação exotérmica,

ocorre a liberação de certa quantidade de calor

para o meio ambiente.

Resolução: A condensação dos vapores

formando as gotículas e a solidificação das

mesmas formando a neve são processos

exotérmicos, ou seja, ocorrem com a substância

cedendo calor, no caso, ao meio ambiente.

Assim, a sensação térmica de frio sofre é

reduzida. (Resposta D)

30) Um recipiente cilíndrico, dotado de êmbolo

móvel contém certo líquido em equilíbrio

dinâmico com seu vapor, como mostra o

esquema.

Assinale a proposição verdadeira caso ocorra

uma redução do volume quando o êmbolo for

empurrado sem que se altere a temperatura do

sistema.

a) A pressão exercida pelo vapor, enquanto

existir vapor no recipiente, aumenta

gradativamente.

b) Mais líquido vaporiza.

c) A proporção de líquido e vapor não se altera.

d) O vapor transforma-se em vapor seco.

e) Mais vapor se condensa.

Resolução: Como o equilíbrio é dinâmico, o

vapor existente é o vapor úmido o qual exerce

sua máxima pressão de vapor. Na tentativa de

aumentar a pressão deslocando o êmbolo para

baixo, apenas mais vapor se condensa para que

a pressão exercida pelo vapor se mantenha

constante e igual a máxima. (Resposta E)

31) (CEFET – PR) Evaporação é um processo

de vaporização que ocorre de forma lenta, com a

superfície do líquido exposta ao meio ambiente.

A velocidade com que ocorre a evaporação

depende de alguns fatores. Qual das alternativas

contempla 3 destes fatores?

a) Temperatura, calor específico e área da

superfície exposta ao ambiente.

b) Pressão atmosférica, ponto de ebulição e

umidade relativa do ambiente.

33

c) Pressão atmosférica, temperatura e ponto de

ebulição.

d) Temperatura, umidade relativa do ambiente e

área da superfície exposta ao ambiente.

e) Umidade relativa do ambiente, calor

específico e ponto de ebulição.

Resolução: Os fatores que influem na

velocidade de evaporação de um líquido são:

natureza do líquido, temperatura, pressão

atmosférica, área da superfície livre e umidade

relativa do ar. Portanto, a alternativa mais

completa é a de letra D. (Resposta D)

32) (ENEM) Umidade relativa do ar é o termo

usado para descrever a quantidade de vapor de

água contido na atmosfera. Ela é definida pela

razão entre o conteúdo real de umidade de uma

parcela de ar e a quantidade de umidade que a

mesma parcela de ar pode armazenar na mesma

temperatura e pressão quando está saturada de

vapor, isto é, com 100% de umidade relativa. O

gráfico representa a relação entre a umidade

relativa do ar e sua temperatura ao longo de um

período de 24 horas em um determinado local.

Considerando-se as informações do texto e do

gráfico, conclui-se que

a) a insolação é um fator que provoca variação

da umidade relativa do ar.

b) o ar vai adquirindo maior quantidade de

vapor de água à medida que se aquece.

c) a presença de umidade relativa do ar é

diretamente proporcional à temperatura do ar.

d) a umidade relativa do ar indica, em termos

absolutos, a quantidade de vapor de água

existente na atmosfera.

e) a variação da umidade do ar se verifica no

verão, e não no inverno, quando as temperaturas

permanecem baixas.

Resolução: As alterações de temperatura, em

virtude da insolação podem provocar uma

variação na umidade relativas do ar tendo em

vista uma alteração na pressão máxima do vapor

existente na atmosfera. (Resposta A)

33) (PUC – RS) A massa de vapor de água

contida num dado volume de ar pela massa de

vapor de água que este volume de ar

comportaria, na mesma temperatura, se

estivesse saturado. Num determinado recinto

onde a temperatura ambiente é de 20º C, tem-se

8,5 g/m³ de vapor de água presente no ar. Sabe-

se que ar saturado a 20º C contém cerca de 17

g/m³ de vapor de água.

A umidade relativa do ar no recinto considerado

é de

a) 8,5%

b) 10%

c) 25%

d) 40%

e) 50%

Resolução: H = f/F = m/M = 8,5/17 = 0,5 =

50%. (Resposta: E)

34) Apresentamos a seguir os valores da pressão

e da temperatura críticas para algumas

substâncias.

Sabe-se que, nas condições ambientes, todas as

substâncias apresentadas podem ser encontradas

no estado gasoso. Sendo assim, e nas condições

ambientes, indique quais são gases e quais são

vapores.

Resolução: Pressão ambiente aproximadamente

1 atm e temperatura ambiente aproximadamente

22 º C. A pressão ambiente é menor que a

crítica para as 4 substâncias. Assim, ou são

sólidas ou vapores ou gases. A temperatura

ambiente é menor que a crítica para a água e

para o CO2. Assim ambos são vapores e os

demais, oxigênio e hélio são gases. Resposta:

Gases: oxigênio e hélio; vapores: água e CO2.

35) (PUC – RS) O ar de uma sala com 400m³

contém 4,0 kg de vapor de água e, se estivesse

saturado, na mesma temperatura, o ar teria 8,0

kg de vapor de água. Assim, os valores da

umidade absoluta e da umidade relativa do ar

são, respectivamente,

a) 1,0 x 10-2

kg/m³ e 40%.

b) 1,0 x 10-2

kg/m³ e 50%.

c) 1,0 x 10-3

kg/m³ e 50%.

d) 1,0 x 10-3

kg/m³ e 60%.

e) 1,0 x 10-3

kg/m³ e 80%.

Resolução: U = M/V = 8/400 = 2.10–2

u = m/V = 4/400 = 1.10–2

kg/m³.

H = f/F = u/U = 1.10–2

/2.10–2

= 0,5 = 50%.

(Resposta: C)

36) A velocidade de evaporação de certo líquido

é 2 g/min. Para que a velocidade se torne igual a

2,4 g/min, a área de sua superfície livre deve

sofrer qual aumento porcentual ?

34

Resolução: v = K.A.(F-f)/p 3 = c. A1 e 2,4

= c. A2 3/2,4 = A1/A2 1,25 = A1/A2

A2 = 1,25 A1 = 1255 A1 (aumento percentual

de 25%). (Resposta: 25%)

37) A velocidade de evaporação de um líquido

A é o dobro da de um líquido B. Sendo a massa

específica de A igual a 0,8 g/cm³, a de B igual a

1,0 g/cm³, qual a relação entre os volumes de A

e B, evaporados num mesmo intervalo de

tempo?

Resolução: a velocidade de evaporação é v =

m/V = . V’/V vA = 0,8.V’/VA e vB =

1,0.V’/VB vB/vA = (1/0,8)VA/VB 2vB/vB

= 1,25 VA/VB 2/1,25 = VA/VB VA/VB =

1,6 (Resposta: 1,6)

38) O diagrama a seguir mostra a pressão

parcial do vapor d'água no ar, em duas cidades,

A e B. Obtenha a umidade relativa do ar em

cada uma das cidades.

Resolução:

Cidade A: HA = f/F = 0,6/0,9 0,67 ou 67%

Cidade B: HB = f/F = 0,8/1,0 = 0,8 ou 80%.

(Respostas: 67% e 80%)

39) (PUC – RS) Verifica-se que o ar de um

ambiente a 20 ºC contém 3,64kg de vapor de

água. Se estivesse saturado, também a 20 ºC,

conteria 5,20kg de vapor de água. É correto

afirmar, então, que a umidade relativa do ar do

ambiente considerado é

a) 70%

b) 60%

c) 50%

d) 40%

e) 30%

Resolução: H = f/F = m/M = 3,64/5,20 = 0,7 ou

70%. (Resposta A)

40) Um líquido colocado em um recipiente

estático, ao ser resfriado lentamente, pode

atingir, mantendo-se no estado líquido,

temperaturas inferiores à de sua solidificação,

ou seja, eventualmente, pode ocorrer um atraso

na solidificação do líquido, a tal ponto de ele

não solidificar, embora sua temperatura seja

inferior à de solidificação.

Este fenômeno é denominado sobrefusão ou

superfusão, sendo que o líquido encontra-se

num estado denominado equilíbrio meta-estável.

A simples agitação ou a imersão de um

fragmento sólido causa a imediata solidificação

de parte ou de todo o líquido, elevando sua

temperatura.

O diagrama seguinte mostra as curvas de

resfriamento para duas massas iguais de água,

submetidas a uma mesma pressão. A primeira

massa (A) solidificou sem que ocorresse a

sobrefusão enquanto a segunda massa (B) sofre

o fenômeno.

Sendo assim, suponha que 100 gramas de água

estejam em estado de sobrefusão a -20 ºC e sob

pressão de 1 atm. Determine a massa de gelo

formada caso o equilíbrio metaestável fosse

interrompido devido a uma agitação. Adote:

Calor específico da água: 1,0 cal/g.ºC

Calor latente de solidificação da água: -80 cal/g

Resolução: QXYZ = QXZ m.c.∆ = m’.LS

100.1.(-20) = m’.(-80) m’ = 25 g

(Resposta: 25 g)

35

Transmissão do calor

41) O esquema a seguir mostra uma pessoa

fervendo certa quantidade de água e aquecendo

uma frigideira com a finalidade de fritar ovos e

assim ter um suculento café da manhã. Ela

utiliza como fonte térmica certa quantidade de

lenha em combustão.

Preencha as linhas pontilhadas as quais se

referem a possíveis processos de transmissão do

calor.

Resolução: Bule: Movimentação dos vapores

convecção; Cabo da panela Calor flui de

partícula para partícula ao longo do cabo; chama

calor flui através das radiações. (Resposta:

convecção; condução e radiação)

42) (ENEM) Numa área de praia, a brisa

marítima é uma conseqüência da diferença no

tempo de aquecimento do solo e da água, apesar

de ambos estarem submetidos às mesmas

condições de irradiação solar. No local (solo)

que se aquece mais rapidamente, o ar fica mais

quente e sobe, deixando uma área de baixa

pressão, provocando o deslocamento do ar da

superfície que está mais fria (mar).

À noite, ocorre um processo inverso ao que se

verifica durante o dia.

Como a água leva mais tempo para esquentar

(de dia), mas também leva mais tempo para

esfriar (à noite), o fenômeno noturno (brisa

terrestre) pode ser explicado da seguinte

maneira:

a) O ar que está sobre a água se aquece mais; ao

subir, deixa uma área de baixa pressão,

causando um deslocamento de ar do continente

para o mar.

b) O ar mais quente desce e se desloca do

continente para a água, a qual não conseguiu

reter calor durante o dia.

c) O ar que está sobre o mar se esfria e dissolve-

se na água; forma-se, assim, um centro de baixa

pressão, que atrai o ar quente do continente.

d) O ar que está sobre a água se esfria, criando

um centro de alta pressão que atrai massas de ar

continental.

e) O ar sobre o solo, mais quente, é deslocado

para o mar, equilibrando a baixa temperatura do

ar que está sobre o mar.

Resolução: Como a água está mais aquecida, o

ar sobre ela se aquece mais, subindo e criando

uma zona de baixa pressão forçando os

deslocamentos do ar (brisas) a fluírem do solo

para o mar. (Resposta A)

43) (UEL – PR) Embalagens tipo “longa vida”

(abertas, com a parte interna voltada para cima,

embaixo das telhas) podem ser utilizadas como

material isolante em telhados de amianto, que

no verão atingem temperaturas de 70°C. Sobre

essa utilização do material, é correto afirmar:

a) O calor emitido pelas telhas de amianto é

absorvido integralmente pelo “forro longa

vida”.

c) O calor específico do “forro longa vida” é

muito pequeno, e por isso sua temperatura é

constante, independentemente da quantidade de

calor que recebe da telha de amianto.

c) A superfície de alumínio do “forro longa

vida” reflete o calor emitido pelas telhas de

amianto.

d) A camada de papelão da embalagem tipo

“longa vida” isola o calor emitido pelas telhas

36

de amianto, pois sua capacidade térmica absorve

a temperatura.

e) A superfície de alumínio do “forro longa

vida” é um isolante térmico do calor emitido

pelas telhas de amianto, pois está revestida por

uma camada de plástico.

Resolução: A parte metálica da embalagem,

voltada para as telhas de amianto, refletem o

calor irradiado por elas, amenizando a

temperatura ambiente. (Resposta C)

44) (PUC – SP) Analise as afirmações

referentes à condução térmica:

I- Para que um pedaço de carne cozinhe mais

rapidamente, pode-se introduzir nele um espeto

metálico. Isso se justifica pelo fato de o metal

ser um bom condutor de calor.

II- Os agasalhos de lã dificultam a perda de

energia (na forma de calor) do corpo humano

para o ambiente, devido ao fato de o ar

aprisionado entre suas fibras ser um bom

isolante térmico.

III- Devido à condução térmica, uma barra de

metal mantém-se a uma temperatura inferior à

de uma barra de madeira colocada no mesmo

ambiente.

Podemos afirmar que

a) I, II e III estão corretas.

b) I, II e III estão erradas.

c) Apenas I está correta.

d) Apenas II está correta.

e) Apenas I e II estão corretas.

Resolução:

I(V) O metal, por ser bom condutor, permite

que a temperatura ni interior da carne também

se eleve.

II(V) O ar possui baixo coeficiente de

condutividade térmica sendo, portanto, um bom

isolante térmico.

III(F) As temperaturas são as mesmas. Apenas a

sensação térmica é diferente.

(Resposta: E)

45) (ENEM ) O resultado da conversão direta de

energia solar é uma das várias formas de energia

alternativa de que se dispõe. O aquecimento

solar é obtido por uma placa escura coberta por

vidro, pela qual passa um tubo contendo água.

A água circula, conforme mostra o esquema

abaixo:

São feitas as seguintes afirmações quanto aos

materiais utilizados no aquecedor solar:

I- o reservatório de água quente deve ser

metálico para conduzir melhor o calor.

II- a cobertura de vidro tem como função reter

melhor o calor, de forma semelhante ao que

ocorre em uma estufa.

III- a placa utilizada é escura para absorver

melhor a energia radiante do Sol, aquecendo a

água com maior eficiência.

Dentre as afirmações acima, pode-se dizer que,

apenas está(ão) correta(s):

a) I. b) I e II. C) II. D) I e III. E) II e III.

Resolução:

I- F. Se o reservatório for de metal, a água

aquecida “cederá” calor rapidamente,

diminuindo a sua temperatura, pois o metal é

um bom condutor de calor. O reservatório deve

ser feito de um material isolante e não com um

condutor.

II- V. O vidro permite que entre a luz visível,

mas impede a saída dos raios infravermelhos

(ondas de calor) a uma estufa de plantas.

III- V. A cor escura absorve melhor as radiações

eletromagnéticas. (Resposta: E)

46) (UFRN – RN) Certos povos nômades que

vivem no deserto, onde as temperaturas durante

o dia podem chegar a 50°C, usam roupas de lã

branca, para se protegerem do intenso calor da

atmosfera. Essa atitude pode parecer-nos

estranha, pois, no Brasil, usamos a lã para nos

protegermos do frio.

O procedimento dos povos do deserto pode,

contudo, ser explicado pelo fato de que:

a) a lã é naturalmente quente (acima de 50°C) e,

no deserto, ajuda a esfriar os corpos das

pessoas, enquanto o branco é uma “cor fria”,

ajudando a esfriá-los ainda mais.

b) a lã é bom isolante térmico, impedindo que o

calor de fora chegue aos corpos das pessoas, e o

branco absorve bem a luz em todas as cores,

evitando que a luz do sol os aqueça ainda mais.

c) a lã é bom isolante térmico, impedindo que o

calor de fora chegue aos corpos das pessoas, e o

branco reflete bem a luz em todas as cores,

evitando que a luz do sol os aqueça ainda mais.

37

d) a lã é naturalmente quente (embora esteja

abaixo de 50°C) e, no deserto, ajuda a esfriar os

corpos das pessoas, e o branco também é uma

“cor quente”, ajudando a refletir o calor que

vem de fora.

Resolução: A lã é bom isolante térmico, e a cor

branca reflete a radiação incidente amenizando

o calor mediante a redução do fluxo do exterior

para o corpo da pessoa. (Resposta C)

47) (ENEM) A Terra e cercada pelo vácuo

espacial e, assim, ela só perde energia ao

irradiá-la para o espaço.

O aquecimento global que se verifica hoje

decorre de pequeno desequilíbrio energético, de

cerca de 0,3%, entre a energia que a Terra

recebe do Sol e a energia irradiada a cada

segundo, algo em torno de 1 W/m². Isso

significa que a Terra acumula, anualmente,

cerca de 1,6.1022

J.

Considere que a energia necessária para

transformar 1 kg de gelo a 0 oC em água liquida

seja igual a 3,2.105 J. Se toda a energia

acumulada anualmente fosse usada para derreter

o gelo nos polos (a 0 oC), a quantidade de gelo

derretida anualmente, em trilhões de toneladas,

estaria entre

a) 20 e 40.

b) 40 e 60.

c) 60 e 80.

d) 80 e 100.

e) 100 e 120.

Resolução: m = Eac/Em = 1,6.1022

/3,2.105

= 0,5.1017

= 5.1016

kg ou 50 trilhões de

toneladas. (Resposta B)

48) (CEFET – PR) O papel alumínio é muito

utilizado na indústria de alimentos em função de

suas propriedades ópticas. No caso dos iogurtes

e do chocolate, além de ele vedar o alimento do

contato com o ar, ele também:

a) diminui as trocas de calor com o ambiente em

função da reflexão.

b) funciona como um isolante térmico em

função da absorção.

c) absorve melhor a energia térmica.

d) dispersa melhor a energia.

e) refrata melhor a energia.

Resolução: O papel alumínio reflete a radiação

incidente reduzindo as trocas de calor com o

ambiente. (Resposta: A)

49) (UFMG – MG) Atualmente, a energia solar

está sendo muito utilizada em sistemas de

aquecimento de água.

Nesses sistemas, a água circula entre um

reservatório e um coletor de energia solar. Para

o perfeito funcionamento desses sistemas, o

reservatório deve estar em um nível superior ao

do coletor, como mostrado nesta figura:

No coletor, a água circula através de dois canos

horizontais ligados por vários canos verticais. A

água fria sai do reservatório, entra no coletor,

onde é aquecida, e retorna ao reservatório por

convecção.

Nas quatro alternativas seguintes estão

representadas algumas formas de se conectar o

reservatório ao coletor. As setas indicam o

sentido de circulação da água.

Assinale a alternativa em que estão

corretamente representados o sentido da

circulação da água e a forma mais eficiente para

se aquecer toda a água do reservatório.

a) b) c) d)

Resolução: A água sai por baixo do reservatório

entra no coletor pela parte inferior. É aquecida e

retorna para a parte superior do reservatório.

(Resposta D)

50) (ENEM ) Uma garrafa de vidro e uma lata

de alumínio, cada uma contendo 330 mL de

refrigerante, são mantidas em um refrigerador

pelo mesmo longo período de tempo. Ao retirá-

las do refrigerador com as mãos desprotegidas,

tem-se a sensação de que a lata está mais fria

que a garrafa.

É correto afirmar que:

a) a lata está realmente mais fria, pois a

capacidade calorífica da garrafa é maior que a

da lata.

38

b) a lata está de fato menos fria que a garrafa,

pois o vidro possui condutividade menor que o

alumínio.

c) a garrafa e a lata estão à mesma temperatura,

possuem a mesma condutividade térmica, e a

sensação deve-se à diferença nos calores

específicos.

d) a garrafa e a lata estão à mesma temperatura,

e a sensação é devida ao fato de a condutividade

térmica do alumínio ser maior que a do vidro.

e) a garrafa e a lata estão à mesma temperatura,

e a sensação é devida ao fato de a condutividade

térmica do vidro ser maior que a do alumínio.

Resolução: A temperatura é a mesma, porém

inferior a das mãos. Esta cede calor aos

recipientes e, como a condutividade térmica da

lata é maior que a da garrafa, o calor flui de

forma mais rápida das mãos para a lata,

acentuando a sensação térmica de frio.

(Resposta D)

51) (ENEM) O uso mais popular de energia

solar está associado ao fornecimento de água

quente para fins domésticos.

Na figura a seguir, é ilustrado um aquecedor de

água constituído de dois tanques pretos dentro

de uma caixa termicamente isolada e com

cobertura de vidro, os quais absorvem energia

solar.

Nesse sistema de aquecimento:

a) os tanques, por serem de cor preta, são maus

absorvedores de calor e reduzem as perdas de

energia.

b) a cobertura de vidro deixa passar a energia

luminosa e reduz a perda de energia térmica

utilizada para o aquecimento.

c) a água circula devido à variação de energia

luminosa existente entre os pontos X e Y.

d) a camada refletiva tem como função

armazenar energia luminosa.

e) o vidro, por ser bom condutor de calor,

permite que se mantenha constante a

temperatura no interior da caixa.

Resolução: A cobertura de vidro permite a

passagem da energia luminosa, a qual é

absorvida pelos tanques revestidos em preto e

aquecendo a água que se movimenta de um

tanque a outro por convecção. A camada de

vidro ainda impede a saída da energia radiante

pois o vidro, além de ser bom isolante térmico

ainda se comporta como um diatérmico.

(Resposta B)

52) (MACK – SP) Uma das razões que faz a

água, próxima à superfície livre de alguns lagos,

congelar no inverno, em regiões de baixas

temperaturas, é o fato de que ao ser resfriada, no

intervalo aproximado de 4°C a 0°C, ela sofre

um processo de dilatação. Com isso seu volume

____________ e sua densidade____________ .

Desprezando os efeitos da irradiação térmica,

durante esse resfriamento a água do fundo do

lago não consegue atingir a superfície livre, pois

não ocorre mais a ____________ e sua

temperatura diminuirá, devido ao processo

de____________ .

As informações que preenchem corretamente as

lacunas, na ordem de leitura são

respectivamente:

a) aumenta, diminui, convecção térmica e

condução térmica.

b) diminui, aumenta, convecção térmica e

condução térmica.

c) aumenta, diminui, condução térmica e

convecção térmica.

d) diminui, aumenta, condução térmica e

convecção térmica.

e) aumenta, aumenta, condução térmica e

convecção térmica.

Resolução: Durante o resfriamento de 4 ºC a 0 oC o volume da água diminui e em conseqüência

sua densidade aumenta. Assim, a água no fundo,

por estar mais densa, não sobe, cessando a

convecção Assim a temperatura diminuirá

devido ao processo de condução térmica do

calor das camadas mais quentes para as mais

frias. (Resposta B)

53) (UFRN – RN) Em várias cidades do

nordeste do Brasil, ainda existe a tradição de se

acender fogueiras durante o mês de junho, em

comemoração às chamadas “festas juninas”.

Num ato de coragem, algumas pessoas

aproveitam para caminhar com os pés descalços

sobre pedaços de carvão de madeira em brasa,

sem, no entanto, queimar os pés.

Essas pessoas não queimam os pés

principalmente devido à

a) baixa capacidade de absorção de calor pelos

pés das pessoas.

b) alta condutividade térmica da madeira.

c) alta capacidade de absorção de calor pelos

pés das pessoas.

d) baixa condutividade térmica da madeira.

Resolução: A madeira tem baixo coeficiente de

condutividade térmica o que proporciona um

fluxo de calor mais lento dela para os pés da

pessoa. (Resposta D)

39

54) (UERJ – RJ) Duas chaleiras idênticas, que

começam a apitar no momento em que a água

nelas contida entra em ebulição, são colocadas

de duas formas distintas sobre o fogo, como

indica a figura:

Assim, a situação relativa ao tempo de duração

dos apitos das chaleiras e a explicação física do

fenômeno estão descritas na seguinte

alternativa:

a) A chaleira I continuará apitando por mais

tempo, pois a placa metálica está mais quente do

que a água.

b) Ambas as chaleiras deixam de apitar no

mesmo instante, pois as chamas foram apagadas

simultaneamente.

c) Ambas as chaleiras deixam de apitar no

mesmo instante, pois a temperatura da água nas

duas é a mesma.

d) A chaleira II continuará apitando por mais

tempo, pois a capacidade térmica do metal é

menor do que a da água.

Resolução: A chaleira I continua sobre a placa

metálica cuja temperatura é maior que a da água

contida. Assim a chaleira continuará a apitar por

mais tempo. A chaleira II deixa de apitar tão

logo cesse a fonte de calor, pois o processo em

seu interior se torna apenas exotérmico.

(Resposta A)

55) (UFSC – SC) Um grupo de amigos compra

barras de gelo para um churrasco, num dia de

calor. Como as barras chegam com algumas

horas de antecedência, alguém sugere que sejam

envolvidas num grosso cobertor para evitar que

derretam demais. Essa sugestão:

a) é absurda, porque o cobertor vai aquecer o

gelo, derretendo-o ainda mais depressa.

b) é absurda, porque o cobertor facilita a troca

de calor entre o ambiente e o gelo, fazendo com

que ele derreta ainda mais depressa.

c) é inócua, pois o cobertor não fornece nem

absorve calor ao gelo, não alterando a rapidez

com que o gelo derrete.

d) faz sentido, porque o cobertor facilita a troca

de calor entre o ambiente e o gelo, retardando o

seu derretimento.

e) faz sentido, porque o cobertor dificulta a

troca de calor entre o ambiente e o gelo,

retardando o seu derretimento.

Resolução: O cobertor é um isolante térmico

dificultando a troca de calor do meio externo

para o gelo. Assim a proposta é correta.

(Resposta E)

56) (ENEM) Para diminuir as perdas térmicas

de uma geladeira, podem ser tomados alguns

cuidados operacionais:

I. Distribuir os alimentos nas prateleiras

deixando espaços vazios entre eles, para que

ocorra a circulação do ar frio para baixo e do

quente para cima.

II. Manter as paredes do congelador com

camada bem espessa de gelo, para que o

aumento da massa de gelo aumente a troca de

calor no congelador.

III. Limpar o radiador ("grade" na parte de trás)

periodicamente, para que a gordura e a poeira

que nele se depositam não reduzam a

transferência de calor para o ambiente.

Para uma geladeira tradicional é correto indicar,

apenas,

a) a operação I.

b) a operação II.

c) as operações I e II.

d) as operações I e III.

e) as operações II e III.

Resolução:

I(V) Os espaços entre os alimentos depositados

nas grades permitem a livre convecção das

massas de ar existentes dentro da geladeira.

II(F) Na situação descrita, o interior do

congelador apenas permaneceria em 0 oC.

III(V) Com a grade limpa a transferência de

calor se torna grande, efetivando o

funcionamento da geladeira. (Resposta D)

57) (CEFET – PR) As recentes conquistas

espaciais, como a construção da ISS (Estação

Espacial Internacional) e a chegada de sondas

exploratórias em Marte e Saturno, são passos

iniciais para a futura chegada do homem a essas

longínquas fronteiras. No caso do pouso em

Marte, estime a energia transmitida através da

roupa de um astronauta durante um passeio de 3

minutos pela superfície que se encontra a – 60

ºC. A vestimenta espacial apresenta área de 2

m², espessura de 5 mm e é constituída de um

material sintético de coeficiente de

condutibilidade térmica 0,005 J/s.m.K.

(Considere a temperatura corpórea = 37º C)

a) 8,92 kJ.

b) 8,00 kJ.

c) 13,60 kJ.

d) 36,32 KJ.

e) 34,92 kJ.

Resolução: = En/∆t = K.A.∆T/L

En/180 = 0,005.2.97/0,005 Em = 34920 J ou

34,2 J. (Resposta E)

40

58) (UFRN – RN) Numa aula prática de

Termologia, o professor realizou a

demonstração a seguir:

I) colocou massas iguais de água e óleo, à

mesma temperatura, respectivamente, em dois

recipientes de vidro pirex, isolados

termicamente em suas laterais e respectivas

partes superiores;

II) pegou dois termômetros idênticos e colocou

um em cada recipiente;

III) em seguida, colocou esses recipientes sobre

uma chapa quente.

Passado algum tempo, o professor mostrou para

seus alunos que o termômetro do recipiente com

óleo exibia um valor de temperatura maior que

o do recipiente com água, conforme ilustrado na

figura.

Considerando-se que a água e o óleo receberam

a mesma quantidade de calor da chapa quente, é

correto afirmar que a temperatura do óleo era

mais alta porque

a) a condutividade térmica da água é igual à do

óleo.

b) a condutividade térmica da água é maior que

a do óleo.

c) o calor latente da água é igual ao do óleo.

d) o calor específico da água é maior que o do

óleo.

Resolução: A água e o óleo recebem taxas

iguais de calor através do vidro pirex. No

entanto, em virtude de o calor específico do óleo

ser menor que o da água, este se aquece mais

rapidamente, pois cada unidade de massa

necessita receber menos calor para sua

temperatura variar uma unidade, em relação ao

que acontece com a água. (Resposta: D)

59) (UFMG – MG) Em uma experiência

colocam-se gelo e água em um tubo de ensaio,

sendo o gelo mantido no fundo por uma tela de

metal. O tubo de ensaio é aquecido conforme a

figura. Embora a água ferva o gelo não se funde

imediatamente. As afirmações, a seguir,

referem-se a esta situação:

I - Um dos fatores que contribui para que o gelo

não se funda é o de a água quente ser menos

densa que a água fria.

II - Um dos fatores que concorre para a

situação observada é o de o vidro ser um bom

isolante térmico.

III - Um dos fatores que concorre para a

situação observada é o de a água ser bom

isolante térmico.

Responda:

a) se apenas a afirmativa I é verdadeira.

b) se apenas a afirmativa II é verdadeira.

c) se apenas a afirmativa III é verdadeira.

d) se todas as afirmativas são corretas.

e) se todas as afirmativas são falsas.

Resolução: O vidro é um isolante dificultando o

fluxo do calor através dele para a parte inferior

do tubo. A água, aquecida em sua parte superior

e mesmo sendo menos densa que as camadas

inferiores não entra em convecção,

permanecendo nas camadas acima. O fato de

também ser um isolante térmico dificulta a

propagação do calor para as partes inferiores.

Portanto I(V); II(V); III(V). (Resposta D)

60) (CEFET – PR) Um recipiente de vidro

comum contém água. Na primeira situação

(figura I), o fundo do recipiente é colocado em

contato direto com a chama. Na segunda

situação (figura II), uma tela metálica é

interposta entre a chama e o fundo do recipiente.

Com relação a essas situações, analise as

alternativas a seguir e assinale a correta.

a) Na situação ( I ), a água irá ferver e na

situação ( II ) não.

b) Na situação ( II ), a água não chega a

aquecer, pois a tela funciona como um isolante

térmico, devido ao ar em repouso retido entre as

malhas.

41

c) Sendo boa condutora, a tela transmite

rapidamente o calor para toda sua parte

superior, causando um aquecimento uniforme

no recipiente sem o contato direto com a chama.

d) Na situação ( II ), a chama ultrapassa a tela

nos seus espaços vazios e o fundo do recipiente

se rompe, pois as regiões diretamente aquecidas

se dilatam mais que as regiões vizinhas.

e) Na situação ( II ), a chama, ao

ultrapassar a tela, permite que o tempo que a

água leva para ferver seja inferior ao da

situação ( I ).

Resolução: O aquecimento integral da tela

permite a distribuição uniforme do calor na base

do vidro sem que ocorra contato com a chama.

(Resposta C)

61) (UFRN – RN) O calor e suas formas de

propagação se manifestam em diversas

situações tanto na Natureza quanto nas

atividades humanas. Assim, fenômenos

aparentemente muito diferentes são

semelhantes, quando analisados mais

detidamente.

Veja-se, por exemplo: A energia do Sol que

aquece nosso Planeta e a energia emitida pelo

magnetron do forno de microondas, que aquece

os alimentos colocados em seu interior, são

fenômenos que envolvem propagação de calor.

Pode-se afirmar que as formas de propagação de

energia entre o Sol e a Terra e entre o

magnetron e os alimentos são, respectivamente

a) convecção e condução.

b) convecção e convecção.

c) condução e radiação.

d) radiação e radiação.

Resolução: Nos dois casos o calor é transmitido

por radiação. A diferença, apenas é que em se

tratando do Sol o meio é o vácuo e em se

tratando do forno, o meio é o ar. (Resposta D)

62) (MACK – SP) Numa noite de inverno, o

dormitório de Serginho apresentava uma

temperatura ambiente de 10 ºC. Para não sentir

frio durante a madrugada, ele esticou sobre a

cama três cobertores de lã bem espessos e

aguardou alguns minutos. Em seguida, deitou-se

e percebeu que a cama continuava muito fria.

Após certo tempo na cama, bem coberto, sentiu

que o “frio passou” e que a cama estava quente.

Tal fato explica-se, pois:

a) o frio não existe e a sensação de Serginho era

apenas psicológica.

b) os cobertores não são aquecedores, mas

isolantes térmicos. Depois de Serginho deitar-

se, seu corpo aqueceu a cama.

c) a cama provavelmente não tinha lençóis de lã

e, então, o calor produzido pelos cobertores foi

perdido para o ambiente. Quando Serginho se

deitou, interrompeu esse processo.

d) os cobertores de lã provavelmente eram de

cor clara e, por isso, demoraram muito para

aquecer a cama. Após Serginho ter-se deitado,

foi necessário mais algum tempo para que a

cama ficasse quente.

e) a lã utilizada para a confecção dos cobertores

é um aquecedor natural muito lento e a

temperatura de Serginho, de aproximadamente

37 ºC, não era suficiente para aquecer a cama.

Resolução: Os cobertores são isolantes

térmicos. O calor emitido pelo corpo do

Serginho aqueceu a cama, pois não flui em taxa

acentuada para o meio externo. (Resposta B)

63) (ACAFE – SC) O forno de microondas é um

eletrodoméstico de grande utilidade no

descongelamento de alimentos, no preparo

rápido das refeições e no aquecimento rápido de

comida pronta. O forno de microondas, além

disso, é mais econômico que o elétrico, quanto

ao consumo de energia. Nesse forno, grande

parte da energia das ondas eletromagnéticas é

absorvida pelas moléculas de água dos

alimentos, penetrando neles e os cozinhado por

inteiro. A radiação na região das microondas

não é, no entanto, absorvida por alguns vidros e

cerâmicas.

Uma pessoa colocou um prato de cerâmica

contendo o alimento, ambos à temperatura

ambiente, no formo de microondas e o ligou. Ao

retirá-lo do forno, imediatamente após o instante

em que ele desliga, pode ocorrer que:

a) ambos, prato e alimento, estejam na mesma

temperatura devido à condução do calor.

b) o prato esteja mais quente que o alimento

devido à radiação.

c) ambos, prato e alimento, estejam na mesma

temperatura devido à radiação.

d) o prato esteja mais quente que o alimento

devido à condução do calor.

e) o prato esteja menos quente que o alimento

devido à radiação.

Resolução: A radiação é absorvida pelo

alimento aumentando sua temperatura e

refletida pela cerâmica, deficultando a elevação

da sua temperatura. (Resposta E

64) (CEFET – PR) Com relação ao fenômeno

da transmissão do calor, são feitas as afirmações

a seguir:

I) Os iglus, embora feitos de neve compactada,

possibilitam aos esquimós neles residirem

porque este material não é um bom condutor de

calor.

II) Sentimos frio muito rapidamente ao nos

aproximarmos de uma parede gélida. É possível

afirmar que não é o frio que se propaga até nós,

mas que nós irradiamos mais energia do que

recebemos dela.

42

III) Se um ventilador de teto, desligado, estiver

colocado sobre uma lâmpada incandescente

acesa, ele pode girar lentamente devido à

convecção do ar aquecido.

Podemos afirmar que:

a) somente I e II são corretas.

b) todas são corretas. (CORRETA)

c) somente II e III são corretas.

d) somente I e III são corretas.

e) nenhuma é correta.

Resolução:

I(V) Além do que sua forma impede a saída por

convecção do ar aquecido.

II(V) A sensação de frio está relacionada com a

perda rápida de calor.

III(V) As correntes de convecção podem

movimentar as pás do ventilador. (Resposta B)

65) (PUC – RS) Numa cozinha, é fácil constatar

que a temperatura é mais elevada próximo ao

teto do que próximo ao chão, quando há fogo no

fogão. Isso é devido ao fato de o

a) calor não se propagar para baixo.

b) calor não se propagar horizontalmente.

c) ar quente subir, por ser menos denso do que o

ar frio.

d) ar quente subir, por ser mais denso do que o

ar frio.

e) ar frio descer, por ser menos denso do que o

ar quente.

Resolução: As correntes de convecção são

formadas com o ar quente (menor densidade) se

deslocar para cima e o ar menos quente (menor

densidade) se deslocar para baixo. (Resposta C)

66) (CEFET – PR) Sobre os processos de

propagação do calor, é incorreto afirmar que:

a) a inversão térmica é um fenômeno que ocorre

no inverno e caracteriza-se pela ausência de

convecção entre o ar puro das camadas

superiores a o ar frio poluído, resultante da

contaminação por veículos e indústrias.

b) a estufa de plantas é feita de vidro, ou outro

material transparente, para que a energia

radiante do Sol penetre, mas não permita a

passagem das ondas puramente térmicas,

emitidas pelos objetos no seu interior.

c) na construção de garrafas térmicas, utiliza-se

uma dupla parede de vidro por ser este um

material bom condutor de calor.

d) a absorção da radiação térmica está

intrinsecamente relacionada com a cor e o grau

de polimento do corpo que a recebe.

e) no inverno os pássaros eriçam suas penas

para que haja acúmulo de ar em seu interior, que

atua como um isolante térmico.

Resolução: Nas garrafas térmicas as paredes

são de vidro pois este material é um bom

isolante térmico ou ainda um mau condutor de

calor. (Resposta: C)

67) (MACK – SP) Encostado em um dos lados

de uma placa de cobre, temos vapor de água a

100 ºC e, encostado do outro lado desta mesma

placa, temos gelo a 0 ºC. Considere que o calor

somente pode se propagar do vapor para o gelo

e que o sistema se encontra ao nível do mar.

Sabendo que em 1,0 minuto são fundidos 90 g

de gelo e que não há variação de temperatura, o

fluxo de calor que atravessa a placa é de

a) 80 cal/s

b) 100 cal/s

c) 120 cal/s

d) 140 cal/s

e) 160 cal/s

Dados:

Calor latente de vaporização da água = 540

cal/g

Calor latente de fusão da água = 80 cal/g

Resolução: = En/∆t = m.LF/∆t = 90.80/60 =

120 cal/s. (Resposta C)

68) (UFMS – MS) O calor é uma forma de

energia que pode se propagar através dos

processos de condução, convecção e irradiação.

A respeito desses processos, é correto afirmar

que

a) a radiação infravermelha, oriunda do Sol,

necessita de um meio material para se propagar

até a pele de uma pessoa exposta ao sol.

b) o processo de condução de calor através de

uma janela de vidro de uma residência depende

somente da diferença de temperatura entre o

interior e o lado externo da residência.

c) nos três processos de propagação de calor, há

também a propagação da matéria.

d) o principal responsável pelo aquecimento da

Terra é a irradiação térmica.

e) da mesma forma que no processo de

irradiação térmica, na convecção térmica não há

deslocamento de massa.

Resolução:

a(F). As ondas eletromagnéticas podem se

propagar no vácuo.

c(F) Também depende da espessura do vidro.

c(F) A irradiação também pode ocorrer no

vácuo.

d(V) A quantidade de calor írradiada pela Terra

é menor que a quantidade nela incidente

proporcionando um aquecimento global.

e(F) A convecção térmica se dá através da

movimentação das massas a diferentes

temperaturas. (Resposta D)

69) (UFRN – RN) Num dia muito ensolarado,

Luciana se questionou sobre como o

aquecimento interno de um carro é alterado

quando seus vidros são revestidos por películas.

43

Ela sabe que, ao estacionar o carro sob o sol,

com todas as janelas fechadas, por alguns

minutos, o aquecimento no interior do veículo

se dá predominantemente por irradiação e

condução. Nessa mesma situação, se os vidros

do carro estiverem revestidos por películas, a

intensidade da radiação em seu interior será

menor, causando um menor aquecimento. Para

que o aquecimento interno do automóvel seja

mínimo, deve-se usar uma película que faça a

radiação solar incidente sobre ele ser

maximamente

a) difratada.

b) refratada.

c) absorvida.

d) refletida.

Resolução: Como a energia solar chega por

irradiação térmica, é conveniente uma boa taxa

sofrer reflexão na película. (Resposta D)

70) (PUCC – SP) Uma estufa está à temperatura

de 40 ºC, quando no exterior a temperatura é de

0 ºC. As paredes da estufa são constituídas de

placas de vidro de espessura de 2 mm e área de

2500 cm². Qual o calor transmitido em cada

segundo através da placa de vidro?

Dado: coeficiente de condutibilidade térmica =

0,0015 cal./s.cm.ºC.

a) 570 cal/s

b) 487 cal/s

c) 847 cal/s

d) 750 cal/s

Resolução: = En/∆t = K.A.∆T/L =

0,0015.2500.40/0,2 = 750 cal/s (Resposta D)

71) (ENEM) O diagrama representa, de forma

esquemática e simplificada, a distribuição da

energia proveniente do Sol sobre a atmosfera e a

superfície terrestre. Na área delimitada pela

linha tracejada, são destacados alguns processos

envolvidos no fluxo de energia na atmosfera.

Com base no diagrama, conclui-se que

a) a maior parte da radiação incidente sobre o

planeta fica retida na atmosfera.

b) a quantidade de energia refletida pelo ar,

pelas nuvens e pelo solo é superior à absorvida

pela superfície.

c) a atmosfera absorve 70% da radiação solar

incidente sobre a Terra.

d) mais da metade da radiação solar que é

absorvida diretamente pelo solo é devolvida

para a atmosfera.

e) a quantidade de radiação emitida para o

espaço pela atmosfera é menor que a irradiada

para o espaço pela superfície.

Resolução: A radiação absorvida diretamente

pelo o solo corresponde a 60%. Desta, 44% é

devolvida para a atmosfera mediante três

processos. (Resposta D)

72) (ENEM) O Sol representa uma fonte limpa

e inesgotável de energia para o nosso planeta.

Essa energia pode ser captada por aquecedores

solares, armazenada e convertida posteriormente

em trabalho útil. Considere determinada região

cuja insolação — potência solar incidente na

superfície da Terra — seja de 800 watts/m².

Uma usina termossolar utiliza concentradores

solares parabólicos que chegam a dezenas de

quilômetros de extensão. Nesses coletores

solares parabólicos, a luz refletida pela

superfície parabólica espelhada é focalizada em

um receptor em forma de cano e aquece o óleo

contido em seu interior a 400 °C. O calor desse

óleo é transferido para a água, vaporizando-a

em uma caldeira. O vapor em alta pressão

movimenta uma turbina acoplada a um gerador

de energia elétrica.

Considerando que a distância entre a borda

inferior e a borda superior da superfície refletora

tenha 6 m de largura e que focaliza no receptor

os 800 watts/m² de radiação provenientes do

Sol, e que o calor específico da água é 1cal.g-

1ºC

-1 = 4.200 J kg

-1ºC

-1, então o comprimento

linear do refletor parabólico necessário para

elevar a temperatura de 1 m³ (equivalente a 1 t)

de água de 20 °C para 100 °C, em uma hora,

estará entre

44

a) 15 m e 21 m.

b) 22 m e 30 m.

c) 105 m e 125 m.

d) 680 m e 710 m.

e) 6.700 m e 7.150 m.

Resolução: Q = mc∆ = 1000.4200.80 =

336.106

J I = Pot/A 800 =

(336.106/3600)/(6.L) L = 19,44m (Resposta

A)

73) (MACK – SP) Têm-se três cilindros de

mesmas secções transversais de cobre, latão e

aço, cujos comprimentos são, respectivamente,

de 46 cm, 13 cm e 12 cm. Soldam-se os

cilindros formando o perfil em Y, indicado na

figura.

O extremo livre do cilindro de cobre é mantido

a 100 ºC, e os cilindros de latão e aço a 0 ºC.

Supor que as superfícies laterais dos cilindros

estão isoladas termicamente. As

condutibilidades térmicas do cobre, latão e aço

valem, respectivamente, 0,92, 0,26 e 0,12

expressas em cal/cm.s.ºC. No estado

estacionário, a temperatura na junção é igual a :

a) 100 ºC

b) 80 ºC

c) 67 ºC

d) 50 ºC

e) 40 ºC

Resolução: O sentido do fluxo é do corpo de

maior temperatura para os de menores. Assim

COBRE = LATÃO + AÇO KC.AC.(100 – )/LC

= KL.AL.( – 0)/LL + KA.AA.( – 0)/LA

0,92(100 – )/46 = 0,26. ( – 0)/13 + 0,12( –

0)/12 (92 – 0,92)/46 = 0,26/13 + 0,1212

2 – 0,02 = 0,02 + 0,01 = 40 ºC.

(Resposta E)

Gases perfeitos

74) (URCA) Em uma partida de futebol, a bola

utilizada, tem diâmetro interno igual a 20 cm.

Quando cheia, a bola apresenta, em seu interior,

ar sob pressão de 1,0 atm e temperatura de 27

ºC. Considere π = 3, constante universal dos

gases ideais R = 0,080 atm.L/mol.K e a

molécula grama do ar igual a 30 g/mol.

A massa de ar no interior da bola cheia, em

gramas, corresponde a:

a) 2,5

b) 5,0

c) 7,5

d) 10

e) 12

Resolução: Volume da bola em m³: V = (4/3)

πr³ = (4/3).3.0,1³ = 0,004 m³ ou 4 L. O número

de mols será: pV = nRT 1.4 = n.0,08.300

4 = n.24 n = 1/6 mols. A massa de ar será:

n = n/M 1/6 = m/30 m = 5g.

(Resposta B)

75) (CEFET – PR) O reservatório representado

contém 0,249 m³ de um gás perfeito a 27 ºC e se

comunica com um manômetro de tubo aberto

que contém mercúrio. Sabe-se que a pressão

atmosférica no local vale 680 mmHg, que a

constante dos gases vale 8,30 J.mol–1

.K–1

e que

1.105 Pa corresponde a 760 mmHg.

Desconsiderando o volume do manômetro é

possível afirmar que existe no reservatório:

a) 5 mols de gás.

b) 36 mols de gás.

c) 8 mols de gás.

d) 3 x 104 mols de gás.

e) 22 mols de gás.

Resolução: pressão exercida pelo gás: p = 680 –

300 = 380 mmHg que em pascals corresponde a

380.1. 105/760 = 5.10

4 Pa. Assim: pV = nRT

5.104 . 0,249 = n.8,3.300 n = 5 mols.

(Resposta A)

45

76) (EFGO – GO) Um bujão de gás de cozinha

contém 13 kg de gás liquefeito, à alta pressão.

Um mol desse gás tem massa de,

aproximadamente, 52 g. Se todo o conteúdo do

bujão fosse utilizado para encher um balão, à

pressão atmosférica e à temperatura de 300K, o

volume final do balão seria aproximadamente

de: Constante dos gases R = 8,3 J/(mol.K) ou

0,082 atm.L /(mol.K)

patmosférica = 1 atm = 1.105 Pa

(1Pa = 1 N/m²) 1 m³ = 1000 L

a) 13 m³

b) 6,2 m³

c) 3,1 m³

d) 0,98 m³

e) 0,27 m³

Resolução: pV = nRT 1.105.V =

(13/0,052).8,3.300 V = 6,25 m³ 6,2 m³.

(Resposta B)

77) (CEFET – PR) Um recipiente cilíndrico

contém certa massa de gás hidrogênio sob

pressão de 2,0.107 Pa.

O volume interno do cilindro é 3,0.10–2

m³ e a

temperatura é de 27oC. Sendo a massa molar do

hidrogênio 2 g, a massa desse gás contido no

cilindro é, em kg, igual a:

(A constante de Clapeyron é R = 8,3 J/mol.K)

a) 0,48

b) 1,23.

c) 2,31.

d) 4,83.

e) 5,33.

Resolução: pV = nRT 2.107. 3.10

–2 =

(m/0,002).8,3.300 6.105 = m. 1245000

m = 0,481 kg (Resposta A)

78) (MAC – SP) Um mol de gás ideal encontra-

se inicialmente (estado A) nas C.N.T.P..

Em seguida esse gás sofre duas transformações

sucessivas, conforme mostra o diagrama P x V.

O volume ocupado pelo gás no estado C é:

Dado: R = 0,082 (atm.litro)/(mol.K)

a) 11,2 litros.

b) 16,8 litros.

c) 22,4 litros.

d) 33,6 litros.

e) 44,8 litros.

Resolução: O volume VA é: pV = nRT 1.VA

= 1.0,082.273 VA = 22,386 L. Em C,

levando em conta apenas os estados inicial e

final, temos:

pAVA/TA = pCVC/TC 1.22,386/273 = 0,75.

VC/409,5 VC = 44,772 n44,8 L

(Resposta E)

79) (UFRN – RN) O departamento de Física da

UFRN possui um laboratório de pesquisa em

criogenia, ciência que estuda a produção e

manutenção de temperaturas muito baixas,

contribuindo para o entendimento das

propriedades físicas e químicas de sistemas

nessas temperaturas pouco comuns.

Nesse laboratório, uma máquina retira o gás

nitrogênio do ar e o liquefaz a uma temperatura

de 77,0 kelvin (K), que corresponde a -196

graus Celsius (ºC). Nessa temperatura o

nitrogênio é usado cotidianamente pelos

departamentos de Física, Química e Biologia da

UFRN, como também por pecuaristas no

congelamento de sêmen para reprodução

animal.

O nitrogênio líquido, em virtude de suas

características, necessita ser manuseado

adequadamente, pois pessoas não habilitadas

poderão sofrer acidentes e serem vítimas de

explosões. Imagine uma pessoa desavisada

transportando, num dia quente de verão, uma

porção de nitrogênio líquido numa garrafa

plástica fechada.

Como o nitrogênio líquido tende a entrar em

equilíbrio térmico com o ambiente, mudará de

estado físico, transformando-se em um gás. A

tendência desse gás é se expandir, podendo

provocar uma explosão.

Admita que

I) o nitrogênio rapidamente se transforma em

gás ideal, ou seja, obedece à equação PV = nRT.

Em que R é a constante universal dos gases e P,

V, T, n são, respectivamente: a pressão, o

volume, a temperatura e o número de moles do

gás;

II) a pressão interna e a temperatura iniciais

desse gás são, respectivamente, 2,00.105 pascal

(Pa) e 78,0 K;

III) a garrafa utilizada pode suportar uma

pressão máxima de 4,00.105 Pa e o volume

dessa garrafa não varia até que a explosão

ocorra.

Diante dessas considerações, é correto dizer que

a temperatura limite (do gás nitrogênio) que a

garrafa suporta sem explodir é

a) 273 K. b) 156 K. c) 234 K. d) 128 K.

Resolução: Aplicando a lei geral das

transformações, temos: p1V1/T1 = p2V2/T2

2.105V/78 = 4.105V/ T2 T2 = 156 K.

(Resposta B)

46

80) (UFMS – MS) Quando um gás ideal, em

uma transformação isovolumétrica, tem sua

temperatura variando de 20 ºC a 40 ºC, é correto

afirmar que

a) a pressão do gás dobra.

b) o volume do gás aumenta .

c) sua pressão aumenta aproximadamente em

6,8% .

d) a temperatura do gás aumenta em 200% .

e) a pressão do gás não se altera.

Resolução: p1/T1 = p2/T2 p1/293 = p2/313

p2.293 = p1.313 p2 = 1,068p1 ou seja, a

pressão aumentou 0,068 ou 6,8% em relação a

inicial. (Resposta: C)

81) (UERJ – RJ) O vapor contido numa panela

de pressão, inicialmente à temperatura T0 e à

pressão p0 ambientes, é aquecido até que a

pressão aumente em cerca de 20% de seu valor

inicial.

Desprezando-se a pequena variação do volume

da panela, a razão entre a temperatura final T e

inicial T0 do vapor é:

a) 0,8

b) 1,2

c) 1,8

d) 2,0

Resolução: p0/T0 = p/T p0/T0 = 1.2p0/T

T/T0 = 1.2. (Resposta: B)

82) (UFRPE – PE) Um gás ideal confinado num

recipiente fechado possui, inicialmente, volume

de 2 m³ e está sob uma pressão de 105 N/m², a

uma temperatura de 200 K. Tal gás sofre uma

transformação isobárica, ao final da qual a sua

temperatura é de 100 K. O volume final do gás

após essa transformação é igual a:

a) 1 m³

b) 2 m³

c) 3 m³

d) 4 m³

e) 5 m³

Resolução: p1V1/T1 = p2V2/T2 105.2/200 =

105. V2/100 1/100 = V2/100 V2 =1 m³.

(Resposta A)

83) (IFCE – CE) Um gás monoatômico, em

condições de baixa pressão e alta temperatura,

apresenta uma temperatura Ti, uma pressão Pi e

está encerrado em um volume Vi. Este gás sofre

uma transformação isobárica onde sua

temperatura final vale Tf. Seu volume Vf, ao

final do processo, é dado por

a) Vf = Vi. Ti/ Tf b) Vf = Vf . Pi / Ti

c) Vf = Vi . Tf / Ti d) Vf = Vi

e) Vf = Vi . Pf / Ti

Resolução: piVi/Ti = pfVf/Tf pVi/Ti = pVf/Tf

Vf = ViTf /Ti. (Resposta: C)

84) (UECE – CE) Uma dada massa de gás ideal

sofreu evolução termodinâmica que a levou de

um estado inicial de equilíbrio P situado no

plano pressão x volume, para um estado final de

equilíbrio Q, conforme a figura. Se no estado

inicial P a temperatura era 100 K, no estado

final Q a temperatura é:

a) 200 K

b) 350 K

c) 400 K

d) 700 K

Resolução: pPVP/TP = pQVQ/TQ 2.2/100 =

7.4/TQ TQ = 700 K. (Resposta D)

85) (UFPE – PE) Qual dos gráficos a seguir

melhor representa o que acontece com a pressão

no interior de um recipiente contendo um gás

ideal, a volume constante, quando a temperatura

aumenta?

Resolução: Sob mantendo o volume constante,

a pressão torna-se diretamente proporcional à

temperatura absoluta. (p/T = constante).

(Resposta D)

86) (CEFET – PR) Um gás, contido em um

cilindro, à pressão atmosférica, ocupa apenas a

metade de seu volume à temperatura ambiente.

O cilindro contém um pistão, de massa

desprezível, que pode mover-se sem atrito. Esse

gás é aquecido, fazendo com que o pistão seja

empurrado, atingindo o volume máximo

permitido. Observa-se que a temperatura

absoluta do gás é aumentada em 3 vezes do seu

valor inicial.

47

Na situação final, a pressão do gás no cilindro

deverá ser:

a) 1/3 da pressão atmosférica.

b) igual à pressão atmosférica.

c) 3 vezes a pressão atmosférica.

d) 1,5 vezes a pressão atmosférica.

e) 4 vezes a pressão atmosférica.

Resolução: piVi/Ti = pfVf/Tf patm.(V/2)/ Ti =

pf.V/3T pf = 1,5 patm (Resposta D)

87) (FUVEST – SP) Um motorista pára em um

posto e pede ao frentista para regular a pressão

dos pneus de seu carro em 25 "libras"

(abreviação da unidade "libra força por

polegada quadrada" ou "psi"). Essa unidade

corresponde à pressão exercida por uma força

igual ao peso da massa de 1 libra, distribuída

sobre uma área de 1 polegada quadrada. Uma

libra corresponde a 0,5 kg e 1 polegada a 25.10-3

m, aproximadamente. Como 1 atm corresponde

a cerca de 1.105 Pa no SI e 1 Pa = 1 N/m²,

aquelas 25 "libras" pedidas pelo motorista

equivalem aproximadamente a:

a) 2 atm

b) 1 atm

c) 0,5 atm

d) 0,2 atm

e) 0,01 atm

Resolução: 25 libras ou 25 psi = 25. 1 lbf/pol² =

25. 0,5.10N/(25.10–3

m)² = 2.105 N/m² ou 2 atm.

(Resposta A)

88) (MACK – SP) Um mol de gás ideal,

inicialmente num estado A, ocupa o volume de

5,6 litros. Após sofrer uma transformação

isotérmica, é levado ao estado B. Sabendo que

em B o gás está nas CNTP (condições normais

de temperatura e pressão), podemos afirmar que

em A:

a) a pressão é desconhecida e não pode ser

determinada com os dados disponíveis.

b) a pressão é 1,0 atmosfera.

c) a pressão é 2,0 atmosferas.

d) a pressão é 4,0 atmosferas.

e) a pressão é 5,6 atmosferas.

Resolução: : piVi/Ti = pfVf/Tf pA 5,6/TA =

pA .VB/TB pA 5,6 = pA .VB

Os dados são insuficientes para se determinar a

pressão final exercida pelo gás. (Resposta A)

89) (UERJ – RJ) Para um mergulhador, cada 5

m de profundidade atingida corresponde a um

acréscimo de 0,5 atm na pressão exercida sobre

ele. Admita que esse mergulhador não consiga

respirar quando sua caixa torácica está

submetida a uma pressão acima de 1,02 atm.

Para respirar ar atmosférico por um tubo, a

profundidade máxima, em centímetros, que

pode ser atingida pela caixa torácica desse

mergulhador é igual a:

a) 40

b) 30

c) 20

d) 10

Resolução: 0,5 atm 5 m então 0,02 atm

0,2 m ou 20 cm. (Resposta C)

90) (ITA – SP) Uma bolha de ar de volume 20,0

mm3, aderente à parede de um tanque de água a

70 cm de profundidade, solta-se e começa a

subir. Supondo que a tensão superficial da bolha

é desprezível e que a pressão atmosférica é de

1.105 Pa, logo que alcança a superfície seu

volume é aproximadamente:

a) 19,2mm3

b) 20,1mm3

c) 20,4mm3

d) 21,4mm3

e) 34,1mm3.

Resolução: A 70 cm ou 0,7 m a pressão

absoluta será 1,07 atm. Na superfície é 1 atm.

Assim: piVi/Ti = pfVf/Tf 1,07.20/T = 1.

Vf/T Vf = 21,4 mm³. (Resposta D)

91) (UFF – RJ) A pressão do ar dentro dos

pneus é recomendada pelo fabricante para a

situação em que a borracha está fria. Quando o

carro é posto em movimento, os pneus aquecem,

a pressão interna varia e o volume do pneu tem

alteração desprezível.

Considere o ar comprimido no pneu como um

gás ideal e sua pressão a 17 ºC igual a 1,7.105

N/m2.

Depois de rodar por uma hora, a temperatura do

pneu chega a 37 ºC e a pressão do ar atinge o

valor aproximado de:

a) 7,8.104 N/m

2

b) 1,7.105 N/m

2

c) 1,8.105 N/m

2

d) 3,4.105 N/m

2

e) 3,7.105 N/m

2

48

Resolução: piVi/Ti = pfVf/Tf 1,7.105.V/290

= pfV/310 pf = 1,81.105 N/m².

(Resposta C)

92) (UFC – CE) Um gás ideal sofre o

processo cíclico mostrado no diagrama P x

T, conforme figura abaixo. O ciclo é composto

pelos processos termodinâmicos a b, b c

e c a .

Assinale entre as alternativas abaixo aquela que

contém o diagrama P x V equivalente ao ciclo P

x T.

Resolução: O gráfico dado mostra AB

(aquecimento isométrico – volume constante).

BC (compressão isobárica – pressão constante)

CA (expansão isotérmica – temperatura

constante). (Resposta A)

93) (FUVEST – SP) Um equipamento possui

um sistema formado por um pistão, com massa

de 10 kg, que se movimenta, sem atrito, em um

cilindro de secção transversal S = 0,01 m².

Operando em uma região onde a pressão

atmosférica é de 10,0.104 Pa (1 Pa = 1 N/m2), o

ar aprisionado no interior do cilindro mantém o

pistão a uma altura H = 18 cm. Quando esse

sistema é levado a operar em uma região onde a

pressão atmosférica é de 8,0.104 Pa, mantendo-

se a mesma temperatura, a nova altura H no

interior do cilindro passa a ser

aproximadamente de

a) 5,5 cm

b) 14,7 cm

c) 20 cm

d) 22 cm

e) 36 cm

Resolução: piVi/Ti = pfVf/Tf (10.104 +

10.10/0,01). S.18/T = (8.104 + 10.10/0,01). S .

H’/T H’ = 14,7 cm. (Resposta: B)

94) (UFMG – MG) Um cilindro tem como

tampa um êmbolo, que pode se mover

livremente.

Um gás, contido nesse cilindro, está sendo

aquecido, como representado nesta figura:

Assinale a alternativa cujo diagrama melhor

representa a pressão em função da temperatura

nessa situação

Resolução: Supondo o aquecimento feito de

forma lenta, ocorrerá uma transformação

isobárica, ou seja, sob pressão constante.

(Resposta A)

49

95) (UFMA – MA – Modificado) Um cilindro

contém uma certa massa m0 de um gás a T0 = 7

ºC (280 K) e pressão p0. Ele possui uma válvula

de segurança que impede a pressão interna de

alcançar valores superiores a p0. Se essa pressão

ultrapassar p0, parte do gás é liberada para o

ambiente. Ao ser aquecido até T = 77 ºC (350

K), a válvula do cilindro libera parte do gás,

mantendo a pressão interna no valor p0. No final

do aquecimento, a massa de gás que permanece

no cilindro é, aproximadamente, de

a) 1,0 m0

b) 0,8 m0

c) 0,7 m0

d) 0,5 m0

e) 0,1 m0

Resolução: Aplicando a equação de Clapeyron

para os dois estados, temos:

Estado inicial: pV = nRT p0. V = n0.R.280

Estado final: pV = nRT p0. V = n.R 350

Dividindo um estado pelo outro, fica:

1 = n0.R.280/n.R.350 n = 280/350n0 =

0,8n ou 80%n0 o que significa que o sistema

perdeu 20% da sua massa inicial, permanecendo

no cilindro 80% n0.ou 0,8 n0. (Resposta B)

96) (UERJ – RJ) Um gás ideal sofre uma

transformação cíclica A→B→C→A , em que

A→B é uma transformação isotérmica, B→C,

isobárica e C→A , isovolumétrica.

Os gráficos da temperatura em função do

volume (T x V) e da pressão em função do

volume (P x V), para as transformações A →B e

B→C são, respectivamente:

Resolução: A B: Isotérmica: Temperatura

constante. Volume inversamente proporcional à

pressão (Diagramas A e C). B C: Isobárica:

Pressão constante. Volume diretamente

proporcional a temperatura (Diagramas A e B).

(Resposta A)

97) (UNESP – SP) Em uma cidade brasileira, no

horário mais quente do dia, um motorista

calibrou os pneus de seu carro a uma pressão de

30 lb/in² (libras por polegada quadrada ou psi),

usando gás nitrogênio à temperatura ambiente.

Contudo, a chegada de uma frente fria fez com

que a temperatura ambiente variasse de 27 ºC

para 7 ºC, ao final do dia. Considerando as

características do nitrogênio como as de um gás

ideal e que os pneus permanecem em equilíbrio

térmico com o ambiente, a pressão nos pneus ao

final do dia, devido à variação de temperatura,

foi de aproximadamente

a) 7 lb/in2.

b) 14 lb/in2.

c) 28 lb/in2.

d) 30 lb/in2.

e) 32 lb/in2

Resolução: piVi/Ti = pfVf/Tf 30.V/300 =

pfV/280 pf = 28lb/in². (Resposta C)

98) (ITA – SP) Um tubo capilar fechado em

uma extremidade contém uma quantidade de ar

aprisionada por um pequeno volume de água. A

7,0 °C e à pressão atmosférica (76,0cm Hg) o

comprimento do trecho com ar aprisionado é de

15,0cm. Determine o comprimento do trecho

com ar aprisionado a 17,0 °C. Se necessário,

empregue os seguintes valores da pressão de

vapor da água: 0, 75cm Hg a 7,0 °C e 1,42cm

Hg a 17,0 °C.

Resolução: piVi/Ti = pfVf/Tf 76,75.S.15/280

= 77,42. S.x/ 290 x = 15,4 cm.

(Resposta: 15,4 cm)

99) (UERJ – RJ) As mudanças de pressão que o

ar atmosférico sofre, ao entrar nos pulmões ou

ao sair deles, podem ser consideradas como uma

transformação isotérmica. Ao inspirar, uma

pessoa sofre uma diminuição em sua pressão

intrapulmonar de 0,75%, no máximo.

Considere 0,60 L de ar à pressão atmosférica de

740 mmHg. A variação máxima de volume, em

litros, sofrida por essa quantidade de ar ao ser

inspirado é aproximadamente de:

a) 4,5.0

b) 4,5.10−1

c) 4,5.10−2

d) 4,5.10−3

50

Resolução: piVi/Ti = pfVf/Tf 740.0,60/T =

99,25%.740V/T V = 0,6045 L o que

significa uma variação de volume de 0,0045 L

ou 4,5.10–3

L. (Resposta D)

100) (UNESP – SP) Uma bexiga, confeccionada

com látex altamente flexível, é utilizada para

vedar o bocal de um recipiente contendo

nitrogênio líquido. Este conjunto é colocado

sobre o prato de uma balança de precisão,

conforme ilustrado na figura. A indicação da

balança é registrada durante o período de tempo

em que a bexiga se expande como conseqüência

da evaporação controlada do nitrogênio líquido.

O pesquisador responsável pela experiência

concluiu que a indicação L da balança (com

escala em gramas), em função do tempo, em

segundos, poderia ser representada pela função

L = 318 – 3t/7.

Considerando que no instante t = 0 a bexiga está

completamente murcha, pode-se dizer que a

massa de ar deslocada em um intervalo de

tempo de 28s foi de

a) 10 g.

b) 12 g.

c) 16 g.

d) 20 g.

e) 24 g.

Resolução: Para t0 = 0 temos L0 = 318g. Para t

= 28 temos L = 318 – 3.28/7 = 306. Portanto a

massa de ar deslocada foi de ∆L = 318 – 306 =

12 g. (Resposta B)

Máquinas térmicas

101) (ACAFE – SC - Modificado) Um

recipiente contém uma mistura constituída por 1

mol de gás hidrogênio e 1 mol de gás oxigênio

na temperatura ambiente. Todas as moléculas da

mistura estão se deslocando com uma enorme

variedade de velocidades.

Em relação aos valores médios, as moléculas do

oxigênio apresentam ________ energia cinética

média e ________ velocidade que as moléculas

do hidrogênio.

A alternativa que completa as lacunas acima,

em seqüência, é:

– igual

– igual

– menor

– maior

e) maior – igual

Resolução: A energia cinética das partículas

compõe a energia interna do sistema a qual é

função do número de mols e da temperatura

absoluta. Assim a energia cinética é a mesma.

Quanto às velocidades temos que é função da

temperatura e da molécula grama. A velocidade

quadrática é inversamente proporcional à

molécula grama, portanto é menor para o gás

oxigênio. Deste modo ficam: igual – menor.

(Resposta C)

102) (UNESP – SP) Uma bexiga vazia tem

volume desprezível; cheia, o seu volume pode

atingir 4,0.10–3

m3. O trabalho realizado pelo ar

para encher essa bexiga, à temperatura

ambiente, realizado contra a pressão

atmosférica, num lugar onde o seu valor é

constante e vale 1,0.105 Pa, é

no mínimo de

a) 4 J.

b) 40 J.

c) 400 J.

d) 4000 J.

e) 40000 J.

Resolução: = p.∆V = 1,0.105. (4,0.10

–3 – 0) =

400 J. (Resposta C)

103) (UFMS –MS) Sem variar sua massa, um

gás ideal sofre uma transformação a volume

constante. É correto afirmar que

a) a transformação é isotérmica.

b) a transformação é isobárica.

c) o gás não realiza trabalho.

d) sua pressão diminuirá, se a temperatura do

gás aumentar.

e) a variação de temperatura do gás será a

mesma em qualquer escala termométrica.

Resolução: Como não ocorre variação do

volume, não ocorre a realização do trabalho.

(Resposta C)

104) (UFMT – Modificado) A energia interna U

de uma certa quantidade de gás, que se

comporta como gás ideal, contida em um

recipiente, é proporcional à temperatura T, e seu

valor pode ser calculado utilizando a expressão

U = 12,5T. A temperatura deve ser expressa em

kelvins e a energia, em joules. Se inicialmente o

gás está à temperatura T = 300 K, e em uma

transformação a volume constante, recebe 1250J

de uma fonte de calor, sua temperatura final será

51

a) 200 K.

b) 300 K.

c) 400 K.

d) 600 K.

e) 800 K.

Resolução: Q = + ∆U 1250 = 0 + 12,5∆T

(T – 300) = 1250/12.5 T = 400 K.

(Resposta C)

105) (ACAFE – SC) Em uma prova de Física,

numa das questões, os alunos devem calcular o

trabalho realizado por um gás ideal que, ao se

expandir, vai do estado inicial A, até o estado

final C. O gráfico apresenta a pressão, p, em

função do volume, V.

Alguns alunos encontraram como resultado,

10J. Eles:

a) erraram os cálculos, pois o resultado é 14J.

b) erraram os cálculos, pois o resultado é 8J.

c) acertaram o resultado.

d) erraram os cálculos, pois o resultado é 6J.

e) erraram os cálculos, pois o resultado é 12J.

Resolução: O trabalho é numericamente igual à

área sob o diagrama PV. Assim = AAB + ABC

= (4 + 2).2/2 + 4.1 = 6 + 4 = 10J. (Resposta C)

106) (CEFET – PR) Na seqüência são feitas

algumas proposições a respeito do

comportamento térmico de um gás:

I) Numa expansão adiabática, o gás perde calor

para o meio ambiente.

II) Numa compressão isotérmica, a energia

interna do gás não varia.

III) Numa transformação cíclica, o calor trocado

com o exterior é numericamente igual ao

trabalho realizado pelo gás.

IV) Uma quantidade de calor fornecida ao gás,

em parte, serve para que ele numa

transformação isobárica, realize trabalho e, em

parte, para que ele varie sua energia interna.

Assinale a alternativa correta sobre as

afirmações.

a) Somente as proposições I e II estão corretas.

b) Somente as proposições I, III e IV estão

corretas.

c) Somente as proposições III e IV estão

corretas.

d) Somente as proposições I, II e IV estão

corretas.

e) Somente as proposições II, III e IV estão

corretas.

Resolução:

I(F) Nas transformações adiabáticas não

ocorrem trocas de calor com o meio ambiente.

II(V) A energia interna é função da temperatura

e esta se mantém constante.

III) (V) Isso ocorre, pois ao completar cada

ciclo a energia interna se torna a mesma. Assim

o calor trocado é igual ao trabalho recebido ou

realizado.

IV(V) Q = + ∆U (Está de acordo com o 1º

princípio da Termodinâmica. (Resposta E)

107) Um gás ideal, confinado no interior de um

pistão com êmbolo móvel, é submetido a uma

transformação na qual seu volume é reduzido à

quarta parte do seu volume inicial, em um

intervalo de tempo muito curto. Tratando-se de

uma transformação muito rápida, não há tempo

para a troca de calor entre o gás e o meio

exterior. Pode-se afirmar que a transformação é

a) isobárica, e a temperatura final do gás é

maior que a inicial.

b) isotérmica, e a pressão final do gás é maior

que a inicial.

c) adiabática, e a temperatura final do gás é

maior que a inicial.

d) isobárica, e a energia interna final do gás é

menor que a inicial.

e) adiabática, e a energia interna final do gás é

menor que a inicial.

Resolução: A movimentação rápida do pistão

ocasiona uma redução no volume sem que haja

tempo para o sistema trocar calor com o meio

externo. Assim a transformação é adiabática.

Como o trabalho é negativo (compressão), a

variação da energia interna é positiva, o que

significa um aumento na temperatura ( = –∆U)

(Resposta C)

108) (CEFET – PR) Um gás, supostamente

ideal, sofre três evoluções sucessivas como

indicado.

Com relação a essa situação, são feitas as

afirmativas a seguir:

I) A temperatura do gás no estado B é maior do

que no estado A.

II) O trabalho realizado durante o ciclo é nulo.

III) O trecho CA corresponde a uma compressão

isométrica e o trabalho trocado nesse trecho

entre o gás e o meio é igual, em módulo, a 30J.

Podemos afirmar que:

a) apenas I é correta.

52

b) apenas I e II são corretas.

c) apenas II é correta.

d) todas são corretas.

e) todas são incorretas.

Resolução:

I(V) O produto pV para o ponto B (estado B) é

maior que para o ponto A (estado A).

II(F) O trabalho corresponde numericamente a

área do ciclo.

III(F) compressão isobárica. O volume diminui

e a pressão permanece constante. (Resposta A)

109) (ITA – SP) Um centímetro cúbico de água

passa a ocupar 1671 cm3

quando evaporado à

pressão de 1,0 atm. O calor de vaporização a

essa pressão é de 539cal/g. O valor que mais se

aproxima do aumento de energia interna da água

é

a) 498 cal

b) 2082 cal

c) 498 J

d) 2082 J

e) 2424 J

Resolução:

- 1cm³ de água possui 1g de massa quando

submetido a pressão normal.

- 1 g corresponde a 10-3

kg

- 1 cal corresponde a 4,18 J

- a pressão de 1 atm corresponde a 105 N/m²

- a variação de volume de 1670 cm³ corresponde

a 1,67.10–3

Assim: Q = + ∆U m.LV = p. ∆V + ∆U

10-3

. 539.4,18/10–3

= 105.1,67.10

–3 + ∆U

∆U = 2086,8 J 2082 J. (Resposta D)

110) (ENEM – Modificado) O gráfico a seguir

refere-se às curvas de distribuição de energia

cinética entre um mesmo número de partículas,

para quatro valores diferentes de temperatura

T1, T2, T3 e T4, sendo T1 < T2 < T3 < T4. Note

que as áreas sob cada uma das curvas são

idênticas, uma vez que são proporcionais aos

números de partículas.

As transformações químicas serão tanto mais

rápidas quanto maior for o número de colisões

possíveis.

Mas isso depende não só do valor do número de

colisões, mas também do valor mínimo da

energia, chamado energia de limiar ou de

ativação (por exemplo, a energia E indicada no

gráfico).

Assim, com relação ao gráfico apresentado, a

transformação química torna-se mais rápida na

seguinte temperatura:

a) T1

b) T2

c) T3

d) T4

Resolução: Na temperatura T4 observa-se que

um número reduzido de partículas possuem uma

grande quantidade de energia. O que lhes

faculta grande velocidade e conseqüentemente

um grande número de colisões tornando a

transformação mais rápida. (Resposta D)

111) (IFCE – CE) Um mol de um gás ideal

monoatômico se expande isotermicamente, à

pressão atmosférica, dobrando seu volume

inicial, que era de 1,0.10-3

m3. Neste processo, o

calor recebido pelo gás ideal foi de

a) 1,0.10² J.

b) 1,5.10² J.

c) 2,0.10² J.

d) 2,5.10² J.

e) 3,0.102 J.

Dado: use a pressão atmosférica p0 = 1,0.105 Pa.

Resolução: Nas transformações isotérmicas a

variação da energia interna do sistema (∆U) é

nula. Assim: Q = p.∆V + ∆U = 1.105(2.10

–3 –

1.10–3

) + 0 = 200J. (Resposta C)

112) (UFPE – PE) :Um gás ideal evolui de um

estado A (pressão 2p0 e volume V0) até um

outro estado C (pressão p0 e volume 2V0).

Considere os dois processos ABC e ADC,

indicados no diagrama a seguir. Sejam W(ABC) e

W(ADC) os trabalhos realizados pelo gás nestes

dois processos, respectivamente. Podemos

concluir que:

a) W (ABC) = 3W (ADC)

b) W (ABC) = 2W (ADC)

c) W (ABC) = W (ADC)

d) W (ABC) = 4W (ADC)

e) W (ABC) = W (ADC) = 0

Resolução: O trabalho é numericamente igual a

área sob a linha gráfica, sendo maior no trajeto

53

ABC. Ainda: ABC = 2p0.V0 e ADC = p0.V0

Portanto ABC = 2ADC . (Resposta B)

113) (PUC – RS) Uma amostra de gás hélio,

contida num cilindro com êmbolo, é

comprimida isotermicamente recebendo

trabalho de –500J, passando do estado (1,00

atm; 2,00L; 300K) para o estado (p2; 0,50L; T2).

Os valores de p2, T2 e da variação de energia

interna são, respectivamente,

a) 1,00atm, 600K e zero J

b) 1,00atm, 600K e 500J

c) 2,00atm, 300K e 500J

d) 4,00atm, 300K e –500J

e) 4,00atm, 300K e zero J

Resolução: Compressão isotérmica T = cte =

300K e como não houve variação da

temperatura, não houve variação da energia

interna ou seja ∆U = 0. A nova pressão será:

piVi/Ti = pfVf/Tf 1.2/300 = p2. 0,5/300

p2 = 4 atm. (Resposta E)

114) (CEFET – PR) Numa transformação

gasosa cíclica, em forma de quadrado de lados

paralelos aos eixos de um gráfico da pressão

absoluta de um gás, em função de seu volume

gasoso, podemos afirmar que ela apresenta:

a) duas transformações isobáricas e duas

isométricas.

b) duas transformações isotérmicas e duas

isométricas.

c) duas transformações adiabáticas e duas

isométricas.

d) duas transformações adiabáticas e duas

isobáricas.

e) duas transformações isobáricas e duas

adiabáticas.

Resolução: Os lados paralelos ao eixo das

pressões determinam volume constante

(transformações isométricas) enquanto os lados

paralelos ao eixo do volume determinam

pressão constante (transformações isobáricas).

(Resposta A)

115) (MACK – SP) Um gás, contido em um

recipiente dotado de um êmbolo que pode se

mover sofre uma transformação. Nessa

transformação fornecemos 800 cal ao gás e ele

realiza o trabalho de 209 J. Sendo 1 cal = 4,18 J,

o aumento da energia interna desse gás foi de:

a) 209 J

b) 3135 J

c) 3344 J

d) 3553 J

e) 3762 J

Resolução: Aplicando o 1º princípio da

Termodinâmica, temos: Q = + ∆U

800.4,18 = 209 + ∆U ∆U = 3135 J

(Resposta B)

116) (ACAFE – SC) O primeiro princípio da

termodinâmica estabelece uma relação entre as

energias interna, mecânica e calor, reafirmando

a idéia geral da conservação de energia.

Pode-se afirmar, portanto que, em uma

expansão adiabática de um gás ideal, ocorre a

transformação de:

a) energia mecânica em calor.

b) energia interna em energia mecânica.

c) calor em energia interna.

d) energia interna em calor.

e) energia mecânica em energia interna.

Resolução: Na expansão adiabática o sistema

não troca calor. Assim a energia mecânica usada

para a expansão do gás mediante uma

movimentação do pistão se dá as custas da sua

energia interna a qual, portanto, sofre uma

redução. (Resposta B)

117) (FGV-SP) - Pode-se afirmar que máquina

térmica é toda máquina capaz de transformar

calor em trabalho. Qual dos dispositivos pode

ser considerado uma máquina térmica?

a) Motor a gasolina b) Motor elétrico

c) Chuveiro elétrico d) Alavanca

e) Sarilho

Resolução: Máquina térmica transforma calor

em trabalho útil. Assim apenas o motor a

gasolina satisfaz tal condição. (Resposta A)

118) (UFRN – RN - Modificado) A figura

representa um pistão dentro da câmara de uma

bomba mecânica hermeticamente fechada.

Em dado momento, o pistão é deslocado

adiabaticamente até que o volume da câmara de

ar seja triplicado.

Com isso, determine a pressão no interior da

câmara em função da pressão inicial.

(Suponha a atmosfera formada por gases

diatômicos (γ = cp/cV = 1,4) e que 31,4

= 4,65)

Resolução: Lei de Poisson: p1 . V1 = p2 . V2

p1 . V1 = p2 .(3V1)

p1/p2 . = 3

γ p2

= p1 /3γ. e, sendo γ = 1,4, temos p2 = p1 /3

1.4 =

p1/31.4

= p1/4,65. (Resposta: p2 = p1 /4,65)

119) (PUC – RS) Uma certa quantidade de ar

contido num cilindro com pistão é comprimida

adiabaticamente, realizando-se um trabalho de

−1,5kJ. Portanto, os valores do calor trocado

com o meio externo e da variação de energia

54

interna do ar nessa compressão adiabática são,

respectivamente,

a) −1,5kJ e 1,5kJ.

b) 0,0kJ e −1,5kJ.

c) 0,0kJ e 1,5kJ.

d) 1,5kJ e −1,5kJ.

e) 1,5kJ e 0,0kJ.

Resolução: A quantidade de calor trocada entre

o sistema e o meio é nula (T. adiabática).

Assim, fica: Q = + ∆U 0 = + ∆U ∆U

= – ∆U = – (–1,5) = 1,5kJ. (Resposta

C)

120) (UFRJ – RJ) O gráfico a seguir representa

dois modos de levar uma certa massa de gás

ideal de uma temperatura inicial TA até uma

temperatura TC.

O primeiro (I) representa uma evolução a

pressão constante, e o segundo (II) uma

evolução a volume constante. O trabalho

realizado foi igual a 80 J.

a) Em qual dos dois processos foi necessário

ceder maior quantidade de calor à massa

gasosa? Justifique sua resposta.

b) Determine a quantidade de calor cedida a

mais.

Resolução: a) Pelo caminho I temos Q = +

∆U Q = 80 + ∆U e pelo caminho II temos Q

= + ∆U = 0 + ∆U. Como a variação da energia

interna (∆U) é a mesma pelos dois caminhos,

temos que através do caminho I foi necessário

ceder a maior quantidade de calor.

b) 80 J a mais, que é justamente o trabalho

realizado através do caminho I. (Resposta: a)

Caminho II. Devido ao trabalho realizado; b)

80 J)

121) (URCA) Uma amostra de um gás perfeito

passa do estado A para o estado B absorvendo

8,0.105

Joules de calor do meio exterior. O

aumento da energia interna, durante essa

transformação, é igual a:

a) 2.105 J

b) 3,5.105 J

c) 4,5.105 J

d) 8x105 J

e) 10.105 J

Resolução: O trabalho é numericamente igual à

área sob o diagrama. Assim: Q = + ∆U

8.105 = (3.10

5 + 6.10

5).1/2 + ∆U 8.10

5 =

4,5.105 + ∆U ∆U = 3,5.10

5 J. (Resposta B)

122) (IFBA –BA) Uma maquina térmica

funciona realizando, a cada segundo, quinze

ciclos idênticos ao ciclo representado na figura.

A potência da maquina, no SI, e igual a

a) 2,5.10³

b) 4,0.10³

c) 5,0.10³

d) 7,5.10³

e) 8,0.10³

Resolução: O trabalho é numericamente igual à

área do ciclo. Assim, Pot = /∆t = ((30.10–3

+

20.10–3

).2.104/2)/(1/15) = 2500 W = 2,5 . 10³ W

(Resposta A)

123) (UNB – DF) No diagrama adiante, a

energia interna do sistema, em J, é dada por U =

10 + 2PV, em que P é a pressão, em PA, e V, o

volume, em m³.

55

Calcule, em joules, a quantidade de calor

envolvida no processo AC, desprezando a parte

fracionária de seu resultado, caso exista.

Resolução: O trabalho é numericamente igual à

área sob o diagrama. Assim: Q = + ∆U Q

= (500 + 200).0,02/2 + ((10 + 2.200.0,03) – (10

+ 2.500.0,01) Q = 7 + (12 – 10) = 9 J.

(Resposta: 9 J)

124) (PUC – RS) Responder a questão seguinte

com base na figura a seguir, que representa as

variações da pressão de um gás, cujo

comportamento é descrito pela equação de

estado do gás ideal, em função do seu volume.

O gás passa sucessivamente pelos estados (1),

(2) e (3), retornando ao estado (1).

Considerando que entre os estados (1) e (2) a

transformação é adiabática, ocorre troca de calor

com o ambiente

a) somente entre (1) e (2).

b) somente entre (2) e (3).

c) somente entre (3) e (1).

d) entre (1) e (2) e entre (2) e (3).

e) entre (2) e (3) e entre (3) e (1).

Resolução: As trocas de calor não ocorrem nas

transformações adiabáticas. Assim, as duas

outras transformações (23 e 31) se dão com

o sistema trocando calor com o meio. (Resposta

E)

125) (UERJ –RJ) Um gás ideal se encontra em

um estado de equilíbrio termodinâmico A no

qual tem volume V0 e pressão p0 conhecidos. O

gás é então comprimido lentamente até atingir

um estado de equilíbrio termodinâmico B no

qual seu volume é V0/3.

Sabendo que o processo que leva o gás do

estado A ao estado B é o indicado pelo

segmento de reta do diagrama, e que os estados

A e B estão em uma mesma isoterma, calcule o

calor total QAB cedido pelo gás nesse processo.

Resolução: Como as temperaturas inicial e final

são iguais, não ocorre variação da energia

interna do estado inicial A ao final B. A pressão

pB corresponde a 3p0 já que o volume é reduzido

à terça parte. Assim, Q = + ∆U Q = -(3p0

+ p0)(2/3)V0/2 + 0 = -(4/3) p0V0. (Resposta: -

(4/3) p0V0)

126) (PUC – RS) Motores de potências

relativamente altas são Utilizados em

embarcações marítimas, locomotivas, geradores

e caminhões, tendo por base o ciclo Diesel de

quatro tempos. Esses motores, em geral, são

alimentados com a injeção direta do

combustível em cada cilindro. O gráfico abaixo,

da pressão em função do volume, representa

esquematicamente o ciclo Diesel, por meio de

seus quatro processos: compressão adiabática

AB, expansão isobárica BC, expansão

adiabática CD e transformação isovolumétrica

DA.

Considerando o ciclo Diesel apresentado no

gráfico,

a) não há variação de temperatura durante o

processo AB.

b) não há variação de temperatura durante o

processo DA.

c) a temperatura aumenta durante o processo

AB.

d) a temperatura aumenta durante o processo

CD.

56

e) a temperatura diminui durante o processo BC.

Resolução:

a(F): AB é transformação adiabática. Embora

não ocorram trocas de calor a temperatura varia.

b(F) a transformação DA é isovolumétrica. Com

a redução da pressão ocorre redução na

temperatura.

c(V) com a redução do volume o trabalho é

negativo e em conseqüência a variação da

energia interna é positiva o que significa uma

elevação na temperatura.

d(F) ocorre o contrário do ocorrido na

transformação AB.

e(F) se trata de uma expansão isobárica. O

volume aumenta com o aumento da

temperatura. (Resposta C)

127) (UFRN – RN) Manoel estava se

preparando para a “pelada” dos sábados, quando

notou que a bola de futebol estava vazia. Para

resolver essa pequena dificuldade, pegou uma

bomba manual e encheu a bola comprimindo

rapidamente o êmbolo da bomba.

Considerando que

-

termodinâmico;

-

após completar cada compressão; podemos

afirmar que, numa dada compressão,

a) a compressão do ar é um processo reversível.

b) o processo de compressão do ar é isotérmico.

c) a energia interna do ar aumenta.

d) a pressão do ar permanece constante durante

o processo.

Resolução: A movimentação rápida impõe que

a transformação sofrida pelo ar seja adiabática,

ou seja, não ocorrem trocas de calor com o

meio. Assim. O aumento da energia interna é

devido ao trabalho recebido pelo ar

(compressão). (Resposta C)

128) (ITA – SP) Uma certa massa de gás ideal

realiza o ciclo ABCD de transformações, como

mostrado no diagrama pressão-volume da

figura. As curvas AB e CD são isotermas. Pode-

se afirmar que

a) o ciclo ABCD corresponde a um ciclo de

Carnot.

b) o gás converte trabalho em calor ao realizar o

ciclo.

c) nas transformações AB e CD o gás recebe

calor.

d) nas transformações AB e BC a variação da

energia interna do gás é negativa.

e) na transformação DA o gás recebe calor, cujo

valor é igual à variação da energia interna.

Resolução:

a(F) o ciclo de Carnot é constituído por duas

transformações adiabáticas e duas isotérmicas.

b(F) Q = + ∆U Q = o gás converte

calor em trabalho.

c(F) são isotérmicas, o que significa temperatura

constante e conseqüentemente não ocorre

variação da energia interna. Em AB temos uma

expansão, ou seja, trabalho positivo e calor

recebido. Em CD temos uma compressão, ou

seja, trabalho negativo e calor cedido.

d(F) em AB não ocorre variação da energia

interna (t. isotérmica) e em BC a variação é

negativa tendo em vista uma redução na

temperatura (isoterma que passa por C mais

próxima dos eixos)

e(V) o volume é constante o que significa

trabalho nulo. Sendo Q = + ∆U, fica Q = ∆U

(Resposta E)

129) (UFF – RJ) O diagrama pressão (P) x

volume (V), a seguir, representa uma

transformação quase estática e cíclica de um gás

ideal.

Considere o diagrama e assinale a opção

correta.

a) A maior temperatura atingida pelo gás no

ciclo ocorre na passagem do estado 3 para o

estado 4.

b) O trabalho realizado pelo gás no ciclo é nulo.

c) A transformação que leva o gás do estado 2

para o estado 3 é isotérmica.

d) A variação da energia interna no ciclo é nula.

e) O gás sofre uma expansão adiabática ao

passar do estado 1 para o estado 2.

Resolução:

a(F) 3 para 4 é uma transformação isotérmica.

b(F) o trabalho é positivo tendo em vista que o

ciclo é horário.

c)(F) tendo em vista o sistema trocar de

isotermas significando alterar a temperatura.

d)(V) em uma transformação cíclica a variação

da energia interna, ao final de cada ciclo, é nula.

e)(F) expansão isotérmica. (Resposta D)

57

130) (UFRN – RN) Na cidade de Alto do

Rodrigues, está sendo construída a TermoAçu,

primeira usina termelétrica do estado com

capacidade para produzir até 70% da energia

elétrica total consumida no Rio Grande do

Norte. O princípio básico de funcionamento

dessa usina é a combustão de gás natural para

aquecer água que, uma vez aquecida, se

transformará em vapor e, finalmente, será

utilizada para mover as pás giratórias de uma

turbina. A produção da energia elétrica será feita

acoplando-se ao eixo da turbina algumas

bobinas imersas em um campo magnético.

Considere que, em cada ciclo dessa máquina

termelétrica real, se tenha: Q: o calor produzido

na combustão do gás; W: a energia mecânica

nas turbinas obtida a partir da alta pressão do

vapor acionando as pás giratórias; E: a energia

elétrica produzida e disponibilizada aos

consumidores. Para a situação descrita, é correto

afirmar:

a) Q = W = E

b) Q > W > E

c) Q = W > E

d) Q < W < E

Resolução: Aplicando o 1º princípio da

Termodinâmica, temos: Q = + ∆U onde Q =

W + E e como a energia Elétrica é obtida

através da movimentação das turbinas, temos

que Q > W >E. (Resposta B)

131) (UFPI – PI – Modificado) As máquinas a

vapor começaram a ser utilizadas, com algum

sucesso, no século XVII, apesar de sua história

ter começado com Heron, cerca de 100 d.C.

Desde então, várias máquinas a vapor de

diferentes modelos se destacaram pela

produtividade e diversidade de uso. O estudo

das máquinas térmicas chamou a atenção dos

físicos para uma série de transformações que

nunca ocorrem, embora não violem a lei da

conservação da energia. Essas “proibições”

deram origem à segunda Lei da Termodinâmica.

Com relação a este assunto, analise as

afirmativas abaixo e assinale V, para as

verdadeiras, ou F, para as falsas.

1 ( ) Para a construção de uma máquina térmica,

é sempre necessário haver duas fontes térmicas

em temperaturas diferentes, de modo que uma

parte do calor retirado da fonte quente é

rejeitado para a fonte fria.

2 ( ) Para uma máquina térmica que opera em

ciclos, é possível converter integralmente calor

em trabalho mecânico.

3 ( ) Há modelos de usinas termoelétricas nas

quais o movimento das turbinas a vapor é obtido

através do vapor de água à altíssima pressão.

Pode-se dizer que tais máquinas são máquinas

térmicas, pelo fato de transformarem parte da

energia interna de um gás em energia de

movimento das turbinas.

Resolução:

1(V) Trata-se do 2º princípio da

Termodinâmica.

2(F) É impossível uma máquina térmica ter

rendimento de 100%, ou seja, todo o calor

cedido pela fonte quente ser integralmente

utilizado para o trabalho sem haver rejeição à

fonte fria.

3(V) a máquina transforma calor (obtido através

da energia interna) em trabalho útil. (Respostas:

1(V); 2(F); 3(V))

132) (UFRN – RN) No radiador de um carro, a

água fica dentro de tubos de metal (canaletas),

como na figura abaixo.

Com a ajuda de uma bomba d’água, a água fria

do radiador vai para dentro do bloco do motor,

circulando ao redor dos cilindros. Na circulação,

a água recebe calor da combustão do motor,

sofre aumento de temperatura e volta para o

radiador; é então resfriada, trocando calor com o

ar que flui externamente devido ao movimento

do carro. Quando o carro está parado ou em

marcha lenta, um termostato aciona um tipo de

ventilador (ventoinha), evitando o

superaquecimento da água.

A situação descrita evidencia que, no processo

de combustão, parte da energia não foi

transformada em trabalho para o carro se mover.

Examinando-se as trocas de calor efetuadas,

pode-se afirmar:

a) Considerando o motor uma máquina térmica

ideal, quanto maior for o calor trocado, maior

será o rendimento do motor.

b) Considerando o motor uma máquina térmica

ideal, quanto menor for o calor trocado, menor

será o rendimento do motor.

c) Ocorre um aumento da entropia do ar nessas

trocas de calor.

d) Ocorrem apenas processos reversíveis nessas

trocas de calor.

58

Resolução: O calor rejeitado à fonte fria (ar)

contribui para um aumento da temperatura do

meio e conseqüentemente do nível de

desorganização, ou seja, da entropia. (Resposta

C)

133) (UFMA – MA) Uma maquina térmica,

operando em ciclos, recebe 800 J de calor e

realiza 240J de trabalho por ciclo.

Qual sua potencia útil em watts, sabendo-se que

a máquina opera com 30 ciclos por minuto?

a) 300

b) 240

c) 120

d) 360

e) 800

Resolução: Pot = /∆t = 30.240/60 = 120 W.

(Resposta C)

134) (URCA) Uma maquina que opera em ciclo

de Carnot tem a temperatura de sua fonte quente

igual a 330 °C e fonte fria a 10 °C. O

rendimento dessa maquina e melhor

representado por:

a) 53%

b) 5,3%

c) 0,53%

d) 46%

e) 0,46%

Resolução: = 1 – TF/TQ = 1 – 283/603 = 0,53

ou 53%. (Resposta A)

135) (ENEM) A invenção da geladeira

proporcionou uma revolução no aproveitamento

dos alimentos, ao permitir que fossem

armazenados e transportados por longos

períodos. A figura apresentada ilustra o

processo cíclico de funcionamento de uma

geladeira, em que um gás no interior de uma

tubulação é forçado a circular entre o

congelador e a parte externa da geladeira. É por

meio dos processos de compressão, que ocorre

na parte externa, e de expansão, que ocorre na

parte interna, que o gás proporciona a troca de

calor entre o interior e o exterior da geladeira.

Disponível em: http://home.howstuffworks.com.

(adaptado).

Nos processos de transformação de energia

envolvidos no funcionamento da geladeira

a) a expansão do gás é um processo que cede a

energia necessária ao resfriamento da parte

interna da geladeira.

b) o calor flui de forma não espontânea da parte

mais fria, no interior, para a mais quente, no

exterior da geladeira.

c) a quantidade de calor cedida ao meio externo

é igual ao calor retirado da geladeira.

d) a eficiência é tanto maior quanto menos

isolado termicamente do ambiente externo for o

seu compartimento interno.

e) a energia retirada do interior pode ser

devolvida à geladeira abrindo-se a sua porta, o

que reduz seu consumo de energia.

Resolução: Os processos de expansão e

compressão do gás circulante forçam (processo

não espontâneo) o calor fluir da parte mais fria

para a mais quente (máquina térmica reversa).

(Resposta B)

136) (IFCE – CE) Uma máquina térmica ideal

trabalha em um ciclo de Carnot entre as

temperaturas iguais a 300K e 450K. O trabalho

total, realizado pela máquina, em um ciclo, é

igual a 100J.

Calcule.

a) O calor retirado do reservatório quente pela

máquina.

b) O calor liberado para o reservatório frio pela

máquina.

Resolução:

a) O rendimento da máquina é: = 1 – TF/TQ =

1 – 300/450 = 1/3 ou 33% e, sendo = /QQ,

fica 1/3 =100/QQ QQ = 300 J.

b) = QQ – QF 100 = 300 - QF QF =

200 J. (Respostas: a) 300J; b) 200J)

137) (UFRN – RN) Até o século XVIII,

pensava-se que uma máquina térmica, operando

numa condição mínima de atrito, poderia

converter em trabalho útil praticamente toda a

energia térmica a ela fornecida.

Porém, Sadi Carnot (1796-1832) mostrou que,

em se tratando da energia fornecida a uma

máquina térmica, a fração máxima que pode ser

convertida em trabalho útil depende da

diferença de temperatura entre a fonte quente e a

fonte fria e é dada por:

= (T2 – T1)/T2

T1 é a temperatura da fonte fria, e T2 é a

temperatura da fonte quente.

Dessas afirmações, pode-se concluir que uma

máquina térmica

59

a) pode converter em trabalho útil toda a energia

térmica a ela fornecida, mesmo que funcione em

condições mínimas de atrito.

b) não pode converter em trabalho útil toda a

energia térmica a ela fornecida, mesmo que

funcione em condições mínimas de atrito.

c) pode converter em trabalho útil toda a energia

térmica a ela fornecida, desde que a temperatura

da fonte fria seja 0 oC.

d) não pode converter em trabalho útil toda a

energia térmica a ela fornecida, a menos que a

temperatura da fonte fria seja diferente de 0 oC .

Resolução: A máquina térmica jamais poderá

ter rendimento igual a 100% tendo em vista que,

nessa condição, a temperatura da fonte fria

deverá ser zero absoluto. Assim nunca poderá

converter integralmente o calor recebido em

trabalho útil. (Resposta B)

138) (VUNESP – SP) Uma geladeira retira, por

segundo, 1000 kcal do congelador, enviando

para o ambiente 1200 kcal. Considere 1 kcal =

4,2 kJ. A potência do compressor da geladeira

vale:

a) 700 KW

b) 800 kW

c) 840 kW

d) 600 kW

e) 500 kW

Resolução: Na geladeira temos = QQ – QF

= (-1200) – (-1000) = - 200 kcal. Assim a

potência do compressor deverá ser: Pot = /∆t =

200.4,2/1 = 840 W. (Resposta C)

139) (ITA – SP) Uma máquina térmica

reversível opera entre dois reservatórios

térmicos de temperaturas 100°C e 127°C,

respectivamente, gerando gases aquecidos para

acionar uma turbina. A eficiência dessa máquina

é melhor representada por

a) 68%.

b) 6,8%.

c) 0,68%.

d) 21%.

e) 2,1%.

Resolução: Supondo que a máquina obedeça ao

ciclo de Carnot, fica = 1 – TF/TQ = 1 –

373/400 = -0,0675 ou 6,75%. (Resposta B)

140) (UNIMEP - SP) Uma máquina térmica

executa 10 ciclos por segundo. Em cada ciclo

retira 800 J da fonte quente e cede 400 J à fonte

fria. Sabe-se que a máquina opera com a fonte

fria de 27°C. Com estes dados afirma-se que o

rendimento da máquina e a temperatura da fonte

quente valem, respectivamente:

a) 60% e 500 K

b) 50% e 600 K

c) 40% e 700 K

d) 30% e 327 K

e) 20% e 327°C

Resolução: Rendimento: = /QQ = (QQ – QF)/

QQ = (800 – 400)/800 = 0,5 ou 50%. A

temperatura da fonte quente, supondo que a

máquina obedeça ao ciclo de Carnot será: = 1

– TF/TQ 0,5 = 1 – 300/TQ TQ = 600 K.

(Resposta B)

141) (UFCE – CE) Uma máquina de Carnot,

reversível, é projetada para operar entre duas

fontes térmicas, seguindo o ciclo representado a

seguir.

O rendimento da referida máquina é

a) 10%

b) 20%

c) 25%

d) 50%

e) 80%

Resolução: = 1 – TF/TQ = 1 – (127 +

273)/(227 + 273) = 0,2 ou 20%. (Resposta B)

142) (UFRN – RN) Num dia quente de

verão, sem vento, com a temperatura

ambiente na marca dos 38oC, Seu

Onório teria de permanecer bastante tempo

na cozinha de sua casa. Para não sentir

tanto calor, resolveu deixar a porta do

refrigerador aberta, no intuito de esfriar a

cozinha. A temperatura no interior da

geladeira é de aproximadamente 0 oC.

A análise dessa situação permite dizer que

o objetivo de Seu Onório

a) será alcançado, pois o refrigerador vai

fazer o mesmo papel de um condicionador

de ar, diminuindo a temperatura da

cozinha.

b) não será atingido, pois o refrigerador

vai transferir calor da cozinha para a própria

cozinha, e isso não constitui um processo

de refrigeração.

c) será alcançado, pois, atingido o

equilíbrio térmico, a cozinha terá sua

temperatura reduzida para 19oC.

d) não será atingido, pois, com a porta do

refrigerador aberta, tanto a cozinha como o

próprio refrigerador terão suas

60

temperaturas elevadas, ao receberem calor

de Seu Onório.

Resolução: A temperatura da cozinha deverá se

elevar em virtude de o compressor retirar calor

da cozinha e o devolver à própria cozinha,

somando-se, ainda, o trabalho do compressor.

Uma solução para o senhor Onório seria

desligar a geladeira e deixar sua porta aberta.

(Resposta B)

143) (UFC – CE) A eficiência de uma máquina

de Carnot que opera entre a fonte de

temperatura alta (T1) e a fonte de temperatura

baixa (T2) é dada pela expressão h = 1 – (T2/T1),

em que T1 e T2 são medidas na escala absoluta

ou de Kelvin.

Suponha que você dispõe de uma máquina

dessas com uma eficiência h = 30%. Se você

dobrar o valor da temperatura da fonte quente, a

eficiência da máquina passará a ser igual a:

a) 40%

b) 45%

c) 50%

d) 60%

e) 65%

Resolução: Se a eficiência é 30% então a

relação entre as temperaturas será h = 1 –

(T2/T1) 30% = 1 - T2/T1 T2/T1 = 0,7. Ao

se dobrar a temperatura da fonte quente T1 a

relação fica T2/T1 = 0,7/2 = 0,35. Assim a

eficiência será h = 1 – (T2/T1) = 1 – 0,35 = 0,65

ou 65%. (Resposta E)

144) (CESCEM- SP) Um inventor informa ter

construído uma máquina térmica que recebe em

certo tempo 10³ calorias e fornece, ao mesmo

tempo, 5.10² calorias de trabalho útil. A

máquina trabalha entre as temperaturas de 177

ºC e 227 ºC. Nestas condições:

a) o rendimento desta máquina é igual ao da

máquina que executa o ciclo de Carnot;

b) o rendimento desta máquina é superado pela

máquina que executa o ciclo de Carnot;

c) a afirmação do inventor é falsa, pois a

máquina, trabalhando entre as temperaturas

dadas, não pode ter rendimento superior a 10%

d) mantendo as temperaturas dadas, pode-se

aumentar o rendimento, utilizando combustível

de melhor qualidade;

e) nada do que se afirmou é correto.

Resolução:

Rendimento da máquina: = /QQ = 500/1000

= 0,5 ou 50%. Rendimento da máquina de

Carnot operando entre as temperaturas dadas:

= 1 – TF/TQ = 1 – 450/500 = 0,1 ou 10%.

(Resposta C)

145) (UFC – CE) A figura mostra um ciclo de

Carnot, representado no diagrama pxV. Se no

trecho bc, desse ciclo, o sistema fornece 60 J

de trabalho ao meio externo, então é verdade

que, nesse trecho

T1

T2

a

b

c

d

p

V

a) o sistema recebe 60 J de calor e sua energia

interna diminui.

b) o sistema recebe 60 J de calor e sua energia

interna não varia.

c) o sistema rejeita 60 J de calor e sua energia

interna não varia.

d) não há troca de calor e sua energia interna

aumenta de 60 J.

e) não há troca de calor e sua energia interna

diminui de 60 J.

Resolução: O trecho BC representa uma

transformação adiabática onde não ocorrem

trocas de calor. Assim a expansão do gás ( > 0)

se dá as custas de uma diminuição da sua

energia interna (∆U < 0). (Resposta E)

146) (IME - RJ) Uma fonte calorífica, mantida a

uma temperatura de 327 ºC, transfere 1000

calorias de calor para o meio ambiente a 27 ºC.

O trabalho máximo, em kcal, que se poderia

obter da fonte calorífica considerada é:

a) 600 kcal b) 1,5 kcal c) 3,0 kcal

d) 427 kcal e) 0,5 kcal

Resolução: Máximo rendimento será: = 1 –

TF/TQ = 1 – 300/600 = 0,5 ou 50%. Assim, =

/QQ 0,5 = /1000 = 500 cal ou 0,5

kcal. (Resposta E)

147) (MED. ABC – SP – Modificado) Uma

dona de casa no Alasca deseja manter o interior

de sua cozinha à temperatura t1 = 27 ºC. A

temperatura externa é t2 = - 23 ºC. Nessas

condições, as paredes (diatérmicas) da cozinha

perdem calor para o ambiente externo à razão de

18 kW. Esse calor deve ser reposto por uma

termo bomba (aquecedor).

Determine, em kW, o menor consumo teórico

de potência da termo bomba.

Resolução: = 1 – TF/TQ = 1 – 250/300 = 1/6

e, sendo = 1 – QF/QQ , temos, a cada segundo

1/6 = 1 – 18/ QQ QQ = 21,6 kJ. Assim, =

/ QQ 1/6 = /21,6 = 3,6 J e

potência igual a 3,6 kW. (Resposta: 3,6 kW)